FLK1 Business Flashcards

1
Q

What are cumulative preference shares?

A

If profits are sufficient to call dividends, the preference shareholder has to be paid any missed dividends from previous years as well as any from the current financial year.

This right ranks before the payment of dividends to ordinary shareholders in the current financial year.

How well did you know this?
1
Not at all
2
3
4
5
Perfectly
2
Q

What is a transaction at an undervalue?

Corporate insolvency

A

This is where the company makes a gift to another person or enters into a transaction and receives consideration which is significantly lower in value than market value.

A liquidator/administrator can challenge any transaction which the company has entered into at an undervalue at the relevant time.

How well did you know this?
1
Not at all
2
3
4
5
Perfectly
3
Q

What decisions does the board have to make with regards to the allotment of shares?

A

The board will decide the price and how many shares it wishes to allot, usually having taken the advice of the company’s accountant or banker.

Solicitors advise on the procedure needed to allot shares.

How well did you know this?
1
Not at all
2
3
4
5
Perfectly
4
Q

What are the main types of security for LLPs and companies?

A

Usually mortgages, fixed and floating charges.

How well did you know this?
1
Not at all
2
3
4
5
Perfectly
5
Q

What are book debts? Can they be charged, if so, how?

A

These are debts owed to the company. They are an asset and can be charged.

Floating charge over the book debts:

As book debts vary over time, they are suitable for a floating charge. If the charge holder allows the company/LLP to use the proceeds from the book debts for its business purposes then this would indicate a floating charge.

Fixed charge over book debts:

Book debts can also be secured by a fixed charge where the charge holder had control over both the debts and the proceeds once they were paid.

This could arise where the charge holder allowed the company/LLP to collect the book debts but then the company/LLP had to pay over the money to the charge holder to settle part of the debt owed to the charge holder.

How well did you know this?
1
Not at all
2
3
4
5
Perfectly
6
Q

What is calculated at Step 4 of a corporation tax calculation?

A

Steps 1 to 3 establish the company’s taxable profit for the accounting period. At Step 4, the appropriate tax rate is applied to taxable profit, to calculate tax payable.

The corporation tax financial year runs from 1 April to 31 March. If the company’s accounting period – its financial year – is different from the corporation tax financial year, and the corporation tax rate changes from one corporation tax year to the next, the company will have to pay tax at one rate on a proportion of its profits and at the new rate on the rest of its profits for the financial year.

How well did you know this?
1
Not at all
2
3
4
5
Perfectly
7
Q

Why operate as a general partnership?

A

a)No setting up formalities

As soon as two people start running a business together they are likely to be a partnership even if they are not aware of this.

b) No administration formalities

Do not have to adhere to extensive administrative and accounting requirements and make this information public

c) Less time and money spent on legal and accounting

This is because they do not need to file public accounts. Partners can concentrate on their trade rather than spending time and money on legal and administrative matters.

d) Tax advantages

This depends on the business of the partnership and the partners’ individual circumstances.

How well did you know this?
1
Not at all
2
3
4
5
Perfectly
8
Q

In a CVA, 75% or more in value of the company’s creditors and
50% or more of non-connected creditors must approve a CVA proposal.

Who decides whether the creditors are connected or not?

A

The chair of the CVA meeting, usually the Insolvency Practitioner.

How well did you know this?
1
Not at all
2
3
4
5
Perfectly
9
Q

Before borrowing money, what should the directors do?

A

a) Check constitution

Private company incorporated under CA 2006 = there will be no restrictions on borrowing.

Company formed before 1 October 2009 and not updated articles = need to check there are no restrictions.

Restriction in articles = if there are any restrictions on borrowing money in the company’s articles or memorandum, the shareholders will need to pass a special resolution to change the articles to remove the restrictions.

This also applies to the memorandum as it will be treated as part of the constitution.

b) Check whether directors have authority to act on behalf of the company

Company with MA = the authority will come from MA 3.

Company with amended or bespoke articles = the articles need to be checked for restrictions or conditions.

Partnerships and debt finance:

Need to check the partnership agreement before borrowing.

If there is a provision, this can only be changed by unanimous consent.

How well did you know this?
1
Not at all
2
3
4
5
Perfectly
10
Q

What is the director’s duty to exercise independent judgement?

A

Self-explanatory. This is not infringed by the director acting in accordance with an agreement entered into by the company that restricts future exercise of discretion by its directors or in a way that is authorised by the company’s constitution.

The duty always comes first.

How well did you know this?
1
Not at all
2
3
4
5
Perfectly
11
Q

In terms of post-decision requirements, what must be filed at companies house?

A

a) Correct forms
b) Copies of all special resolutions
c) Some ordinary resolutions must be filed

How well did you know this?
1
Not at all
2
3
4
5
Perfectly
12
Q

What are charges?

A

This form of security does not transfer the legal ownership from the chargor to the charge holder and does not given them the right to immediate possession.

A charge gives the lender rights over the asset should the borrower fail to repay the money borrowed.

There are two types: floating charge and fixed charge.

How well did you know this?
1
Not at all
2
3
4
5
Perfectly
13
Q

How many directors does a private company need to have?

A

1

How well did you know this?
1
Not at all
2
3
4
5
Perfectly
14
Q

What additional requirements need to be met for a buyback out of capital?

Additional to requirements for share buyback out of distributable profit

A

Statement of solvency

Auditors report

Special resolution

Copy of the directors’ statement of solvency and auditors’ report must be available to members at GM/with WR

Notice in London Gazette

Notice in national newspaper

Filing copy of directors’ statement and auditors’ report at CH

Directors’ statement and auditors’ report kept available for inspection

Board meeting

Payment out of capital

How well did you know this?
1
Not at all
2
3
4
5
Perfectly
15
Q

What is the director’s duty not to accept benefits from third parties?

A

A director of a company must not accept a benefit from a third party conferred on them by reason of them either being a director or doing/not doing anything as a director.

There is no breach if the acceptance of the benefit cannot reasonably be regarded as likely to give rise to a conflict of interest.

Normal corporate hospitality is not caught by this duty as long as, viewed objectively, it cannot be reasonably regarded as likely to give rise to a conflict of interest.

How well did you know this?
1
Not at all
2
3
4
5
Perfectly
16
Q

How can a CVA be used in relation to a moratorium?

A

The CVA does not affect the rights of secured and preferential creditors, unless they agree to it. Small companies used to be entitled to a moratorium of 28 days after filing the CVA proposal at court but this is no longer the case. However, companies could use the moratorium created by CIGA 2020 as well as entering into the CVA, and the moratorium would give the company the breathing space it needed to give the company the chance to enter into the CVA.

How well did you know this?
1
Not at all
2
3
4
5
Perfectly
17
Q

What is the order of distribution of assets in administration?

A
  1. Fixed charges holders

These are paid first. They will receive the amount they are owed when the asset subject of the fixed charge was sold.

Any surplus is paid to the liquidator.

What happens if there is a shortfall once the asset has been sold?

If there is a shortfall instead, the fixed charge holder can join the pool of unsecured creditors and try and obtain some kind of contribution towards the outstanding debt.

  1. Unsecured creditors

Admin:

The liquidator will send a standard form to unsecured creditors. They fill this in outlining the debts that are owed to them to prove their debt.

The liquidator then approves or rejects the creditor’s debts.

If the claim is under the sum of £1,000 these claims are admitted automatically.

Once the liquidator has told the company’s assets and collected as much money as they can and paid the holders of valid fixed charges, they will make the payments in the following order.

Order of distribution:

a) Expenses of the winding up (fees payable to the liquidator and their professional advisors)
b) Preferential debts which rank and abate equally
c) Money which is the subject of a floating charge in order or priority
d) Unsecured creditors who rank and abate equally.
e) Remaining money is distributed to shareholders

What are preferential creditors?

The most common preferential debt are the wages/salaries of employees for work carried out in the four months immediately preceding the date of the winding up order.

There is a maximum of £800 per employee.

Employees’ accrued holiday pay is a preferential debt.

HMRC is a preferential creditor in relation to PAYE and VAT but only these.

How well did you know this?
1
Not at all
2
3
4
5
Perfectly
18
Q

What is an auditor’s liability?

A

They do not owe a duty of care to either the shareholders or to potential new shareholders when conducting their annual audit.

For liability there has to be proximity between the relevant parties.

Auditors can be sued for negligence by the company.

How well did you know this?
1
Not at all
2
3
4
5
Perfectly
19
Q

When can a wrongful trading claim be brought?

A

Such claims can only be brought when a company is in insolvent liquidation or insolvent administration.

As a remedy, the court may order the director to make a contribution to the company’s assets, increasing the amount of money available to pay creditors.

How well did you know this?
1
Not at all
2
3
4
5
Perfectly
20
Q

What checks on the company does the lender need to make before agreeing to a loan?

A

Inspecting the articles of the company, searching the company’s records at Companies House and requesting copies of relevant board resolutions.

The lender should also search the company records at CH to see if any charges have been registered against it already and ensure that there is sufficient value in that property to provide adequate security for the proposed loan.

From the register the lender can find out:
a) Date of creation of any existing charge
b) The amount secured
c) Which property is subject of the charge
d) Who holds that charge

If the charge is over land, the lender should also conduct a search at LR to check the company’s title to the land and to see if any pre-existing charges have been registered.

The lender should conduct a winding up search at the Companies Court to check that no insolvency proceedings have been commenced against the company.

How well did you know this?
1
Not at all
2
3
4
5
Perfectly
21
Q

Under PA 1890, the firm is bound by any contract or deed entered into by partners in the firm’s name, provided that the partner’s actions were authorised by the partners.

How may an action may be authorised?

A

a) The partners may have acted jointly in making the contract
b) Express actual authority
c) Implied actual authority - if all partners are involved in the running of the business without limitations, it will be implied that each partner has authority e.g. to sell the firm’s products in the ordinary course of business. Or authority may be implied by a regular course of dealing by one of the partners to which the other partners have not objected.

How well did you know this?
1
Not at all
2
3
4
5
Perfectly
22
Q

In public companies, what number of people are required to call a general meeting on short notice?

A

A majority of number of company’s shareholders who between them hold 95% or more of the company’s voting shares must consent.

How well did you know this?
1
Not at all
2
3
4
5
Perfectly
23
Q

What is the default position under statute regarding a member leaving an LLP and what filing requirements are there?

A

Members can leave the LLP by giving reasonable notice to the other members.

Members cannot be expelled so if they wish there to be a right of expulsion, this needs to be included in the LLP e.g. for bankruptcy.

Filing:

LLP must notify Companies House on form LL TM01 within 14 days of the member leaving.

How well did you know this?
1
Not at all
2
3
4
5
Perfectly
24
Q

What should be included in a PA regarding work input?

A

Under PA 1890, partners may take part in the management of business but they are not required to do so.

The partnership agreement should set out each of the partner’s working hours or state that they must work full-time for the business so that it is clear what is required of each partner.

A common clause in a partnership agreement is that a partner must devote the whole of their time and attention to the business and partners must not engage in any other business whilst they are partner.

Non compete clauses are common although in the absence of an express agreement, one will be implied by default under PA 1890.

The agreement should set out holiday entitlement, sickness and maternity and paternity provisions. These matters are not provided for in PA 1890.

How well did you know this?
1
Not at all
2
3
4
5
Perfectly
25
Q

How does voting work on a show of hands work in general meetings?

A

On show of hands and each shareholder has one vote.

Shareholders are not prevented from counting in the quorum or voting if they have a personal interest in the matter.

However, there are two key shareholder resolutions where the votes of a shareholder with a personal interest in the matter are not counted.

How well did you know this?
1
Not at all
2
3
4
5
Perfectly
26
Q

What are the formalities after redemption of a loan?

A

When a loan secured by a registered charge is repaid by the borrower to the lender, a person who originally had an interest in the registration of the charge may complete sign and send form MR04 to the Registrar of Companies at Companies House.

This will be done to ensure the company file is up to date.

The Registrar will include a statement of satisfaction on the company’s file.

If any entries were made against the land at the Land Registry, these should be removed.

How well did you know this?
1
Not at all
2
3
4
5
Perfectly
27
Q

Which two things must company officers do after company decisions have been made either by board resolution or shareholder resolution?

A

Filing at companies house

Internal administration

How well did you know this?
1
Not at all
2
3
4
5
Perfectly
28
Q

What is an advantage of administration?

A

The main advantage of the administration is that while the administration is underway, there is a statutory moratorium.

This means that it is not possible for anyone to commence or continue with legal action against the company, enforce a judgment or issue a winding up petition without the administrator’s consent.

This gives the administrator time and space to investigate the company, assess its viability and maximise the amount of money available to creditors.

How well did you know this?
1
Not at all
2
3
4
5
Perfectly
29
Q

What is the deadline for filing accounts for newly incorporated companies?

A

They have the option of filing accounts and reports three months after the end of the company’s first accounting reference period instead.

How well did you know this?
1
Not at all
2
3
4
5
Perfectly
30
Q

Is a trading certificate required when converting from a private company to a public company?

A

A trading certificate is not required. Only need new certificate of incorporation.

How well did you know this?
1
Not at all
2
3
4
5
Perfectly
31
Q

Comparing debt and equity finance, what are the restrictions on sale of shares and sale of debenture?

A

Equity finance

The transfer of shares is governed by the company’s articles and a private company usually restricts the shareholder’s freedom to sell their shares.

Debt finance

If a lender wishes to realise its capital earlier than the repayment date agreed, it may sell its debenture to a third party.

No restriction in the articles will affect its right to sell.

How well did you know this?
1
Not at all
2
3
4
5
Perfectly
32
Q

Can a director of a company be a company secretary?

A

Yes

How well did you know this?
1
Not at all
2
3
4
5
Perfectly
33
Q

What are the steps in calculation of corporation tax?

A

Step 1: Calculate income profits

Step 2: Calculate chargeable gains:
Stage 1: Identify a chargeable disposal
Stage 2: Calculate the gain (or loss)
Stage 3: Apply reliefs

Step 3: Calculate total profits and apply any available reliefs against total profits

Step 4: Calculate the tax

How well did you know this?
1
Not at all
2
3
4
5
Perfectly
34
Q

What must be sent to companies house when a director resigns or is removed?

A

They must complete form TM01 (as an individual) or TM02 (company) within 14 days of resignation.

Look out for a clause in the TM01 that gives the company power of attorney to complete the TM01 on the director’s behalf.

How well did you know this?
1
Not at all
2
3
4
5
Perfectly
35
Q

In private companies, what number of people are required to call a general meeting on short notice?

A

A majority of number of company’s shareholders who between them hold 90% or more of the company’s voting shares must consent.

How well did you know this?
1
Not at all
2
3
4
5
Perfectly
36
Q

What are novation contracts?

A

A novation contract is used when a retiring partner wants to be released from an existing debt. The creditor will release the original partners from liability under the contract and the firm is as newly constituted takes over liability.

This is advantageous for an outgoing partner but disadvantageous for an incoming partner.

If a partner retires and no new partner joins, in order to ensure that the novation is contractually binding, there must be consideration for the creditor’s promise to release the retiring partner from the liability or the contract must be executed as a deed.

The creditor would likely not agree to this as it decreases the number of people the creditor can sue. This is not a common solution so should not be overstated and are almost only used for ongoing liabilities such as loans and not used for one-off debts.

How well did you know this?
1
Not at all
2
3
4
5
Perfectly
37
Q

How is the process of creditors’ voluntary liquidation initiated?

A

The process is initiated by the company, through discussion and agreement between the company’s directors and shareholders and the creditors then take over at an early stage.

Technically voluntary but the directors will feel pressured to enter into a CVL by the creditors.

And they will be aware of potential personal claims for misfeasance and fraudulent/wrongful trading if they continue to trade and the company goes into liquidation.

How well did you know this?
1
Not at all
2
3
4
5
Perfectly
38
Q

What is the quorum for a GM?

A

Two (subject to articles).

How well did you know this?
1
Not at all
2
3
4
5
Perfectly
39
Q

What factors should be considered when choosing a business medium?

A

Liability

Tax

Formalities

Publicity of information

Cost

Status

Finance

How well did you know this?
1
Not at all
2
3
4
5
Perfectly
40
Q

Who is a person with significant control?

A

If they hold more than 25% of the shares in the company.

If they hold more than 25% of voting rights in the company.

If they hold the right to appoint or remove a majority of the board of directors of the company.

How well did you know this?
1
Not at all
2
3
4
5
Perfectly
41
Q

What needs to be considered/changed by directors after buying a shelf company?

A

Chairperson (decide whether to have one, if so by board resolution)

Bank account (who can spend the money and how much in one go, need to complete a bank mandate to outline these decisions)

Company seal* (not mandatory but can be good idea)

Changing company name/business name

Accounting reference date

Auditor (if required to file annual accounts)

Service contracts

Tax registrations (HMRC automatically informed and sends introductory pack to company)

Corporation tax form to HMRC in introductory pack
PAYE and NI (directors need to register the company with HMRC to arrange for the deduction of income tax from salaries under the PAYE scheme and for payment of NI by the employees - online via gov website)

VAT

Insurance

Shareholders agreement

How well did you know this?
1
Not at all
2
3
4
5
Perfectly
42
Q

What are the requirements that must be met for a partner to escape liability for debts that have been incurred after they leave the partnership?

A

a) Anyone who has dealt with the firm before must be given actual notice of the partner in question leaving. This means they must be informed directly. If notice is not given, the person is entitled to treat all apparent members of the firm as still being members.
b) Notice placed in London Gazette (relevant newspaper for NI or Scotland rather than England and Wales) to notify anyone who has not had dealings with the firm before the partner in question left of their retirement.

If the reason for ceasing to be a partner is due to death or bankruptcy rather than retirement or expulsion, no notice is required.

The estate of the deceased or bankrupt partner is not liable for partnership liabilities incurred after the death or bankruptcy.

How well did you know this?
1
Not at all
2
3
4
5
Perfectly
43
Q

What is income?

A

Money will be regarded as income if there is an element of recurrence, for example, a salary received every month, or interest received on a bank account.

How well did you know this?
1
Not at all
2
3
4
5
Perfectly
44
Q

When are final accounts prepared?

A

The final accounts are prepared once a year, at the end of the business’s accounting period.

How well did you know this?
1
Not at all
2
3
4
5
Perfectly
45
Q

Is there a time limit for bringing a claim for defrauding creditors?

A

No

How well did you know this?
1
Not at all
2
3
4
5
Perfectly
46
Q

How is a restructuring plan implemented?

A

Two court are held hearings.

First hearing:

The creditors can make representations.

Second hearing:

The court will decide whether to sanction the proposed plan.

Creditor and shareholder meetings

These are held in between the two hearings.

The creditors and shareholders are divided into classes and each class will be deemed to have approved the plan if 75% by value of that class vote in favour.

How well did you know this?
1
Not at all
2
3
4
5
Perfectly
47
Q

What is meant by apparent authority?

A

This is where the director acts without the company’s prior consent whether express or implied but this still binds the company.

The company is estopped from denying the director’s authority.

It is based on a representation by the company to the third party, by words or conduct, that the director is acting with the company’s authority.

It is not the director’s actions that are significant it is the company’s omission to correct the impression.

How well did you know this?
1
Not at all
2
3
4
5
Perfectly
48
Q

What are non-cumulative preference shares?

A

If a dividend payment is missed, the shareholder loses the right to that year’s dividend and does not have the right to receive it in the future.

How well did you know this?
1
Not at all
2
3
4
5
Perfectly
49
Q

What is a partners’ liability for partnership debts if the debt was incurred after leaving the partnership?

A

The partner remains liable for the debts incurred whilst they were partner once they leave the partnership.

They will escape liability for debts that have been incurred after they leave as long as they comply with certain requirements.

How well did you know this?
1
Not at all
2
3
4
5
Perfectly
50
Q

Which court does a creditor bring a bankruptcy petition to? What else must the creditor arrange?

A

The creditor must usually present the petition at the debtor’s local county court hearing centre as long as it has jurisdiction in bankruptcy matters.

The creditor must also arrange for personal service of the petition by engaging an agent who will hand the petition to the debtor and provide a witness statement confirming that they have done so.

If the debtor is avoiding the agent, the creditor can seek to obtain a court order for substituted service which means that another method of service will be deemed sufficient service.

How well did you know this?
1
Not at all
2
3
4
5
Perfectly
51
Q

What are the exceptions to the doctrine of maintenance of share capital?

A

A company can buy back its own shares as long as the correct procedure is followed

A company can buy back its own shares under a court order following a successful unfair prejudice claim

A company can return capital to shareholders after payment of company debts in a winding up

How well did you know this?
1
Not at all
2
3
4
5
Perfectly
52
Q

What requirements are there for the name of an LLP?

A

The LLP must have LLP in its name or Welsh equivalents.

Similar restrictions to the names of companies.

The LLP must have its name on the outside of its place of business and its stationary must state its name, place of registration and registration number and address of registered office.

How well did you know this?
1
Not at all
2
3
4
5
Perfectly
53
Q

What types of credit facility are there?

A

Secured or unsecured. Bilateral or syndicated.

How well did you know this?
1
Not at all
2
3
4
5
Perfectly
54
Q

How is a compulsory liquidation commenced?

A

Creditors will usually prove a debtor’s insolvency by issuing a statutory demand and if the statutory demand remains unpaid after three weeks, issuing a winding up petition against the company.

They do not have access to detailed financial up to date information as public accounts only need to be filed once a year so this is the only way they can prove insolvency other than direct correspondence with the debtor.

Alternatively, they can obtain a judgment against the company as the test of whether a company is insolvent is the inability to pay debts as and when they fall due. If they cannot satisfy the judgment they will be insolvent.

How well did you know this?
1
Not at all
2
3
4
5
Perfectly
55
Q

What is rollover relief?

A

This relief enables sole traders and partners to sell certain assets (‘qualifying business assets’) without paying CGT, provided the proceeds of sale are invested in other qualifying business assets.

The seller will have to pay tax eventually, but the charge to CGT is postponed until the seller disposes of the new asset(s).

  1. Which assets are qualifying business assets?
  • Land
  • Buildings and
  • Goodwill.

The asset must be used in the trade of the business rather than being held as an investment.

Fixed plant and machinery are qualifying business assets, but their sale usually results in a loss because they are wasting assets.

  1. Which assets are not qualifying business assets?

There is no definition of ‘fixed’, but if the asset is moveable, it is unlikely to be classed as fixed. Company shares are not qualifying assets.

  1. When does the relief apply?

The relief applies when a qualifying asset is disposed of, and the asset is owned by:
a) a sole trader, who uses the asset in their trade;
b) a partnership, which uses the asset in its trade;
c) an individual partner, where the partnership uses the asset in the partnership trade; or
d) an individual shareholder, where the asset is used in the trade of the company in which the shareholder owns shares. For this to apply, the company must be their ‘personal company’, meaning that the shareholder must own at least 5 per cent of the voting shares in the company.

  1. Do they have to be the same type of asset?

Provided that both the asset disposed of and the asset acquired fall within the definition of qualifying assets, they do not have to be the same type of asset. For example, it is possible to sell qualifying goodwill, and rollover the gain into the purchase of qualifying buildings.

  1. What is the time limit for acquiring the replacement asset?

The taxpayer must acquire the replacement asset within one year before or three years after the disposal of the original asset, unless HMRC allows the taxpayer an extended time period to claim.

  1. How is the relief applied?

The taxpayer must claim the relief within four years from the end of the tax year in which they acquire the replacement asset (or, if later, within four years from the end of the tax year in which the original asset is sold).

The gain is notionally deducted from the acquisition cost of the replacement asset, which gives a lower acquisition cost to use in CGT calculations when the asset is disposed of in the future.

This means that a later disposal of the replacement asset is likely to produce a gain that includes both the rolled- over gain and any gain on the replacement asset itself.

  1. Disadvantage of rollover relief

Note that the annual exemption cannot be used before the gain is rolled over, so the taxpayer loses the benefit of the annual exemption if they apply for rollover relief.

How well did you know this?
1
Not at all
2
3
4
5
Perfectly
56
Q

How can a chair person be appointed?

A

Can be appointed by an board resolution of the directors.

How well did you know this?
1
Not at all
2
3
4
5
Perfectly
57
Q

How can rollover relief be applied in a group company to convey a tax advantage?

A

When a company is in a group for chargeable gains purposes and it disposes of a chargeable asset outside the group, it can roll over its gain into qualifying assets that it acquires (provided it satisfies the criteria for rollover relief).

As an alternative, it can roll over its gain into qualifying assets acquired by another company in the same group.

Group relief under chargeable gains provisions are subject to certain anti- avoidance provisions. These are designed to stop companies using the rules to their benefit when they join or leave a group.

How well did you know this?
1
Not at all
2
3
4
5
Perfectly
58
Q

What are the liquidator’s powers?

A

a) Carrying on the company’s business
b) Commencing and defending litigation on the company’s behalf
c) Investigating the company’s past transactions
d) Investigating the directors’ conduct
e) Collecting and distributing the company’s assets
f) Doing all that is necessary to facilitate the winding up of the company
g) Selling company assets and distributing money to the company’s creditors
h) Preserving and increasing assets

How well did you know this?
1
Not at all
2
3
4
5
Perfectly
59
Q

What is administration?

A

This is the process whereby an administrator is appointed to run the company and make whatever changes are necessary to improve its financial performance.

Alternatively, the administrator will aim to get the company into a position where it can be sold as a going concern.

How well did you know this?
1
Not at all
2
3
4
5
Perfectly
60
Q

Can a partnership offer a security for a loan by way of a floating charge?

A

No

How well did you know this?
1
Not at all
2
3
4
5
Perfectly
61
Q

What is capital gains tax?

A

Capital gains tax (‘CGT’) is payable on chargeable gains made by a chargeable person on the disposal of chargeable assets in a tax year, which runs from 6 April one year until 5 April the following year.

How well did you know this?
1
Not at all
2
3
4
5
Perfectly
62
Q

What is carry forward relief?

CGT Tax

A

A taxpayer may carry forward their trading loss for a tax year and set it against subsequent profits which the trade produces in subsequent years, taking earlier years first.

Losses can be carried forward indefinitely until the loss is exhausted so if several years go by before the trade makes a profit against which to set the losses, this is no bar to claiming the relief.

Whilst the losses can be carried forward indefinitely, the tax payer must notify HMRC of its intention to claim the relief no more than four years after the end of the tax year in which the loss was incurred.

How well did you know this?
1
Not at all
2
3
4
5
Perfectly
63
Q

What are the disadvantages of an unfair prejudice claim?

A
  • Expensive and time consuming.
  • Claimant will have to gather evidence and some will be difficult to obtain.
  • The court will need a lot of evidence before it decides that the claimant has been unfairly prejudiced.
  • The parties may need an expert report.
How well did you know this?
1
Not at all
2
3
4
5
Perfectly
64
Q

What needs to be considered when allotting shares?

A

a) Are there any constitutional restrictions on allotment?
b) Do the directors have authority to allot shares?
c) Are there any pre-emption rights?

How well did you know this?
1
Not at all
2
3
4
5
Perfectly
65
Q

What happens if the Registrar of Companies has all the information/documents necessary to incorporate a company?

A

If the Registrar of Companies is satisfied that the requirements have been met they will incorporate the company and issue a certificate of incorporation. The companies comes into existence upon the certificate of incorporation being issued.

How well did you know this?
1
Not at all
2
3
4
5
Perfectly
66
Q

How is the income payable by a partner in a partnership assessed?

A

Calculating the income payable by a partner in a partnership comprises the following steps:

  1. The partnership’s trading profit will be calculated in the same way as trading profit for a sole trader.

Chargeable receipts LESS deductible expenditure LESS capital allowances = trading profit/ loss

  1. The trading profit is then shared between the partners in accordance with their agreement (or, if there is no agreement, the Partnership Act 1890).

There are two elements to this: the agreement may well set out what will be paid first, for example salaries, interest on capital, and finally any remaining profit. The agreement should also set out each partner’s percentage share of the profits.

  1. Each partner will include this figure on their tax return and will be assessed in the ordinary way for income tax, taking account of any applicable reliefs and allowances.

If the partnership makes a trading loss instead, the losses will again be shared between the partners in accordance with their agreement, and the partners can each choose how they will
claim any applicable reliefs for their share of the loss.

How well did you know this?
1
Not at all
2
3
4
5
Perfectly
67
Q

What are the consequences of a share buyback for the shareholders?

A

A buyback will result in the reduction of profits available for declaring dividends or of capital available for creditors in the event that a company cannot pay its debts.

If the company is wound up there is less money available for the shareholders once the creditors have been paid.

How well did you know this?
1
Not at all
2
3
4
5
Perfectly
68
Q

What is classed as a small company?

A

A small company is a company with a balance sheet of not more than £5.1 million, a turnover of not more than 0.2 million and no more than 50 employees in a particular financial year.

How well did you know this?
1
Not at all
2
3
4
5
Perfectly
69
Q

What happens if an unauthorised loan is made to a director?

A

If the company makes a payment in breach of the requirement to obtain ordinary resolution, the money is held by the recipient on trust for the company.

Any director who authorises the payment is jointly and severally liable to indemnify the company that made the payment for any loss resulting from it.

How well did you know this?
1
Not at all
2
3
4
5
Perfectly
70
Q

What does MA 26 do?

A

This gives the board discretion to refuse to register the transfer of shares. So if a company has MA 26 every transfer must be approved by the board before the transferee can be entered on the register of members and therefore become a shareholder.

How well did you know this?
1
Not at all
2
3
4
5
Perfectly
71
Q

What resolution is required to change the articles of association?

A

Special resolution of the shareholders

How well did you know this?
1
Not at all
2
3
4
5
Perfectly
72
Q

Who can request for a company to circulate a written resolution?

A

A shareholder/shareholders who have 5% or more voting rights in the company can require the company to circulate a written resolution.

The company’s articles can reduce this to below than 5% but cannot increase it to more than 5%.

How well did you know this?
1
Not at all
2
3
4
5
Perfectly
73
Q

What is share capital?

A

This is the money provided by shareholders in consideration for shares.

It is a long-standing principle of company law that the share capital cannot be reduced because it is the fund that the creditors look to when they need to pay debts owed to them.

Paid up share capital cannot be reduced and the shareholders liability with regard to any capital that they have not paid on their shares cannot be reduced.

How well did you know this?
1
Not at all
2
3
4
5
Perfectly
74
Q

What must the certificate of incorporation state?

A
  • The name and registered number of the company.
  • The date of its incorporation.
  • Whether it is limited or unlimited and if it is limited whether it is limited by shares or by guarantee.
  • Whether it is a private or a public company.
  • Whether the company’s registered office is situated in England and Wales, Scotland or NI.
How well did you know this?
1
Not at all
2
3
4
5
Perfectly
75
Q

Why are there particular provisions for close companies and groups of companies?

A

Provisions exist within tax legislation to make sure that there is no significant advantage or disadvantage, from a tax point of view, to trading as a company rather than as a sole trader or partnership - some of those provisions relate to close companies and to groups of companies.

How well did you know this?
1
Not at all
2
3
4
5
Perfectly
76
Q

What are the requirements of a moratorium?

A

The company must be unable to pay its debts or be likely to become unable to pay its debts.

A moratorium is not possible if the company has already entered into a moratorium during the previous 12 months.

How well did you know this?
1
Not at all
2
3
4
5
Perfectly
77
Q

How long does a moratorium last?

A

20 business days beginning with the business day after the moratorium comes into force i.e. date of filing the documents at court or the court order.

It can be extended for a further 20 business days by filing certain documents at court.

The moratorium can be extended by the directors for a period of up to one year if the creditors who are not going to get paid as a result of the moratorium consent to this.

How well did you know this?
1
Not at all
2
3
4
5
Perfectly
78
Q

What is a chargeable person?

A

The following are chargeable persons:
* individuals (whether in a personal capacity or as a sole trader);
* personal representatives (‘PRs’), when they dispose of the assets of the deceased person;
* partners, when the partners dispose of a chargeable asset. Each partner is charged separately for their proportion of the gain; and
* trustees, on the disposal of a chargeable asset from a trust fund.

How well did you know this?
1
Not at all
2
3
4
5
Perfectly
79
Q

What is the tax liability of a general partnership?

A

a) VAT
b) National Insurance
c) Income or corporation depending on whether the partnership is an individual or a company

How well did you know this?
1
Not at all
2
3
4
5
Perfectly
80
Q

Are private companies required to have a company secretary?

A

Not required to have a company secretary.

How well did you know this?
1
Not at all
2
3
4
5
Perfectly
81
Q

What are the requirements for a limited partnership?

A

Must be at least one general partner who has unlimited liability for the partnership debts.

Permitted to have a limited partner whose liability to the amount they initially invested in the business.

Must be registered with the Registrar of Companies, who also act as the Registrar of LPs before they start trading.

How well did you know this?
1
Not at all
2
3
4
5
Perfectly
82
Q

What is a standard breathing space?

A

This is a type of debt respite scheme.

A standard breathing space is available to any client with problem debt, and it gives them legal protection from creditor action for up to 60 days (a moratorium).

The protection includes pausing most enforcement action and contact from creditors and freezing most interest and charges on their debts.

How well did you know this?
1
Not at all
2
3
4
5
Perfectly
83
Q

How are company secretaries removed?

A

They can resign or be removed by board resolution.

In some cases there may be a written contract between the company and the company secretary which sets out the consequences of removal from office and include compensation for breach of contract.

How well did you know this?
1
Not at all
2
3
4
5
Perfectly
84
Q

What administration requirements are there following the allotment of shares?

A
  • The company needs to prepare minutes of every board meeting and every general meeting.
  • Copies of resolutions to be sent to CH within 15 days
  • Company forms to be sent to CH
  • Entries in company’s own register
  • Preparation and allocation of share certificates
How well did you know this?
1
Not at all
2
3
4
5
Perfectly
85
Q

In personal insolvency, what are the alternatives to bankruptcy?

A

a) Individual voluntary agreement
b) Negotiation with creditors
c) Debt relief orders (DRO)
d) Debt respite schemes

How well did you know this?
1
Not at all
2
3
4
5
Perfectly
86
Q

What is start-up relief/early trade losses relief?

A

Available when a startup suffers a loss in any of the first four tax years of the new business.

The loss can be carried back and set against the taxpayer’s total income in the three tax years immediately prior to the tax year of the loss.

This relief is useful for anyone who starts a new business but before that had an income from a previous business or employment.

It enables the taxpayer to claim back from HMRC some of the income tax they paid in their previous business or employment in the three tax years prior to the tax year of the loss.

The loss must be set against earlier years than later years.

The claim for the relief must be made on or before the first anniversary of 31 January following the end of the tax year in which the loss is assessed.

Subject to a cap of £50,000 or 25% of the taxpayer’s income in the tax year in which the relief is claimed. But only applies to income from sources other than the trade which produced the loss.

How well did you know this?
1
Not at all
2
3
4
5
Perfectly
87
Q

Which factors count in the director’s favour when they may be disqualified from office?

A

a) Employing qualified financial staff
b) Taking professional advice
c) A personal financial investment in the company

How well did you know this?
1
Not at all
2
3
4
5
Perfectly
88
Q

What happens if a partner of a general partnership cannot pay back their debts to a third party?

A

The third party can enforce the debt in the usual way by e.g. obtaining a charge over their property and applying for an order of sale over those properties in order to satisfy the outstanding debt.

The third party may seize assets belonging to the partner.

How well did you know this?
1
Not at all
2
3
4
5
Perfectly
89
Q

What must notice of a GM set out?

A
  • The time, date and place of the meeting
  • The general nature of the business to be dealt with at the meeting
  • If a special resolution is proposed, the exact wording of the special resolution
  • Each shareholders right to appoint a proxy to attend on their behalf
How well did you know this?
1
Not at all
2
3
4
5
Perfectly
90
Q

What is actual authority?

A

Arises where the director has consent from the other directors to act in a certain way e.g. to spend money.

How well did you know this?
1
Not at all
2
3
4
5
Perfectly
91
Q

How is VAT tax paid to HMRC?

A
  • VAT is paid to HMRC quarterly. The tax return needs to be submitted within one month of the end of the quarter.
  • Only the difference of input vs output is paid to HMRC.
  • If input tax exceeds output tax the person will receive a rebate.
  • HMRC may allow or require a taxpayer to make monthly returns in certain circumstances.
How well did you know this?
1
Not at all
2
3
4
5
Perfectly
92
Q

What rebuttable presumption is made in relation to a transaction at an undervalue (personal insolvency)?

Transaction with a close connection

A

If the transaction was with an associate aka close relative or business associate that was more than two years prior to the bankruptcy petition, then there is a rebuttable presumption that the bankrupt was insolvent.

How well did you know this?
1
Not at all
2
3
4
5
Perfectly
93
Q

What forms need to be sent to CH with regard to appointment of a director?

A

Form AP01 (individual director) and AP02 (corporate directors) used to notify CH of the appointment of a director and must be filed within 14 days of the appointment.

How well did you know this?
1
Not at all
2
3
4
5
Perfectly
94
Q

What number of directors must a private company have?

A

All companies must have one director.

How well did you know this?
1
Not at all
2
3
4
5
Perfectly
95
Q

What are the corporation tax rates?

A

The applicable corporation tax rate will depend on the company’s taxable profits:

  1. Profits up to £50,000

Subject to the standard small profits rate of 19%.

  1. Profits of more than £250,000

Subject to the main rate of 25% (on all of their taxable profits)

  1. Taxable profits above £50,000 but not exceeding £250,000

Subject to the marginal rate, meaning that the corporation tax rate is tapered so that companies in this bracket are charged at an overall corporation tax rate of between 19% and 25%.

How well did you know this?
1
Not at all
2
3
4
5
Perfectly
96
Q

Shareholders pay tax on dividends but there are other possible income tax implications.

What other possible income tax implications are there?

A
  1. Loan to a participator in a close company

When a ‘close company’ makes a loan to a shareholder, there may be income tax consequences for the shareholder if the close company writes off the loan.

  1. Share buyback

When a shareholder sells their shares back to the company in which they are held, their profit will be the difference between the sale price and the issue price of the shares. This will probably be charged to income tax in the same way as a dividend. However, sometimes the shareholder will pay CGT instead on their profit.

  1. Income tax relief for shareholders

Tax legislation includes two income tax reliefs for shareholders.

The first is relevant when an individual borrows money to purchase ordinary shares in a close company that carries on a trade, or to lend money to a close company that carries on a trade.

The second is income tax relief under the Enterprise Investment Scheme (EIS). It allows the individual to deduct from their income tax liability for the year a sum equal to 30% of the amount they have invested in the ordinary shares of qualifying unquoted companies.

The individual can subscribe up to £2 million per tax year in the ordinary shares of qualifying unquoted companies.

During the two years before and three years after the share purchase, the taxpayer must not be ‘connected with’ the company, meaning that the combined shareholdings of the taxpayer and their associates (including spouse and close family) must not exceed 30%.

How well did you know this?
1
Not at all
2
3
4
5
Perfectly
97
Q

Who approves long term service contracts?

A

This must be approved by the shareholders by ordinary resolution.

How well did you know this?
1
Not at all
2
3
4
5
Perfectly
98
Q

Which companies are exempt from statutory audit requirements?

A

Small companies.

Dormant companies (companies that do not trade).

How well did you know this?
1
Not at all
2
3
4
5
Perfectly
99
Q

How can business owners use a profit and loss account?

A

Business owners can use the profit and loss account to assess how profitable the business is.

There are two ways of increasing profits: reducing expenses or increasing income.

Businesses can increase income by selling more, perhaps because they are creating different or better products or marketing them better, or by increasing their prices.

Businesses can decrease expenses by, for example, buying cheaper stock or raw materials, reducing the number of employees, or renting cheaper premises.

How well did you know this?
1
Not at all
2
3
4
5
Perfectly
100
Q

How is a director appointed post-incorporation?

A

They will be appointed in accordance with company’s articles.

Under MA, the directors can be appointed either by the board or by ordinary resolution of the shareholders.

Board resolution is fastest but if the board is already circulating a written resolution or calling a GM on other matters it may make sense to also allow this.

How well did you know this?
1
Not at all
2
3
4
5
Perfectly
101
Q

What are the consequences for failing to file a confirmation statement?

A

It is a criminal offence to fail to file a confirmation statement or to file it late.

How well did you know this?
1
Not at all
2
3
4
5
Perfectly
102
Q

What is a long term service contract?

A

If the company has the power to terminate the contract within the two years it is not a guaranteed term of two years. It is the guaranteed term that requires the authorisation not require authorisation.

I.e. ten year service contract with a notice period of one year does not need authorisation by ordinary resolution.
I.e. a three-year fixed-term service contract with no provision for early termination does need authorisation by OR.

How well did you know this?
1
Not at all
2
3
4
5
Perfectly
103
Q

Who can bring a claim for fraudulent trading?

A

A liquidator or administrator.

How well did you know this?
1
Not at all
2
3
4
5
Perfectly
104
Q

What should be included in a PA regarding drawings and salaries?

A

The partnership agreement should set out how much each partner is allowed to drawn down in any given period, usually a month.

In the absence of such an agreement, they are entitled to share equally in income profits.

In some partnerships, the partners also receive a salary to reflect the work they do for the business whilst other partners may not receive a salary but share in any surplus profit.

How well did you know this?
1
Not at all
2
3
4
5
Perfectly
105
Q

When is insolvency presumed for a transaction at an undervalue?

A

Where the transaction was with a person connected with the company but this can be rebutted.

How well did you know this?
1
Not at all
2
3
4
5
Perfectly
106
Q

What is an auditor’s report?

A

The report must state whether in the auditor’s option the accounts have been prepared property and give a true and fair view of the company.

They must ensure that the shareholders whose money is invested are not defrauded or misled by the directors.

If the auditors report is qualified in any way, this is a warning to the shareholders that there may have been some unethical practices.

How well did you know this?
1
Not at all
2
3
4
5
Perfectly
107
Q

What are the exceptions to requirement for ordinary resolution for a loan to directors?

A

Expenditure on company business up to £50,000 in total

Expenditure on defending civil or criminal proceedings in relation to the company

Expenditure defending himself or herself in an investigation by a regulatory authority

Minor and business transactions as long as the transactions and any other relevant
transaction or arrangement does not exceed £10,000

How well did you know this?
1
Not at all
2
3
4
5
Perfectly
108
Q

What are the disadvantages of a credit facility?

A

The time and expense in negotiating and agreeing all the legal documentation for the loan and the high fees that are charged.

How well did you know this?
1
Not at all
2
3
4
5
Perfectly
109
Q

What reliefs should be considered in Step 3 of a CGT calculation?

A

Rollover relief

Rollover relief on incorporation of a business

Holdover relief on gifts

Business asset disposal relief

Private residence relief

How well did you know this?
1
Not at all
2
3
4
5
Perfectly
110
Q

When can members’ voluntary liquidation be used?

A

If the company is solvent.

How well did you know this?
1
Not at all
2
3
4
5
Perfectly
111
Q

What is the director’s duty to avoid conflicts of interest?

A

Directors have a duty to avoid a situations in which they have or can have a direct of indirect interest that conflicts or may possibly conflict with the interests of the company.

The duty to avoid a conflict applies in particular to the exploitation of any property, information or opportunity.

It does not matter whether the company could take advantage of the property, information or opportunity.

It must relate to a contract in which the company is not involved.

How well did you know this?
1
Not at all
2
3
4
5
Perfectly
112
Q

How long is an individual bankrupt for?

A

One year or possibly longer after which they are discharged.

This means that the bankrupt will be free of almost all their debt, even though the bankrupt has not paid all of their debts in full.

How well did you know this?
1
Not at all
2
3
4
5
Perfectly
113
Q

When does the declaration of interest in existing transaction or arrangement - s 182
this section not apply?

A

It does not apply if the director has already declared the interest under s 177 (duty to declare interest in a proposed transaction or arrangement).

How well did you know this?
1
Not at all
2
3
4
5
Perfectly
114
Q

What can be added into a company’s articles to protect a director from removal from office?

A

This gives someone who is both a shareholder and director greater voting rights as a shareholder if the resolution in question is a resolution to remove that person as a director.

How well did you know this?
1
Not at all
2
3
4
5
Perfectly
115
Q

When does the director’s duty to avoid conflicts of interest not apply?

A

It does not apply to a conflict of interest arising in relation to a transaction or arrangement with the company.

How well did you know this?
1
Not at all
2
3
4
5
Perfectly
116
Q

When is a preference presumed?

A

If a preference is given to a person who is connected to the company, the desire to prefer is presumed but this can be rebutted.

How well did you know this?
1
Not at all
2
3
4
5
Perfectly
117
Q

What is the deadline for filing a confirmation statement?

A

14 days from the company’s confirmation date which is the anniversary of its incorporation.

How well did you know this?
1
Not at all
2
3
4
5
Perfectly
118
Q

Who pays income tax?

A

Individuals, partners, personal representatives and trustees may have to pay income tax. Charities are generally exempt, and companies instead pay corporation tax.

How well did you know this?
1
Not at all
2
3
4
5
Perfectly
119
Q

What set-off against capital gains relief?

A

Allows the tax payer to set their trading losses against chargeable gains in the same tax year and applies when a tax payer has claimed carry across relief but not of the loss has been absorbed.

The claim for the relief must be made on or before the first anniversary of 31 January following the end of the tax year in which the loss is assessed.

How well did you know this?
1
Not at all
2
3
4
5
Perfectly
120
Q

How is trading profit calculated?

A

Chargeable receipts LESS deductible expenditure LESS capital allowance = trading profit/loss

How well did you know this?
1
Not at all
2
3
4
5
Perfectly
121
Q

Which creditors are bound by then IVA?

A

Every ordinary, unsecured creditor who is entitled to attend and vote is bound by the decision, even if they did not actually attend the meeting. The IVA is not binding on preferential or secured creditors unless they specifically agree to it.

How well did you know this?
1
Not at all
2
3
4
5
Perfectly
122
Q

What test will the court apply for unfair prejudice claim?

A

Court will apply an objective test. Whether the hypothetical bystander would believe the act or omission to be unfair.

It is more difficult to succeed in this claim if the conduct about which the claimant is complaining is in accordance with the company’s articles of association.

How well did you know this?
1
Not at all
2
3
4
5
Perfectly
123
Q

How is an LLP incorporated?

A

Formed by filing LL IN01 at CH along with applicable fee.

Similar form to IN01 from for filing a company.

There is no requirement for an LLP to file an LLP agreement.

Companies house will then issue a certificate of registration.

How well did you know this?
1
Not at all
2
3
4
5
Perfectly
124
Q

What is a moratorium?

A

It protects the company from actions by creditors relating to pre-moratorium debts but it must pay debts incurred during the moratorium in full.

During the moratorium the directors remain in control of the company but a qualified insolvency practitioner oversees the moratorium and can terminate it in certain circumstances.

How well did you know this?
1
Not at all
2
3
4
5
Perfectly
125
Q

What is the test for the every step defence for wrongful trading?

A

The test examines the facts that a director ought to have known or ascertained, the conclusions that they ought to have reached an the steps which they ought to have taken.

The standard expected of a director is that of a reasonably diligent person having both:

a) The general knowledge, skill and experience that may reasonably be expected of a person carrying out the same functions as are carried out by that director in relation to the company (OBJECTIVE)

b) The general knowledge, skill and experience that that director has (SUBJECTIVE).

How well did you know this?
1
Not at all
2
3
4
5
Perfectly
126
Q

What is the liability of members of an LLP?

A

Members may be liable for misfeasance, fraudulent and wrongful trading and may have to contribute to the assets of the insolvent LLP.

A member of an LLP could also be disqualified from being a director or a member of a LLP.

How well did you know this?
1
Not at all
2
3
4
5
Perfectly
127
Q

What are shareholders agreements?

A

This binds all shareholders who are party to the agreement and provides a remedy if one of its terms are breached.

How well did you know this?
1
Not at all
2
3
4
5
Perfectly
128
Q

What is the effect of liquidation?

A

The directors lose their powers (and in a compulsory liquidation their appointments are terminated). The liquidator takes over the running of the company.

How well did you know this?
1
Not at all
2
3
4
5
Perfectly
129
Q

What is classed as a micro-entity?

A

A micro-entity is a company with a balance sheet total of not more than £632,000 and no more than 10 employees in a particular financial year.

How well did you know this?
1
Not at all
2
3
4
5
Perfectly
130
Q

What must be included in the written resolution?

A

How to signify the agreement

The deadline for returning the written resolution I.e. the lapse date

How well did you know this?
1
Not at all
2
3
4
5
Perfectly
131
Q

Following a s 303 request, when should the directors call the general meeting?

A

Directors must call it within 21 days of request.

How well did you know this?
1
Not at all
2
3
4
5
Perfectly
132
Q

What needs to be filed at Companies House in order to re-register a private company as a public company?

A

Special resolution

Application for re-registration on Form RR01 including a statement of compliance

Fee for re-registration

Revised articles

A balance sheet and a written statement from company’s auditors and a valuation report on any shares that have been allotted for non-cash consideration between the date of then balance sheet and the passing of the special resolution

How well did you know this?
1
Not at all
2
3
4
5
Perfectly
133
Q

What is a confirmation statement?

A

Every company must file a confirmation statement on form CS01. The purpose of the confirmation statement is to make sure that the information held at CH is correct and up to date.

How well did you know this?
1
Not at all
2
3
4
5
Perfectly
134
Q

What can secured creditors do if they are not getting paid back their debt?

A

Secured creditors may be able to appoint a receiver to take possession of the property which is the subject of the charge and deal with it for the benefit if the charge holder. This will usually involve selling the property.

Once the property is sold, the receiver usually has no interest in the property.

Normally the trigger for going into receivership is that the company has breached the charge holder’s loan agreement e.g. by defaulting on repayments.

The charging document itself will state when a receiver can be appointed and the company does not need to be insolvent.

How well did you know this?
1
Not at all
2
3
4
5
Perfectly
135
Q

What are the initial considerations for the lender when a company borrows money?

A

The lender needs to make sure:
a) There are no restrictions on the company granting security and
b) The directors have authority to act,
c) The people it’s dealing with have been properly appointed as directors of the company.

How well did you know this?
1
Not at all
2
3
4
5
Perfectly
136
Q

Can a breach of requirement to authorise a loan to directors be affirmed?

A

The transaction can be affirmed within a reasonable time by the company or its holding company if it is the holding company that has failed to obtain the ordinary resolution.

To do so, the shareholders have to pass an ordinary resolution. This means that the transaction would no longer be voidable and the directors are no longer liable in relation to the initial failure to obtain ordinary resolution.

How well did you know this?
1
Not at all
2
3
4
5
Perfectly
137
Q

What happens if the DRO is accepted?

A

During the time that the debtor is subject to the DRO, the same restrictions apply to them as to a bankrupt.

As with bankruptcy, these restrictions may last up to 15 years if the debtor is dishonest or culpable, and this extra period is obtained by the Official Receiver applying for a Debt Relief Restrictions Order against the debtor.

How well did you know this?
1
Not at all
2
3
4
5
Perfectly
138
Q

What are the exceptions for the requirement for an ordinary resolution to authorise a substantial property transaction?

A

If the SPT is between the a wholly owned subsidiary and its holding company

A transaction between a company and a person in his character as a member of a company (as opposed to acting in their personal capacity)

A transaction between a wholly owned subsidiary of a company

A transaction between two wholly owned subsidiaries of the same holding company

How well did you know this?
1
Not at all
2
3
4
5
Perfectly
139
Q

What is the personal allowance for 2024/2025?

A

ersonal allowance of £12,570

Everybody is allowed to earn a certain amount of income each year before they start paying income tax. This sum is known as the personal allowance, and it is deducted from net income to obtain the taxpayer’s taxable income.

The personal allowance for 2024/25 is £12,570. It can be set against income of any kind but is applied in a certain order:
1. firstly, against NSNDI
2. if there is surplus, against savings income
3. any remaining surplus is applied against dividend income

This is relevant for step 3 of the income tax calculation.

How well did you know this?
1
Not at all
2
3
4
5
Perfectly
140
Q

What are consolidated accounts?

A

Parent companies must normally produce a consolidated profit and loss account and balance sheet showing the group’s profits or losses, assets and liabilities.

This requirement reflects the commercial reality that the group is a single unit and it enables shareholders to assess the performance of their company within the context of the whole group.

Each individual company in the group still has to prepare its own final accounts in addition to the parent company preparing the consolidated accounts.

How well did you know this?
1
Not at all
2
3
4
5
Perfectly
141
Q

Why should a Partnership Agreement set out clauses that are in the PA 1890?

A

There are some clauses that should be included in a PA. Sometimes a PA will just repeat what the PA 1890 says but this should be included anyway because the it is useful to have partners have a complete document setting out all their rights and obligations rather than having to look at both.

How well did you know this?
1
Not at all
2
3
4
5
Perfectly
142
Q

What is the procedure for a breach under GAAR?

A
  1. HMRC notice

If HMRC finds that a taxpayer is in breach of the GAAR, it will notify the taxpayer of why it considers that a tax advantage has arisen to the taxpayer from tax arrangements that are abusive, and set out the tax adjustments (or ‘counteraction’) that the officer considers ought to be taken.

The adjustments must be ‘just and reasonable’, and can be made either by the taxpayer or HMRC.

  1. Taxpayer written representations

If counteraction is proposed by HMRC, the taxpayer is permitted to make written representations in their defence, and the matter will then be referred to the GAAR advisory panel (‘Panel’), who will issue their opinion by way of a notice to the taxpayer and HMRC.

HMRC will then, provided that they still agree with the Panel’s opinion, give the taxpayer a written notice setting out whether the tax advantage arising from the arrangements is to be counteracted under the GAAR, the adjustments required and the steps that the taxpayer must make to give effect to the adjustment.

  1. Consequences

Anyone who enables an abusive tax arrangement may be required to pay a penalty. An enabler is any person who, in the course of their business, enables the abusive tax arrangements that are defeated.

This can include individuals, companies or partnerships. Individual employees cannot be enablers because they are not acting in the course of their business. Instead, the firm or company employing them will possibly be liable to a penalty. There is a right of appeal to a tax tribunal. The penalty is equal to the value of the financial or other benefit received by the enabler in return for what the enabler did to facilitate the arrangements.

How well did you know this?
1
Not at all
2
3
4
5
Perfectly
143
Q

How can a director protect themselves from being liable under the director’s duty to avoid conflicts of interest?

A

A board resolution authorising the breach or potential breach is enough to protect the director in question from a claim for breach of duty under this section.

How well did you know this?
1
Not at all
2
3
4
5
Perfectly
144
Q

When analysing business accounts, what generally needs to be taken into consideration?

A
  1. Profitability

The profit and loss account shows whether the business has made a profit.

This does not mean, however, that the business has cash in the bank or can pay its debts. To understand this, consider how the profit and loss account is compiled. It will show how much the business has sold or how much it has charged for services, and this will be part of the calculation of profit. But the business may not have received payment. Looking at the balance sheet may reveal a high figure for debtors and no cash in the bank. Alternatively, perhaps the business has made a large profit but used the cash received to buy expensive equipment, so there is no cash in the bank.

Profitability, then, does not mean that the business can pay its debts (although, of course, a profitable business is still more likely to be able to pay its debts than an unprofitable one).

  1. Can a business pay its debts?

The balance sheet shows whether a business can pay its debts.

Liquidity is the availability of liquid assets to a business.

A liquid asset is an asset that can easily be converted into cash in a short amount of time. In the context of a balance sheet, this means current assets.

An extremely valuable factory and office premises are of no use to a business if it needs cash within the next fortnight: it cannot sell a factory and office premises in two weeks, so owning these valuable assets will not help it to pay its debts as they fall due.

Businesses can only feasibly use current assets to meet their liabilities. Therefore, it is important that the business does not run short of current assets. If a company is unable to pay its debts as they fall due, or if its liabilities exceed its assets, a company will be deemed to be insolvent.

How well did you know this?
1
Not at all
2
3
4
5
Perfectly
145
Q

What needs to be included in VAT tax invoices?

A

A person making a taxable supply to a taxable person must provide a tax invoice, an invoice showing information such as the VAT number, the value of supply and the rate of tax charged.

The person charging VAT must have tax invoices for all of the input tax they are reclaiming.

How well did you know this?
1
Not at all
2
3
4
5
Perfectly
146
Q

When analysing business accounts, what needs to be considered in terms of the wider context?

A

Some steps you can take when analysing accounts to help you to understand the wider context, including:

  1. Checking the date of the balance sheet. Is it up to date, and therefore showing an up- to- date financial position? Has it been distorted by a recent seasonal boost in sales, resulting, for example, in a high cash balance and lots of debtors?
  2. Look at the business’s accounts from previous years. This will help to show whether the business is growing and becoming more profitable, or in decline.
  3. Check how valuations have been carried out, and how recently. Freehold premises almost always increase in value over time, and if they have not been valued recently, the business may well be worth more than it appears. Fixed assets, such as machinery, can be valued in different ways, so you need to check how this was done and think about whether this is an accurate reflection of value. Finally, it is really important, in a trading business, to check how closing stock is valued. Closing stock is the stock remaining at the end of the financial year. Stock may appear on the balance sheet at cost price, but on closer investigation the stock is obsolete or damaged, so will never realise this amount if it were sold.
  4. Analyse the debtors figure. Debtors are classed as assets, because when they pay the outstanding debt to the business, the business will have more cash. However, sometimes debtors do not pay.
  5. Is there a bank overdraft? This is classed as a current liability, because it is repayable on demand. It will reduce the current assets figure and make it look as if the business may have difficulty paying its debts in the short- term. However, in reality banks will not demand repayment of an overdraft unless the business is in financial difficulties, so a large overdraft may not be a problem.
  6. Look for exceptional items, which may distort the accounts for that year. For example, perhaps the business has made a major investment in machinery, which will increase profits in the long run, but which has not happened yet.
How well did you know this?
1
Not at all
2
3
4
5
Perfectly
147
Q

What is a debt respite scheme?

A

This can give someone in problem debt the right to legal protection from creditor action.

There are two types of breathing space: a standard breathing space and a mental health crisis breathing space.

How well did you know this?
1
Not at all
2
3
4
5
Perfectly
148
Q

What are the duties of the administrator?

A

They must perform their duties in the interests of the company’s creditors as a whole.

Their primary objective must be to rescue the company as a going concern.

If this is not practicable, they must try to achieve a better result for the company’s creditors as a whole than would be likely if the company were wound up.

If this is not practicable, they must realise the property to pay one or more secured or preferential creditors.

How well did you know this?
1
Not at all
2
3
4
5
Perfectly
149
Q

Which reliefs cannot be used in conjunction with rollover relief on the replacement of qualifying assets?

A

The annual exemption cannot be first applied to any gains which are to be rolled over or held over under these reliefs.

Rollover relief on the replacement of qualifying assets cannot generally be used in conjunction with hold- over relief (because there is usually no gift) or rollover relief on incorporation (because shares are not qualifying business assets).

How well did you know this?
1
Not at all
2
3
4
5
Perfectly
150
Q

In terms of post-decision requirements, what must be filed at companies house?

A

a) Register of members updated
b) Register of directors updated
c) Board minutes
d) General meeting minutes

How well did you know this?
1
Not at all
2
3
4
5
Perfectly
151
Q

Where must the buyback of shares by share capital be shown in order to alert potential creditors who want to oppose the buyback?

A

Within seven days of the SR being passed, the company must put a notice in the London Gazette stating that the shareholders have approved payment out of capital in order that the company can buy back its own shares.

It must specify the amount of capital used, the date of the SR and where the director’s statement and auditor’s report are available for inspection.

It must also state that any creditor of the company can apply within five weeks from the buyback being passed to overturn the decision.

All of these above also needs to be posted in a national newspaper.

How well did you know this?
1
Not at all
2
3
4
5
Perfectly
152
Q

In bankruptcy proceedings, what happens to the debtor’s home if they own it?

A

If the bankrupt is a homeowner, their interest in the home passes to the trustee.

How well did you know this?
1
Not at all
2
3
4
5
Perfectly
153
Q

Is there a time limit for bringing a claim for transaction defrauding creditors (personal insolvency)?

A

No

How well did you know this?
1
Not at all
2
3
4
5
Perfectly
154
Q

Under the PA 1890, when will the partnership dissolve automatically?

A

a) When a partner retires
b) On expiry of a fixed term
c) By death or bankruptcy by any of the partners
d) If the partners give notice of dissolution to a partner who has by order of court granted a charge over their share of the partnership property for debt owed by them alone and not the partnership as a whole
e) Unlawful acts that make it unlawful for business of firm to be carried on

Dissolution of partnership on expiry of fixed term, death/bankruptcy of partners and notice + charge of share can be disapplied in the PA.

Dissolution for unlawful acts cannot be disapplied.

How well did you know this?
1
Not at all
2
3
4
5
Perfectly
155
Q

What are the general rules for taxation of partnerships?

A

Unlike companies, partnerships are not separate legal entities. This means that any tax liability arising from the partnership business is not payable by the partnership but rather by the individual partners, who will pay income tax and/ or capital gains tax.

Obviously, not all of the income or capital gains made by the business are attributable to one partner; the partners will share capital and profits between them.

It is necessary to apportion the profits between the partners. The rules for doing so will be different depending on whether the partnership consists of individuals only or individuals and one or more corporate partners, because companies pay corporation tax rather than income and capital gains tax.

Partners who are individuals may have to pay income tax on trading profits and other income, capital gains tax on capital profits, and inheritance tax.

How well did you know this?
1
Not at all
2
3
4
5
Perfectly
156
Q

What impact does share buyback have on CGT?

A

Usually, when a shareholder sells their shares back to a company, there will be a charge to income tax for the shareholder rather than CGT. The profit represented by the difference between the consideration the shareholder receives and the issue price of the shares will be taxed as a dividend. However, the shareholder’s profit sometimes attracts CGT instead.

  1. Conditions for attracting CGT instead if income tax

Capital gains tax will be relevant when certain conditions are satisfied:

a. the buyer must be a trading company and its shares must not be listed on a recognised stock exchange (AIM is not a recognised stock exchange for the purposes of this test); and
b. the purpose of the buyback must either be to raise cash to pay inheritance tax or be for the benefit of the company’s trade (perhaps where there is a rift between shareholders and the company will function more effectively if one of the shareholders sells their shares back to the company); and
c. the seller must have owned the shares they are selling back to the company for at least five years; and
d. the seller must either be selling all of their shares or substantially reducing their percentage shareholding (by at least 25%) to a maximum of 30% of the issued share capital of the company.

If it is difficult to decide whether these conditions are met, the taxpayer can apply to HMRC for advance clearance of their proposed tax treatment of the buyback.

  1. Tax benefits

Whether paying income tax or CGT would result in a lower tax liability will depend on the taxpayer’s circumstances and how much income they earn. If the taxpayer pays CGT, the availability of reliefs such as business asset disposal relief may significantly reduce their liability to CGT.

If the taxpayer is a basic rate income taxpayer, then they are likely to benefit from the ordinary income tax rate for dividends (8.75%) and the dividend allowance.

It is sometime possible to structure the buyback so that the taxpayer obtains the most favourable tax treatment.

How well did you know this?
1
Not at all
2
3
4
5
Perfectly
157
Q

What is a key limitation for a general partnership?

A

Partners do not enjoy limited liability for debts

How well did you know this?
1
Not at all
2
3
4
5
Perfectly
158
Q

How is the taxable income divided into the separate forms of income?

A

To find out how much taxable income is comprised of NSNDI and how much of savings and
dividend income, the savings and dividend income are deducted from the taxable income
figure arrived at in Step 3:

Taxable income less savings and dividend income = taxable NSNDI

The tax payable on NSNDI can then be calculated.

The order of taxation is:
1. NSNDI
2. Savings
3. Dividend

This is step 4 of working out income tax.

How well did you know this?
1
Not at all
2
3
4
5
Perfectly
159
Q

When should the payment for buyback of shares be made out of the share capital?

A

The payment out of capital must be made no earlier than five weeks after the date of the special resolution to approve the buyback out of capital and no later than seven weeks after the date of the special resolution so the board has a two week window to enter into the contract.

How well did you know this?
1
Not at all
2
3
4
5
Perfectly
160
Q

What information needs to be filled out on an IN01 form?

A

Articles

Company name

Registered office

First directors

Director’s residential and service addresses

Company secretary

First shareholders

Statement of capital

People with significant control

How well did you know this?
1
Not at all
2
3
4
5
Perfectly
161
Q

What is the notice period for a GM?

A

14 clear days (day notice is received and day of the GM are not counted).

How well did you know this?
1
Not at all
2
3
4
5
Perfectly
162
Q

In relation to income tax, what if the taxpayer’s income exceeds £100,000?

A

Where the taxpayer’s income exceeds £100,000, the personal allowance is reduced by £1 for every £2 of income above the £100,000 limit. So a taxpayer earning a net income of £102,000 will benefit from a personal allowance of £11,570 – this is £1,000 less than those earning £100,000, because every £2,000 earned over £100,000 results in a reduction of £1,000 to the personal allowance.

Once a taxpayer’s income reaches £125,140, they will not have a personal allowance because their personal allowance will have been reduced to zero by this point.

How well did you know this?
1
Not at all
2
3
4
5
Perfectly
163
Q

What is a substantial property transaction?

A

A substantial property transaction is where a director, in their personal capacity or someone connected with a director buys from or sells to the company a non-cash asset of substantial value.

How well did you know this?
1
Not at all
2
3
4
5
Perfectly
164
Q

What is classed as substantial in the context of a substantial property transaction?

A

a) It will automatically be classed as substantial if its value is one £100,000.

b) It will also be classed as substantial if it is worth more than £5,000 AND more than 10% of the company’s net asset value.

The company’s net asset value can be ascertained by looking at the company’s balance sheet.

It just needs to pass one of the two tests.

How well did you know this?
1
Not at all
2
3
4
5
Perfectly
165
Q

How are company reserves printed on a company balance sheet?

A

The main feature of a company balance sheet which differs from an unincorporated business is in the ‘Capital Employed’ section, which is often called the ‘Capital and Reserves’ section in company accounts.

  1. Reserves

The reserves part of a balance sheet shows what the company ‘owes’ to its shareholders – the money shown would be paid to shareholders (after payment of any outstanding debts of the company) if the company were wound up.

There are different types of reserve, the main difference being between revenue reserves and capital reserves.

1.1. Revenue reserves

Most reserves are revenue reserves, which can in theory be distributed to the shareholders, for example through the payment of a dividend, even if the company decides not to do so. Capital reserves cannot be distributed to shareholders.

The profit and loss reserve is the most significant revenue reserve. It consists of the company’s profits, after tax and after payment of any dividends the company has decided to pay.

If the company makes a profit every year, and there is still some left after payment of tax and dividends, it will be added to the profit and loss reserve. This means that over time, the figure in the profit and loss reserve will increase unless it is spent.

It is common for the different reserves to be ‘labelled’ in the accounts, to show why the profits have been retained rather than paid to shareholders by way of dividend. An example of a label is the ‘debenture redemption reserve’, shown in the balance sheet extract above, which will indicate that some of the profits have been retained to repay a debenture in the future.

It is important to remember that this is just a label – the money is still retained profit, but the label tells anyone looking at the accounts how much the company is planning to use to repay the debenture.

1.2. Capital reserves

Capital reserves are not available for distribution to shareholders because they are part of the long- term capital of the company.

An example of a capital reserve is the share premium account. When shares are issued at a premium, the company must record the surplus over the nominal value of the shares on the share premium account. The surplus is part of the capital of the company.

How well did you know this?
1
Not at all
2
3
4
5
Perfectly
166
Q

Does the company need to keep a register of the director’s residential addresses?

A

This is for individual directors only.

It is not open for inspection.

Can be kept on the central register at CH instead.

How well did you know this?
1
Not at all
2
3
4
5
Perfectly
167
Q

Does an LLP need to have a registered office?

A

An LLP must have a registered office which is its address for service of official documents and the registered office address must appear on its stationary.

How well did you know this?
1
Not at all
2
3
4
5
Perfectly
168
Q

When do pre-emption rights not apply?

A

They do not apply in relation to the allotment of bonus shares, if the consideration for the allotment is wholly or partly non-cash or if the shares are to be held under, allotted or transferred in an employee share scheme.

How well did you know this?
1
Not at all
2
3
4
5
Perfectly
169
Q

Comparing debt and equity finance, what is the relative risk of investment?

A

Equity finance

From a buyers perspective, shares in a company can be riskier than lending to a company.

If the company is in financial difficulties, the company will not declare a dividend as it can only do this if it has distributable profits and even then the payment of dividends is usually discretionary.

The shareholder will also risk losing the capital value of the shares if the company goes insolvent.

Debt finance

As interest payments are a contractual liability of the company, the lender will be paid before the shareholder.

Furthermore, the loan may be secured over the company’s property and there may be personal guarantees from the directors.

How well did you know this?
1
Not at all
2
3
4
5
Perfectly
170
Q

How can shareholders remove a director?

A

The shareholders can remove a director by ordinary resolution passed at a GM.

Special notice is required before the GM to remove a director.

How well did you know this?
1
Not at all
2
3
4
5
Perfectly
171
Q

What kind of consent is needed to authorise a substantial property transaction?

A

If the director wants to enter into an SPT, the shareholders have to give consent by way of ordinary resolution.

If the person in question is also director of the company’s holding company or a person connected with such a director, the transaction or arrangement must also be approved by ordinary resolution of the shareholders of the parent company.

How well did you know this?
1
Not at all
2
3
4
5
Perfectly
172
Q

Which grounds for partnership dissolution canNOT be disapplied by the PA?

A

Dissolution for unlawful acts cannot be disapplied.

How well did you know this?
1
Not at all
2
3
4
5
Perfectly
173
Q

How are partnerships taxed (some partners are companies)?

A

They would pay corporation tax on their share of the profits.

How well did you know this?
1
Not at all
2
3
4
5
Perfectly
174
Q

What is a partners’ liability for partnership debts if the debt was incurred before leaving the partnership?

A

Each partner is jointly and severally liable with the other partners for debts incurred by the partnership when they were a partner. A claimant can sue any or all of the parters and collect the total damages awarded by any of and all of them, leaving the defendants to seek a contribution from any of the other partners.

Whilst this seems risky, in practice the partnership will likely have a professional indemnity insurance for any professional negligence claims. However this obviously only applies to professional occupations, a manufacturing company would be in severe difficulty.

How well did you know this?
1
Not at all
2
3
4
5
Perfectly
175
Q

Does a company have to be registered with HMRC for corporation tax?

A

Yes, the company must be registered with HMRC for corporation tax. If companies are registered online, they will be registered for corporation tax automatically.

If the application was made by post or through a formation agent or using third party software, the applicant has to make a separate application to HMRC within three months of starting to do business.

How well did you know this?
1
Not at all
2
3
4
5
Perfectly
176
Q

When can there not be considered to be a breach of director’s duty to avoid conflicts of interest?

A

There is no breach if the situation cannot reasonably be regarded as likely to give rise to a conflict of interest.

The duty has not been infringed if it has been authorised by the directors.

How well did you know this?
1
Not at all
2
3
4
5
Perfectly
177
Q

When does income tax have to be paid?

A

There are different tax returns for different types of income and they are issued soon after 5 April each year.

a. Online

Taxpayers are encouraged by HMRC to file an online tax return. The online tax return and any payment must be filed by 31 January following the tax year to which the return relates.

b. Paper

If the taxpayer wishes to file a paper return, the submission date is earlier: no later than 31 October.

The taxpayer must make two payments on account towards the income tax due for any tax year, and a final balancing payment to meet any tax still outstanding.

The payment dates are:
- first payment on account: by 31 January in the tax year in question;
- second payment on account: by 31 July after the end of the tax year; and
- any balancing payment (calculated once the tax year is over) is due on the next 31 January.

  1. How much are the payments on account?

The first and second payments should each be approximately on half of the taxpayer’s tax liability, based on the previous year’s accounts. However, the payments are reduced to give credit for any tax deducted at source.

The taxpayer does not have to make a payment on account if the amount remaining after giving credit for tax deducted at source is below a certain limit.

This is important, because it means that the following people are not required to make payments on account:
- most employees;
- pensioners
- others who receive most of their income after deduction at source; or
- those who have relatively small tax liabilities.

Taxpayers can claim a reduced payment on account or cancellation of the payment on account if they have grounds for believing that if they make payments on account based on the previous year’s accounts, this will result in an overpayment of tax in the current tax year.

  1. What happens if tax is not paid?

HMRC charges interest on any tax unpaid at the due date for payment. This applies to both payments on account and balancing payments.

There are also fixed penalties – fines – for late or non-payment.

2.1. Records

Taxpayers must maintain adequate records to support the information in their tax return, and there is a penalty for default.

HMRC has the power to carry out audits and make enquiries to check whether the tax return is accurate. Taxpayers can appeal against assessments to the First- Tier Tribunal (Tax).

How well did you know this?
1
Not at all
2
3
4
5
Perfectly
178
Q

What is a term loan?

A

A business borrows a fixed amount of money for a specific period at the end of which it must all be repaid.

They also pay interest at regular intervals.

The contract for a term loan may be called a loan agreement, a credit agreement or a facility agreement.

How well did you know this?
1
Not at all
2
3
4
5
Perfectly
179
Q

What is carry forward relief on incorporation of a business?

A

If a taxpayer incorporates their business by transferring it to a company wholly or mainly in return for shares, any trading losses which have not been relieved can be carried forward and set against any income they receive from the company such as their salary or dividends.

To be classed as wholly or mainly in return for shares, 80% or more of the consideration for the business transferred must be shares in the company.

The taxpayer can set the losses against more than one form of income until the loss is fully absorbed e.g. salary/dividends.

They can set the loss against an order of income type as they choose so its down to them to consider their own financial situation and choose the order that has be biggest tax advantage.

There is no cap on the amount that can be relieved under this provision.

Whilst the losses can be carried forward indefinitely, the tax payer must notify HMRC of its intention to claim the relief no more than four years after the end of the tax year in which the loss was incurred.

How well did you know this?
1
Not at all
2
3
4
5
Perfectly
180
Q

What is the income tax relief on borrowings?

A

If an individual borrows money to buy a share in a partnership or to lend money to a partnership, they can deduct the interest they pay on this borrowing from total income.

This loan is a ‘qualifying loan’. This encourages individuals to invest in businesses.

There is a cap on the amount of tax relief, of the greater of £50,000 or 25% of the taxpayer’s total income less allowable pension contributions in the tax year where the relief is claimed.

However, the cap only relates to income from sources other than the trade which produced the loss, so its effect is limited.

How well did you know this?
1
Not at all
2
3
4
5
Perfectly
181
Q

What is a chairperson’s additional purpose in a public company?

A

The chair of the board acts as a figurehead in dealings with shareholders and anyone outside of the company.

How well did you know this?
1
Not at all
2
3
4
5
Perfectly
182
Q

What happens if an eligible member signifies their agreement after the lapse date?

A

If an eligible member signifies their agreement after the lapse date, their agreement will not be counted.

How well did you know this?
1
Not at all
2
3
4
5
Perfectly
183
Q

What is a close company?

A

a) A close company is a company either:
controlled by five or fewer participators; or
b) controlled by participators (any number of them) who are directors or shadow directors.

  1. What is a participator?

A participator is a person who owns shares in the company, or has the right to acquire shares in the company.

To ascertain whether the participators ‘control’ the company, the test is whether the participators own more than half of the shares in the company or have more than half of the voting power in the company, or have the right to acquire more than half of the shares in the company.

When a close company loans money to a participator or their associate (broadly, ‘associate’ means a close relative or business partner), the company must pay to HMRC an amount of money equivalent to 33.75% of the loan. The payment is akin to a deposit: payment will be refunded to the company if and when the participator/ associate repays the loan or if the loan is written off.

  1. When is the tax not payable?

a) the loan is made in the ordinary course of a money- lending business, for example, a bank loan by a bank to a shareholder; or
b) if the loan (added to any other such loan made to the same person) is no more than £15,000 and the borrower works full- time for the company and owns no more than 5% of the company’s ordinary shares.

  1. When will the borrower have to pay tax on the debt?

The borrower does not have to pay any tax in relation to the loan, unless the company writes off the debt.

  1. What is the effect of this provision?

The reason for these provisions is to prevent tax avoidance. If these provisions did not exist, directors or shareholders could borrow money from the company, instead of receiving it as taxable income or dividend, and arrange for the company not to enforce the obligation to repay for many years, if at all.

The loan would then be almost akin to a salary, but tax- free. Shareholders of a close company who take out a loan to purchase shares in the company or to lend money to the close company may be able to claim income tax relief on the interest payable on the loan.

How well did you know this?
1
Not at all
2
3
4
5
Perfectly
184
Q

When must the buyer pay stamp duty on shares after a share transfer?

A

If the sale price of the shares is over £1000. This is recorded on the stock transfer form.

No stamp duty is payable when the shares are a gift.

How well did you know this?
1
Not at all
2
3
4
5
Perfectly
185
Q

What assets can be secured?

A

Most property.

a) Land (freehold/leasehold and fixtures/fittings)
b) Tangible property (machinery, computers and stock)
c) Intangible property (money in bank account, debts owed, shares owned, intellectual property rights)

How well did you know this?
1
Not at all
2
3
4
5
Perfectly
186
Q

What is the advantage of a CVA for creditors?

A

They are likely to be paid more in a CVA than they would if the company went into liquidation or administration

How well did you know this?
1
Not at all
2
3
4
5
Perfectly
187
Q

When assessing the second stage of a derivative claim, when must the court refuse permission to continue?

A

Court will not allow an individual who is not promoting the success of the company to continue the claim.

Where the cause of action arises from an act or omission that has not yet occurred but which has already been authorised by the company.

When the act or omission has already occurred and was authorised before it occurred or has been ratified by the company since it occurred.

How well did you know this?
1
Not at all
2
3
4
5
Perfectly
188
Q

What are the consequences for failing to obtain a shareholder resolution for a long term service contract?

A

The guaranteed term element of the service contract is void, although the rest of the contract would be enforceable.

The service contract would be capable of being terminated on reasonable notice.

How well did you know this?
1
Not at all
2
3
4
5
Perfectly
189
Q

What is the effect of shareholders ratifying a director’s breach of duty?

A

The director escapes liability to the company for breach of duty.

How well did you know this?
1
Not at all
2
3
4
5
Perfectly
190
Q

What is a claim in misfeasance?

A

This is breach of any fiduciary or other duty by the directors.

During the course of winding up, the directors may be ordered to contribute to the company’s assets by way of compensation in respect of the misfeasance.

They may also be ordered to repay, restore or account for any money or property or any part of it that has been misapplied in breach of duty.

How well did you know this?
1
Not at all
2
3
4
5
Perfectly
191
Q

How do CVAs compare to administration?

A

It is cheap, easy to undertake and is available to liquidators, administrators and the company itself.

How well did you know this?
1
Not at all
2
3
4
5
Perfectly
192
Q

Who approves the proposals of a CVA?

A

a) 75% or more in value of the company’s creditors and
b) 50% or more of non-connected creditors

How well did you know this?
1
Not at all
2
3
4
5
Perfectly
193
Q

What are the advantages of a security of a loan for a lender?

A

If the lender fails to repay the loan as agreed, then the lender can seize the secured assets, sell them and pay itself out of the proceeds of sale.

How well did you know this?
1
Not at all
2
3
4
5
Perfectly
194
Q

What are the consequences of not maintaining share capital?

A

Dividends cannot be paid out of capital, only out of distributable profits

The company must not generally purchase its own shares

How well did you know this?
1
Not at all
2
3
4
5
Perfectly
195
Q

What is the director’s duty to declare interest in a proposed transaction or arrangement?

A

There must be a transaction or agreement with the company for this duty to be breached.

A director must declare the nature and extent of any interest they are directly or indirectly interested in.

How well did you know this?
1
Not at all
2
3
4
5
Perfectly
196
Q

When is an IVA sought?

A

Before or during bankruptcy.

How well did you know this?
1
Not at all
2
3
4
5
Perfectly
197
Q

What is the CGT tax rate for trustees and PRs?

A

Gains made by trustees and PRs are all taxed at 20%, or, for residential property, 28%.

How well did you know this?
1
Not at all
2
3
4
5
Perfectly
198
Q

After the buyback of shares out of share capital has been authorised, what needs to be filed at the CH?

A

Filing copy of directors’ statement and auditors’ report at CH

This should be done before or at the same time as it places the notices in the London Gazette and the newspaper.

How well did you know this?
1
Not at all
2
3
4
5
Perfectly
199
Q

Which reliefs cannot be used in conjunction with holdover relief?

A

The annual exemption cannot be first applied to any gains which are to be held over under these reliefs.

Business asset disposal relief cannot apply to any gains which are to be held over on the gift of business assets.

How well did you know this?
1
Not at all
2
3
4
5
Perfectly
200
Q

When is CGT payable?

A

Generally it is payable on or before 31 January following the end of the tax year, or 30 days from the making of an assessment, if later.

However, a taxpayer is required to submit a provisional calculation of any gains made from the sale of a residential property and pay any tax due within 30 days following completion of the sale.

How well did you know this?
1
Not at all
2
3
4
5
Perfectly
201
Q

Which companies have to file annual accounts?

A

Every company, apart from small companies and micro-companies, have to file director’s reports and accounts.

How well did you know this?
1
Not at all
2
3
4
5
Perfectly
202
Q

What form needs to be sent to CH to register an individual on the PSC register for the first time?

A

Form PSC01

How well did you know this?
1
Not at all
2
3
4
5
Perfectly
203
Q

Who can be a company secretary of a private company?

A

They do not need to have any specific qualifications and the role they are expected to fulfil will vary from company to company.

Large companies may have a full-time company secretary who may run an administrative department.

In smaller companies the company secretary may also be a director and will be the person responsible for ensuring that the company keeps up to date with its filings at Companies House.

How well did you know this?
1
Not at all
2
3
4
5
Perfectly
204
Q

What are the key shareholder special resolutions?

A

To change the name of the company
To amend the company’s articles of association
To disapply pre- emption rights
To approve the re- registration of a private company as a public company
To approve a payment out of capital

How well did you know this?
1
Not at all
2
3
4
5
Perfectly
205
Q

Which individuals may be liable for breach of substantial property transaction?

A

Any director of the company (or of its holding company) with whom the company entered into the arrangement

Person connected to the director of the company/holding company that entered into the arrangement and the director themselves,

Any other director who authorised the arrangement/transaction

How well did you know this?
1
Not at all
2
3
4
5
Perfectly
206
Q

If the charge was granted without the company giving fresh consideration in exchange for granting security at the ‘relevant time’ before the onset of insolvency.

When is the ‘onset of insolvency’?

A

Liquidation (CVL):

The date the company formally entered into liquidation.

Administration:

The date that the company files a notice of intention to appoint an administrator or the date it actually goes into administration if that is earlier.

Floating charge given to someone unconnected to the company:

The company must have been insolvent at the time the floating charge was given or become insolvent as a result.

Floating charge given to someone connected to the company:

It is not necessary to show that the company was insolvent when the charge was granted or that it became insolvent as a result.

How well did you know this?
1
Not at all
2
3
4
5
Perfectly
207
Q

What needs to be considered at Step 4 of a CGT calculation?

A

Gains and losses from all sources must be added together, and the annual exemption of £3,000 is deducted at this stage.

The annual exemption is the capital gain every CGT payer can make every year without being taxed on it.

Any unused part of the exemption cannot be carried forward to the following tax year.

How well did you know this?
1
Not at all
2
3
4
5
Perfectly
208
Q

Can a director count in the quorum to authorise a conflict of interest?

A

The director in question will not count in the quorum for the vote to authorise the infringement and if they vote their vote will not be counted even if MA 14 has been excluded.

How well did you know this?
1
Not at all
2
3
4
5
Perfectly
209
Q

With regards to sole traders and partnerships claiming trading loss relief, what happens if they have claimed all of the relief in that provision?

A

If they have claimed all the relief they can in that provision and there are still some unabsorbed losses, the taxpayer can claim relief for the balance of the loss under another provision if they are eligible.

Sometimes that loss is set against total income and sometimes it is set against a particular component of the income. Some of the reliefs are also subject to a cap for that tax year.

The partners will decide individually which reliefs they want to claim in relation to their share of the partnership’s losses. It is an individual decision.

The taxpayer must apply to the relief, they are not automatically applied by HMRC.

How well did you know this?
1
Not at all
2
3
4
5
Perfectly
210
Q

How can a shareholder agreement protect directors from removal from office?

A

There may also be a provision in the shareholder agreement obliging them to vote against the removal of their fellow shareholders from their office of director.

If a shareholder who was party to such an agreement does so, the former director/shareholder will have a claim in breach of contract of the shareholder’s agreement.

How well did you know this?
1
Not at all
2
3
4
5
Perfectly
211
Q

Why is it important to identify a person with ore than 25% but not more than 50% of the company’s shares/voting rights?

A

They can block special resolutions on their own as they have more than 25% of voting rights.

How well did you know this?
1
Not at all
2
3
4
5
Perfectly
212
Q

What trading loss reliefs can partnerships and sole traders claim?

A

Start-up relief/early trade losses relief

Carry-across/one year carry-back relief for trading losses generally

Set-off against capital gains

Carry forward relief

Carry back of terminal trading loss

Carry forward relief on incorporation of a business

How well did you know this?
1
Not at all
2
3
4
5
Perfectly
213
Q

How is a private company converted to a public company?

A

Must pass a special resolution to re-register the company and to alter the company’s name so it is suitable for a public company and altering article to they have a suitable form for a private company.

How well did you know this?
1
Not at all
2
3
4
5
Perfectly
214
Q

Avoidance of certain floating charges

What is the relevant time during which a floating charge made without fresh consideration may be avoided?

A

a) The charge was created in favour of a person who is connected with the company, during the two years ending with the onset of insolvency or

b) The charge was created in favour of any other person, during the 12 months prior to the onset of insolvency

How well did you know this?
1
Not at all
2
3
4
5
Perfectly
215
Q

When a partner leaves a partnership does that mean that they have have stopped working or claimed a pension?

A

No, just that they have left the partnership.

How well did you know this?
1
Not at all
2
3
4
5
Perfectly
216
Q

What are the options for the insolvent person?

A

a) Talk to creditors to come to an agreement regarding the payment of debt
b) Apply for their own bankruptcy to show that they are trying to take control of the situation
c) Apply for an individual involuntary arrangement
d) Apply for a debt relief order
e) Apply for debt respite scheme

How well did you know this?
1
Not at all
2
3
4
5
Perfectly
217
Q

What is the defence for transactions at an undervalue claim?

A

If the transaction was entered into in good faith for the purpose of carrying on the business and where the transaction was entered into there were reasonable grounds for believing that it would benefit the company.

How well did you know this?
1
Not at all
2
3
4
5
Perfectly
218
Q

Who is not allowed to vote in the CVA proposal?

A

The secured creditors unless they vote in relation to an unsecured debt.

How well did you know this?
1
Not at all
2
3
4
5
Perfectly
219
Q

In which events will a floating charge crystallise?

A

a) The chargor goes into receivership
b) The chargor goes into liquidation
c) The chargor ceases to trade
d) Any other event occurs which is specified in the charge document

The above makes it less likely that the chargor will not be able to repay the outstanding borrowings.

How well did you know this?
1
Not at all
2
3
4
5
Perfectly
220
Q

What must be made available to members during a GM or WR for the authorisation of a buyback of capital?

A

If the special resolution was proposed by WR, the statement and the report must be sent to the members along with the WR.

If by GM, a copy of the statement and report must be available for inspection at the meeting.

If this is not required with, the special resolution is ineffective.

How well did you know this?
1
Not at all
2
3
4
5
Perfectly
221
Q

Is a partnership a separate legal entity?

A

A partnership is not a separate legal entity. The partnership does not own any of the assets, they are owned by the partners.

How well did you know this?
1
Not at all
2
3
4
5
Perfectly
222
Q

How is administration commenced through the court route?

A

This is by court order following application to court and a court hearing.

The court will only make an administration order if it is satisfied that the company is likely to become unable to pay its debts and the administration order is reasonably likely to achieve one of the three purposes of the administration.

As soon as reasonably practicable after applying for the administration, the applicant must notify any person who has appointed or is entitled to appoint an administrative receiver of the company and any qualifying floating charge holder who is entitled to appoint an administrator.

How well did you know this?
1
Not at all
2
3
4
5
Perfectly
223
Q

What are the advantages of a term loan?

A

There is greater certainty than an overdraft which is repayable on demand and the borrower has greater control because the bank can only request repayment under the terms of the contract.

How well did you know this?
1
Not at all
2
3
4
5
Perfectly
224
Q

What is the minimum and maximum notice period for a s 303 request?

A

Minimum notice period is 14 clear days but maximum notice period is 28 days.

How well did you know this?
1
Not at all
2
3
4
5
Perfectly
225
Q

What happens if there is late or inaccurate delivery of the required charge documents to CH?

A

If the 21 day period for delivery of the required documents to CH is missed (weekends and bank holidays are included in the 21 days) or if the content of the form is incorrect, the charge is void against a third party.

How well did you know this?
1
Not at all
2
3
4
5
Perfectly
226
Q

Declaration of interest under s 177 v MA 14

A

When a company has misapplied MA 14, the obligation to declare an interest under s 177 remains. It cannot be disapplied by the company as it is a statutory provision.

How well did you know this?
1
Not at all
2
3
4
5
Perfectly
227
Q

How is CH notified of the movement of company statutory records from the SAIL address to company registered office?

A

Form AD04

How well did you know this?
1
Not at all
2
3
4
5
Perfectly
228
Q

How is CGT calculated (overview)?

A

Step 1: Identify the disposal (sale or gift) of a chargeable asset (part disposals are apportioned)

Step 2: Calculate the gain – deduct costs of disposal, initial and subsequent expenditure and incidental costs of disposal

Step 3: Consider reliefs. The main ones are:
* Relief on replacement of business assets (‘rollover’ relief)
* Rollover relief on incorporation of a business
* Hold- over relief on gifts
* Business asset disposal relief

Step 4: Aggregate gains/ losses and deduct the annual exemption (deduct the annual exemption from the gains which would be subject to the highest rate of tax). Capital losses carried over from the previous year can be deducted here.

Step 5: Apply the correct rate of tax:
* Standard rate of 10% for basic rate taxpayers and 20% for any gains above the basic
rate threshold;
* Residential property rate – apply a surcharge of 8%, meaning that the rates are
18% for basic rate taxpayers and 28% for any gains above the basic rate threshold; or
* Business asset disposal relief rate of 10%, whatever the taxpayer’s income.

How well did you know this?
1
Not at all
2
3
4
5
Perfectly
229
Q

What is the self-assessment of income tax?

A

Self- assessment is a system HMRC uses to collect income tax.

Anyone who receives any income from which the tax has not been deducted at source (eg rental income) must complete a tax return, declaring all their income for the tax year.

Any income tax which has been deducted at source should be included on the tax return and the taxpayer’s tax liability will be reduced by the amount of income tax that has already been paid.

Unfortunately for HMRC, the self- assessment method carries the risk that the taxpayer will spend the income received before HMRC has been paid, and there will not be enough left to pay HMRC.

How well did you know this?
1
Not at all
2
3
4
5
Perfectly
230
Q

When can a debt relief order not be used?

A

a) Debtor has total unsecured liabilities exceeding £30,000
b) Has total gross assets exceeding £2,000
c) Has a car worth £2,000 or more unless it has been adapted because the debtor has a disability
d) Has disposable income in excess of £75 per month after deducting normal household expenditure
e) Has been subject to a DRO in the preceding six years
f) Is subject to another formal insolvency procedure or
g) Owns their own home

How well did you know this?
1
Not at all
2
3
4
5
Perfectly
231
Q

What are examples of VAT exempt supplies?

A

a) Supplies of residential land
b) Postal services
c) Education and health services

A person who makes exempt supplies cannot register and will not be able to reclaim any VAT.

How well did you know this?
1
Not at all
2
3
4
5
Perfectly
232
Q

Who cannot become a director?

A

Cannot take office if they are disqualified from doing so.

The person will cease to be a director if a bankruptcy order has been made against them.

A doctor gives a written opinion stating that they have become physically or mentally incapable of acting as a director and may remain so for more than three months.

How well did you know this?
1
Not at all
2
3
4
5
Perfectly
233
Q

Dependant on specific facts, what forms may need to be sent to CH and when following allotment of shares?

A

Return of allotment and statement of capital (Form SH01) within one month of allotment

Possible forms for new persons of significant control or a percentage change of PSC (either update or removal)

How well did you know this?
1
Not at all
2
3
4
5
Perfectly
234
Q

What is an executive director?

A

Those who have been appointed to the board of directors and also have an employment contract with the company.

How well did you know this?
1
Not at all
2
3
4
5
Perfectly
235
Q

What number of directors must a public company have?

A

A public company must have at least two directors.

How well did you know this?
1
Not at all
2
3
4
5
Perfectly
236
Q

What does the balance sheet show?

A

The balance sheet shows the worth or value of the business by listing its assets and liabilities on the last day of the accounting period.

It must be headed with the date of preparation.

It is often described as a snapshot of the business, because it shows how much the business is worth on the day of preparation, and this could change the very next day. For example, a balance sheet showing as one of the business’s assets a debtor who owes £100,000 will not be worth as much as it seems if the very next day it becomes clear that the debtor is insolvent and there is no money left to pay its debts.

The calculation which the balance sheet essentially shows is:
Assets – Liabilities = Net worth of the business

The balance sheet does not just show assets in one section and liabilities in the other.

It has two sections, one showing the value of the assets less liabilities owed to third parties and the other showing the amount owed to the proprietor as capital.

These two amounts will be the same. It might help you to think of the top half as showing you where the money is – tied up in fixed assets, used to buy stock, in the business’s bank account, for ­ example – and the bottom half as showing you where the money came from – the owner’s initial capital contribution and profits earned since trading began, for example:

Employment of capital = where the money is now
Capital employed = where the money originally came from

As with the profit and loss account, the balance sheet calculation (Assets – Liabilities = Net
Worth) is shown vertically.

Assets and liabilities are also sub- divided:

a) Assets

Assets are split into fixed assets and current assets.

Fixed assets are used in the business to enable it to run effectively. Examples are business premises and machinery.

Current assets are short- term assets. Examples are stock, debts and cash.

b) Liabilities

Liabilities are divided into current and long- term liabilities.

Current liabilities are liabilities which are repayable in 12 months or less from the date of the balance sheet, for example, a bank overdraft or an invoice owed to a supplier.

Long- term liabilities are liabilities which are repayable more than 12 months from the date of the balance sheet. The most obvious example is a bank loan.

Assets appear on the balance sheet in increasing order of liquidity, that is, how easy it should
be to turn the business’s assets into cash, to meet its short- term liabilities. So fixed assets are at the top with current assets underneath. This means that any premises will always be at the very top of the balance sheet, at the top of fixed assets, and cash will always be at the bottom of current assets.

c) Net current assets

Net current assets is a key figure for a business. It shows the difference between current assets and current liabilities and it is an important figure because it shows the business’s liquidity.

d) Net assets

Net assets is another key figure, and is calculated by subtracting short- term and long- term liabilities from fixed and current assets.

This figure will always be equal to the amount owing to the business owner as capital at the end of the year.

e) Capital employed

The ‘Capital Employed’ section shows the value of the business to the owner. It consists of the balance on the capital account (which represents the amount put into the business by the owner(s)), and also the net profit from the profit and loss account (which represents the money the business has made over the year).

Sometimes the business owner(s) will have withdrawn money over the year, and this will be shown on the drawings account.

The balance on the drawings account is deducted from the other figures in the Capital Employed section.

Understanding the balance sheet is crucial for a business owner, because it enables them to
analyse the health of the business. If the business’s liabilities exceed its assets, it is clearly in financial difficulty.

Even if it has a high assets figure, it may have little cash to pay invoices, and may risk insolvency proceedings if it cannot pay a creditor.

How well did you know this?
1
Not at all
2
3
4
5
Perfectly
237
Q

Why would a company want to exclude MA 14?

A

The MA states that a director cannot vote in a board resolution if they have a personal interest in the matter and small companies may be hindered by this so they prefer to exclude the MA and submit their own articles when they incorporate the company or subsequent to the incorporation through a special resolution of the shareholders.

How well did you know this?
1
Not at all
2
3
4
5
Perfectly
238
Q

What is the relevant time for a preference?

A

a) If the preference was given to a person who is connected with the company during the two years ending with the onset of insolvency.

b) If the preference was given to any other person during the six months ending with the onset of insolvency.

How well did you know this?
1
Not at all
2
3
4
5
Perfectly
239
Q

What does an auditor do?

A

They are an accountant who’s main duty is to prepare a report on the company’s annual accounts to be sent to shareholders.

How well did you know this?
1
Not at all
2
3
4
5
Perfectly
240
Q

What is the avoidance of extortionate credit transactions (personal insolvency)?

A

This is an action a trustee can take to preserve and increase the bankrupt’s assets?

If the bankrupt has obtained any credit in the three years prior to the bankruptcy order and the terms of the credit are extortionate, the trustee can apply to set aside or vary the terms of the credit.

Extortionate means that the terms require grossly exorbitant payments or have grossly contravened fair dealing.

How well did you know this?
1
Not at all
2
3
4
5
Perfectly
241
Q

Is CGT charged on death?

A

There is no charge to CGT on death, because there is deemed to be no disposal of assets and the deceased’s personal representatives are deemed to acquire the deceased’s assets at their market value at the date of death.

How well did you know this?
1
Not at all
2
3
4
5
Perfectly
242
Q

What is the most common ground on which the petitioner will seek compulsory liquidation?

A

That the company has not been able to pay its debts by establishing one of the grounds for insolvency.

How well did you know this?
1
Not at all
2
3
4
5
Perfectly
243
Q

What is VAT?

A

Charged on any supply of goods and services made in the Unite Kingdom where it is a taxable supply made by a taxable person in the course of any business carried on by them.

Tax is charged on the value of the supply.

Supply of goods and services:

Any transfer of a whole property in goods is a supply of goods and this will include tangible goods such as interest in land or a supply of electricity.

Anything which is not a good is likely to be a service.

Taxable person:

A taxable person is a person who makes or intends to make taxable supples and who is required to be registered under VAT statute.

A person must be registered if the value of their taxable supplies in the preceding 12 months exceeded £90,000.

Course of business:

A business includes any trade, profession or vocation.

A supply in the course of business includes the disposal of a business or any of its assets.

Value of supply:

VAT is charged on the value of the supply of goods and services.

This is what the goods or services would cost if VAT were not charged.

A price is deemed to be VAT inclusive unless stated otherwise.

How well did you know this?
1
Not at all
2
3
4
5
Perfectly
244
Q

What is an alternative director?

A

They can be appointed if an director cannot attend a board meeting. They will vote in accordance with the wishes of the director who cannot attend.

There is no provision for this in the MA, a special article needs to be inserted.

How well did you know this?
1
Not at all
2
3
4
5
Perfectly
245
Q

What is the process of members voluntary liquidation?

A

Directors make statutory declaration of solvency

Members pass special resolution to liquidate the company and ordinary resolution to appoint liquidator. Directors powers cease.

Petition advertised in the Gazette.

Once the formalities of the meeting are concluded, the appointment will be published in the Gazette and the Registrar notified.

Liquidator investigates and reports to the creditors.

Creditors may appoint alternative liquidator if the majority are in favour and a resolution is passed to that effect.

Liquidator collections in assets and realises if necessary and then distributes in the statutory order.

Final accounts sent to creditors and members.

Final return filed with Registrar of Companies.

Company dissolved after three months.

How well did you know this?
1
Not at all
2
3
4
5
Perfectly
246
Q

What is the effect of a breach of authorisation for a substantial property transaction?

A

The transaction is voidable AND certain individuals may be ordered to account to the company for any gain that have made and to indemnify the company for any loss or damage resulting from the arrangement or the transaction.

How well did you know this?
1
Not at all
2
3
4
5
Perfectly
247
Q

What is the solution to a director not being able to act in a quorum on a vote concerning their own personal interests?

A

Change the company’s articles permanently by special resolution to allow the directors to vote even where they have a personal interest in the matter in question or just to allow them to vote when the subject under discussion is their service contract.

Or the shareholders could temporarily suspend the operation of the MA by ordinary resolution.

How well did you know this?
1
Not at all
2
3
4
5
Perfectly
248
Q

How is net income calculated?

A

This is step 2 of the calculation.

Total income less allowable reliefs = net income

Allowable reliefs are deducted from total income to give a figure for net income. The most significant example is interest payments on qualifying loans.

Most interest payments, for example, bank overdrafts and credit card interest, must be paid out of taxed income. But in certain cases, tax relief may be available for interest paid on money which the taxpayer has borrowed.

How well did you know this?
1
Not at all
2
3
4
5
Perfectly
249
Q

How is administration commenced through the out-of-court route - appointment by QFCH
?

A

A QFCH is a floating charge where the charge document states that it empowers the holder of the charge to appoint an administrator or it empowers the holder of the floating charge to appoint an administrative receiver

AND the charge document relates to the whole or substantially the whole of the company’s property (or does so when the other charges held by the lender are aggregated).

If there is another QFCH who would have priority, the lender must notify them in advance and give them the opportunity to appoint an administrator if they want to.

The floating charge must be enforceable i.e. the charge holder must be entitled under the loan agreement to enforce the security (usually because of late payment but there will be other default events).

The lender must file the notice of appointment at court along with certain documents.

The notice of appointment must include a statutory declaration by the lender stating that:
a) The lender is a holder of QFC in relation to the company’s property
b) The floating charge is enforceable
c) The appointment complies with statute

The administration only begins when those documents are filed at court.

How well did you know this?
1
Not at all
2
3
4
5
Perfectly
250
Q

What is the CGT stance on tangible moveable property?

A

Wasting assets are generally exempt from CGT. Wasting assets are assets with a predictable life of less than 50 years. This includes most consumer goods, for example kitchen appliances and televisions.

Tangible property that does constitute a wasting asset:

Not all tangible moveable property constitutes a wasting asset: some assets, such as antiques, will go up in value. These assets are exempt from CGT if the consideration for the disposal is £6,000 or less.

How well did you know this?
1
Not at all
2
3
4
5
Perfectly
251
Q

Will there be a provision for pre-emption rights in companies articles?

A

There are no pre-emption rights in the MA so if there are pre-emption rights they will be a special article.

How well did you know this?
1
Not at all
2
3
4
5
Perfectly
252
Q

What happens if the court orders that the company be wound up in compulsory liquidation ?

A

The Official Receiver will automatically become the company’s liquidator. The OR is a civil servant or court official who is employed by the insolvency service.

The OR can appoint a private insolvency practitioner, depending on the nature of the case and the creditors’ wishes as long as the company has sufficient assets to pay the insolvency practitioner’s fees.

How well did you know this?
1
Not at all
2
3
4
5
Perfectly
253
Q

Who is the company secretary?

A

They are an officer of the company and it is possible to have a corporate company secretary in which case the secretary will act through a human being authorised by the company appointed as company secretary.

Some companies decide to have more than one company secretary and they will act as a joint company secretary.

How well did you know this?
1
Not at all
2
3
4
5
Perfectly
254
Q

What is a revolving credit facility?

A

The bank agrees to make a maximum amount of money available to a business throughout the agree period of the revolving credit facility.

During the lifetime if the facility, the business can borrow and repay money.

Interest is payable at regular intervals.

The business is also able to reborrow amounts that it has already repaid so long as it does not exceed the overall maximum figure.

The contract for a credit facility is usually called a facility agreement.

How well did you know this?
1
Not at all
2
3
4
5
Perfectly
255
Q

What are the civil consequences for breach of director’s duties?

A

Account of profits

Equitable compensation for the loss suffered by the company

Rescission of any contract entered into as a direct or indirect result of the breach

An injunction, to prevent further breaches/a continuing breach

Restoration of property transferred as a result of the breach of duty

Breach of s 174 is the same as negligence so the remedy for breach of duty to exercise reasonable care, skill and diligence is common law damages assessed in the same way as damages for negligence.

How well did you know this?
1
Not at all
2
3
4
5
Perfectly
256
Q

What sources of income (non-comprehensive) do income taxes have to paid on?

A

(a) trading income: profits of trade, profession or vocation. This applies to sole traders, trading partnerships, sole practitioners and professional partnerships;
(b) property income: rents and other receipts from land in the UK;
(c) savings and investment income: interest, annuities and dividends;
(d) employment and pensions income, including social security payments such as sick pay and maternity pay; and
(e) certain miscellaneous income.

How well did you know this?
1
Not at all
2
3
4
5
Perfectly
257
Q

How are LLPs taxed in terms of income tax?

A

When an LLP is used to carry on a trade or profession, it will be treated for most purposes in the same way as an ordinary partnership as far as income tax is concerned. However, the availability of relief for trading loss is restricted for partners in an LLP in certain conditions.

How well did you know this?
1
Not at all
2
3
4
5
Perfectly
258
Q

Allotment of shares

If a company was incorporated pre CA 2006, what needs to be considered with regards to any constitutional restrictions on allotment?

A

If the company was incorporated pre October 2009 and the company’s articles have not been updated, then the company will have an authorised share capital in the company’s articles.

It can be removed by ordinary resolution and a copy of the resolution must be sent to CH.

How well did you know this?
1
Not at all
2
3
4
5
Perfectly
259
Q

Regarding the additional requirements to authorise buyback out of share capital, when must company directors make a statement of solvency?

A

Company directors must make a statement of solvency no sooner than one week before the general meeting stating that the company is solvent and that it will remain solvent during the year following the buyback.

If this is a false statement, the directors may face criminal sanctions and the seller of the shares and the directors may have to contribute to the financial losses of the company.

How well did you know this?
1
Not at all
2
3
4
5
Perfectly
260
Q

What is double entry bookkeeping?

A

Businesses must keep records of every financial transaction, otherwise they will not be able to work out how much they have spent or earned.

They need to record their day- to- day financial transactions in order to prepare accurate accounts. The recording of a business’s financial transactions is known as bookkeeping. All businesses use the double entry bookkeeping system.

It is based on the principle that there are two aspects to every transaction – for example, if a business buys premises, it both acquires premises and has less cash. If a business receives payment for an outstanding invoice, it has more cash and loses the debt owed to it. The two aspects of every transaction are recorded in the business’s books, which have a left- hand column and a right- hand column.

The left- hand column is called the debit column (shortened to DR) and the right- hand column is called the credit column (shortened to CR).

Under the double entry system, the business has separate accounts for each aspect of the business. For example, it will have one for cash, one for each type of expense, and one for each debtor. Every single transaction is recorded as it occurs.

Bookkeepers check the accuracy of the books at regular intervals, perhaps daily or monthly, perhaps weekly. They will also check the business’s books at the end of its accounting period, which is usually one year. They will add all the DR balances and all the CR balances on the accounts. If there are no errors in the books, the two figures should be the same.

This process of adding together all the DR and CR entries is called preparing the trial balance.

Preparing a trial balance is the first step in preparing the business’s final accounts, which are
its year- end summary accounts.

How well did you know this?
1
Not at all
2
3
4
5
Perfectly
261
Q

What is subordination?

A

It is possible for lenders to enter into an agreement between themselves to alter the order of priority of their charges.

This is known as subordination and the agreement is known as a deed of priority.

It is executed by the creditors concerned and sometimes the company.

This may happen if the bank would only advance new funds to allow a borrower to trade if the bank could have priority.

How well did you know this?
1
Not at all
2
3
4
5
Perfectly
262
Q

What is the process of administration?

A

Once the administration has started the moratorium comes into effect.

The administrator puts forward the proposals for the company to the creditors.

The creditors can ask for more information on the amended proposals, especially if the administrator has been appointed by the directors.

Creditors can suggest amendments to the proposals.

The administrator’s proposals will be approved if a majority in value of the creditors, present and voting, vote in favour of them, provided that those who vote against the proposals do not constitute more than 50% in value of the creditors who are unconnected to the company.

How well did you know this?
1
Not at all
2
3
4
5
Perfectly
263
Q

When is a person insolvent in personal insolvency?

A

a) A debt is payable but the debtor does not currently have enough money to pay
b) A debt is payable in the future and there are no reasonable prospect that the individual will be able to pay

How well did you know this?
1
Not at all
2
3
4
5
Perfectly
264
Q

What rights other than the power to vote can a shareholder exercise?

A

Right to receive dividends (as long as there are profits available for this and the directors have made a recommendation as to its amount and this has been approved by shareholders)

Right to apply to court for the company to be wound up on the grounds that it is just and equitable to do so

Right to remove director by ordinary resolution

Right to remove auditor by ordinary resolution

Right to inspect without charge - company minutes of GM and all shareholder resolutions passed otherwise than at GMs, all of company’s statutory registers, directors service contracts and any director’s indemnities and any contracts relating to the company’s of its own shares

Right to receive a copy of the company’s annual accounts and reports

Right to seek an injunction to restrain company from doing something that is prohibited by the constitution

How well did you know this?
1
Not at all
2
3
4
5
Perfectly
265
Q

How does s 182 differ from s 177?

A

The declaration must be made at a meeting of the directors or by notice in writing sent to all the other directors or by general notice of the interest given at a board meeting.

How well did you know this?
1
Not at all
2
3
4
5
Perfectly
266
Q

What happens if the share transferee is never entered onto the register of members?

A

Shares are deemed issued when the shareholder is entered on the register of members.

If the transferee is never entered onto the register of members, they will be the beneficial owner of the shares, but the transferor will remain the legal owner of the shares.

Who can attend a GM and receive dividends in this case?

The transferor as the legal owner.

However, the transferor must vote in accordance with the wishes of the beneficial owner of the acres and must pay any dividends to the beneficial owner.

How well did you know this?
1
Not at all
2
3
4
5
Perfectly
267
Q

Does a partnership have liability in tort?

A

Under the PA 1890, the firm is liable for any wrongful act or omission of a partner who acts in the ordinary course of the firm’s business or with authority of their partners.

How well did you know this?
1
Not at all
2
3
4
5
Perfectly
268
Q

What are the two key shareholder resolutions where the votes of a shareholder with a personal interest in the matter are not counted?

A

A resolution to buy back some or all of a shareholder’s shares

An ordinary resolution to ratify a director’s breach where the director in question is also a shareholder

How well did you know this?
1
Not at all
2
3
4
5
Perfectly
269
Q

What is bankruptcy?

A

The process of the debtor’s assets passing to a trustee in bankruptcy whose job it is to pay as many of the debts as possible to the debtor’s creditors.

The debtor is known as the bankrupt. They are subject to restrictions on their activities and spending during the bankruptcy process and possibly afterwards.

How well did you know this?
1
Not at all
2
3
4
5
Perfectly
270
Q

What are distributable profits?

A

These are the company’s accumulated, realised profits less its accumulated, realised losses. They are shown on the bottom half of the company’s balance sheet under profit/loss reserve.

How well did you know this?
1
Not at all
2
3
4
5
Perfectly
271
Q

How are dividends presented in the business accounts of a company?

A

Companies are only permitted to pay dividends from profits (although they could be paid from previous years’ accumulated profits, rather than the current year).

However, companies are unlikely to distribute all of their net profit. This is partly because they need money to pay expenses and run the business. Secondly, much of the company’s profit may not be in the form of cash.

Debtors are counted as an asset, so will be factored into the net profit calculation in the profit and loss account, but they do not produce any cash for the company until they pay. Or perhaps the company used some of the money it made during its accounting period to buy a fixed asset.

If this is the case, the profit will be shown in the profit and loss account, but will not be represented by cash sitting in the company’s bank account; instead, it has been ploughed into a fixed asset.

Dividends as such will not be included as an item in the profit and loss account but, once paid, the cash figure will be reduced accordingly.

How well did you know this?
1
Not at all
2
3
4
5
Perfectly
272
Q

How are written resolutions weighted?

A

Each shareholder has one vote per share that they own.

How well did you know this?
1
Not at all
2
3
4
5
Perfectly
273
Q

Does a company need to be insolvent to apply for a restructuring plan?

A

No but they must have encountered or be likely to encounter financial difficulties.

How well did you know this?
1
Not at all
2
3
4
5
Perfectly
274
Q

How is personal insolvency proven?

A

a) Serving a statutory demand on the debtor for a liquidated sum of £5,000 or more and waiting three weeks to see whether the debtor pays or applies for the court to set aside the statutory demand
b) Serving a statutory demand on the debtor in respect of a future liability to pay a debt of £5,000 or more and waiting three weeks to see whether the debtor either:
I) Shows a reasonable project of being able to pay the sum when it falls due
II) Applies to court to set aside the statutory demand
c) By obtaining a court judgment of £5,000 or more and attempting execution of the judgment without success

How well did you know this?
1
Not at all
2
3
4
5
Perfectly
275
Q

What kind of clauses will a lender want in the facility agreement concerning the payment of the loan to the borrower?

A

The initial clauses of the facility agreement will set out:
a) The amount of the loan
b) The currency
c) The type of loan if it is to be a term loan or revolving credit facility
d) The availability periods during which the loan can be taken (for revolving credit facility this is almost the entire length)

How well did you know this?
1
Not at all
2
3
4
5
Perfectly
276
Q

What are the administration and notification requirements regarding the change of a company secretary?

A

The company must notify the Registrar of Companies within 14 days of any change of particulars of the company secretary kept in the register of secretaries on form CH03 (human secretary) and CH04 (corporate secretary).

How well did you know this?
1
Not at all
2
3
4
5
Perfectly
277
Q

What is the accounting reference date?

A

This is the date up to which it must prepare annual accounts and will be the last day of the month in which the company was incorporated.

e.g. if company is incorporated 5 May, its accounting reference date will be the 31 May.

If they want to change this they need to make a board resolution and complete AA01 form and file it at CH.

How well did you know this?
1
Not at all
2
3
4
5
Perfectly
278
Q

How are assets distributed in a personal insolvency?

A

Secured creditors can sell their charged assets and take what they are owed and pay any surplus to the trustee.

If the sale of the charged assets does not realise enough money to pay the secured creditor, the entire amount owed to them, the secured creditor will join the unsecured creditors in relation to the outstanding part of the debt.

Order of distribution:

a) The costs of the bankruptcy - means all of the expenses incurred as a result of the bankruptcy which will mainly be the trustees professional charges and disbursements
b) Preferential debts
c) Ordinary unsecured creditors
d) Postponed creditors who are the bankrupt’s spouse or civil partner

Within each category, the creditors will rank and abate equally, meaning that they will share in the money available.

Each creditor will receive the same percentage of the debt owed to them.

Preferential debts:

This includes the wages and salaries of employees for work the carried out in the four months preceding the date of the bankruptcy of their employer up to a max of £800 plus accrued holiday pay owed to employees. Not all bankrupts will have employees only those who were sole traders partners.

HMRC is a secondary preferential creditor.

How well did you know this?
1
Not at all
2
3
4
5
Perfectly
279
Q

What is the distinction between a listed public company and an unlisted public company?

A

Publicly traded companies are more regulated than unlisted public companies and private companies to protect the public.

Unlisted companies can also offer shares to the public but because their shares are not listed it is harder for unlisted public companies to find buyers.

How well did you know this?
1
Not at all
2
3
4
5
Perfectly
280
Q

Who uses overdraft facilities?

A

Usually small and medium sized businesses.

How well did you know this?
1
Not at all
2
3
4
5
Perfectly
281
Q

What can a director do to minimise the risk of a claim for wrongful trading? A solicitor may need to advise on this

A

Director should:
a) Seek professional advice from solicitors and/or accountants at the first sign of problems.
b) Limit spending
c) Check the company’s accounts regularly
d) Keep records of their own actions

How well did you know this?
1
Not at all
2
3
4
5
Perfectly
282
Q

What are the grounds for an unfair prejudice petition?

A

The company’s affairs have been conducted in a manner that is unfairly prejudicial to the interests of the members generally or some part of its members including the claimant or

An actual or proposed act or omission of the company is or would be so prejudicial.

The conduct must be prejudicial in the sense that it causes harm to one or more shareholders and it must also be unfair.

How well did you know this?
1
Not at all
2
3
4
5
Perfectly
283
Q

Who is liable if a charge is not registered in time?

A

If the failure to register was due to the solicitor’s mistake, then this could lead to a claim in negligence.

How well did you know this?
1
Not at all
2
3
4
5
Perfectly
284
Q

Can the PSCs request more privacy on the PSC register?

A

PSCs can apply to keep their residential address private so it does not appear in public registers at CH.

They can also apply to have their name private.

How well did you know this?
1
Not at all
2
3
4
5
Perfectly
285
Q

How are a company’s assets distributed during liquidation?

A

1) Fixed charges holders

These are paid first. They will receive the amount they are owed when the asset subject of the fixed charge was sold.

Any surplus is paid to the liquidator.

What happens if there is a shortfall once the asset has been sold?

If there is a shortfall instead, the fixed charge holder can join the pool of unsecured creditors and try and obtain some kind of contribution towards the outstanding debt.

1) Unsecured creditors

Administration:

The liquidator will send a standard form to unsecured creditors. They fill this in outlining the debts that are owed to them to prove their debt.

The liquidator then approves or rejects the creditor’s debts.

If the claim is under the sum of £1,000 these claims are admitted automatically.

Once the liquidator has told the company’s assets and collected as much money as they can and paid the holders of valid fixed charges, they will make the payments in the following order.

Order of distribution:

a) Expenses of the winding up (fees payable to the liquidator and their professional advisors)
b) Preferential debts which rank and abate equally
c) Money which is the subject of a floating charge in order or priority
d) Unsecured creditors who rank and abate equally.
e) Remaining money is distributed to shareholders

Preferential creditors:

The most common preferential debt are the wages/salaries of employees for work carried out in the four months immediately preceding the date of the winding up order.

There is a maximum of £800 per employee.

Employees’ accrued holiday pay is a preferential debt.

HMRC is a preferential creditor in relation to PAYE and VAT but only these.

Ring fencing:

Statutory procedure whereby by a portion of money for floating charge holders is set aside for be benefit of unsecured creditors.

This only applies if the security was created on or after 15 September 2003.

The amount set aside is 50% of the first £10,000 of money received from the property which is subject to floating charges and 20% of the remaining money up to a limit of £800,000 if the share was made on or after 6 April 2020. If the charge was made before that the upper limit is £600,000. However if the floating charge that was made on or after 6 April 2020 ranks equally or in priority ti the pre-April 2020 charge, the limit of £800,000 will apply to both charge holders.

How well did you know this?
1
Not at all
2
3
4
5
Perfectly
286
Q

Dependant on specific facts, what resolutions may need to be sent to CH and when following allotment of shares?

A

All special resolutions

An ordinary resolution removing the authorised share capital in a pre-CA 2006 company

Any ordinary resolution to activate s 550 in a pre-CA 2006 company

Any s 551 ordinary resolution granting directors authority to allot

When?

Within 15 days

How well did you know this?
1
Not at all
2
3
4
5
Perfectly
287
Q

What is an event of default clause?

A

If the business breaches any of these terms, the lender may terminate the agreement if it so wishes.

Such events include failure to pay any sum due, commencement of an insolvency procedure or breach of other obligations under the facility agreement.

How well did you know this?
1
Not at all
2
3
4
5
Perfectly
288
Q

Can a director act in a quorum on a vote concerning their own personal interests?

A

If a company has MA and the company only has two directors, they cannot approve a service contract at a board meeting as the meeting must be quorate (have two directors) and only one can count in the meeting as the other has a personal interest.

How well did you know this?
1
Not at all
2
3
4
5
Perfectly
289
Q

What are the administrative requirements of paying corporation tax?

A
  1. Inform HMRC

A company must inform HMRC in writing of the beginning of its first accounting period, and must do so within three months of the start of that accounting period.

After that, every year HMRC will issue a notice to the company requiring it to deliver a self- assessment corporation tax return.

  1. Payment

The deadline for filing the self- assessment return with HMRC is 12 months from the end of the relevant accounting period.
For most companies, corporation tax is payable within nine months and one day from the end of the relevant accounting period.

This means that the company is required to pay corporation tax to HMRC before it is required to file its tax return.

Therefore companies will make a payment based on their anticipated corporation tax liability for the period, and will make a balancing payment (or receive a rebate) once the final figure for corporation tax has been established.

Large companies, meaning those with annual taxable profits of £1,500,000 or more, usually (depending on the company’s overall corporation tax liability) have to pay tax in four
instalments.

Very large companies, which means those with annual taxable profits of over £20,000,000,
have to pay the tax in four instalments during the accounting period (different dates compared to companies with annual taxable profits of £1,500,000 or more).

How well did you know this?
1
Not at all
2
3
4
5
Perfectly
290
Q

What are the advantages of overdraft facilities?

A

An overdraft is known as an uncommitted facility which means that it will usually be payable on demand without notice which can be advantageous if there is immediate need.

It is flexible and there are few formalities.

How well did you know this?
1
Not at all
2
3
4
5
Perfectly
291
Q

How is it payable?

A

Payment can be made using HMRC’s real- time CGT service, the HMRC property service or by completing a self- assessment tax return in the same way as is required for income tax.

Sometimes (although rarely) payment by ten annual instalments is an option. This is only available where:
- the disposal was a gift;
- the qualifying asset is land, a controlling shareholding in any company or any shareholding (whether controlling or not) in a company whose shares are unquoted; and
- the conditions for hold- over relief to apply must not be met.

If payment by instalments is an option, the first payment will be due by 31 January following the end of the tax year in which the disposal was made.

How well did you know this?
1
Not at all
2
3
4
5
Perfectly
292
Q

What is the purpose of business accounts?

A

The business owner (or a bookkeeper) will use all of the business’s financial records – statements, invoices, receipts – to prepare the business’s accounts.

The accounts provide a useful summary of the business’s financial position.

There are two common meanings of the word ‘accounts’. The first is the day- to- day records of the business’s transactions, and the second is the accounting year end summary (Final Accounts).

How well did you know this?
1
Not at all
2
3
4
5
Perfectly
293
Q

In bankruptcy proceedings, what happens to the debtor’s salary?

A

Bankrupts are entitled to be paid their salary but if their salary is more than what is sufficient to meet the reasonable needs of the bankrupt and their family, the trustee can ask the bankrupt to enter into income payment agreements (IPA) which requires the bankrupt to pay some of their salary to the the trustee to meet their liabilities,

Agreement of salaried sum to be paid to trustee:

If they cannot agree on the sum to be paid, the trustee can apply to court for an income payments order (IPO) and the court will determine the amount payable.

An IPA is enforceable just as an IPO.

They normally last for a maximum of three years from the date of the arrangement/order. In most cases they last for longer than the debtors discharge from the bankruptcy.

How well did you know this?
1
Not at all
2
3
4
5
Perfectly
294
Q

Can a director be compensated for loss of office?

A

They may be legally entitled to payment as part of the service contract terms.

They may also be entitled to compensation if there is an element of unfair or wrongful dismissal or discrimination.

Payment for more than £200 for loss of office can only be agreed to by ordinary resolution.

How well did you know this?
1
Not at all
2
3
4
5
Perfectly
295
Q

What happens if a partner of a general partnership cannot pay back their debts to their fellow partners?

A

If the partner cannot pay what is owed to their fellow partners, the other parters have the same enforcement mechanism as the third party and there may be other consequences set out in the partnership agreement such as expulsion.

How well did you know this?
1
Not at all
2
3
4
5
Perfectly
296
Q

Do companies incorporated pre 2009 have model articles?

A

They may do if they have if they have modernised their articles to adopt the MA or an amended version of the MA. If not then they will have Table A articles which were the articles under CA 1985.

How well did you know this?
1
Not at all
2
3
4
5
Perfectly
297
Q

What is a sleeping partner in a partnership?

A

They do not make day-to-day decisions and only make fundamental decisions about the business.

How well did you know this?
1
Not at all
2
3
4
5
Perfectly
298
Q

What is the definition of a partnership?

A

A partnership is formed when two or more people are carrying on a business in common with a view to profit.

How well did you know this?
1
Not at all
2
3
4
5
Perfectly
299
Q

When does the tax year run from?

A

The tax year runs from 6 April until 5 April the following year. This means that an individual will pay tax on all income earned between these dates.

The tax year is described by reference to the calendar years which it straddles. So the tax year
beginning on 6 April 2023 and ending on 5 April 2024 is referred to as the tax year 2023/24.

How well did you know this?
1
Not at all
2
3
4
5
Perfectly
300
Q

What happens if the requirement to authorise a loan to directors is breached?

A

The transaction/arrangement is voidable at the instance of the company. *

The director who loan the money from the company and any director who authorised it are liable to account to the company for any gain they have made and are jointly and severally liable to indemnify the company for any loss or damage resulting from the transaction or arrangement.

It is unlikely the board will avoid it if they authorised it. It is more likely that if the company becomes insolvent, an insolvency practitioner would avoid it to pursue the director for immediate repayment of the loan.

How well did you know this?
1
Not at all
2
3
4
5
Perfectly
301
Q

What is the deadline for updating the register of members?

A

The deadline is two months after the transfer was lodged with the company.

If the company has elected to keep this information at Companies House, they must instead notify the registrar of the share registration as soon as practicable with a deadline of two months.

How well did you know this?
1
Not at all
2
3
4
5
Perfectly
302
Q

Who has the authority to act in a winding up of a partnership’s business affairs?

A

All of the partners (unless bankrupt) have authority to act in winding up the business’s affairs. If any of the partners are bankrupt or deceased, the trustee in bankruptcy or PR can also make such an application.

303
Q

How are pre-emption rights disapplied with private companies with one class of share ?

A

Companies which had the authority to allot shares under s 550 can disapply pre-emption rights by special resolution.

304
Q

What is the purpose of a company secretary?

A

They deal with the company’s legal administrative requirements. They will generally write up the company’s board meetings and minutes of general meetings.

They will normally have apparent authority to enter into contracts of an administrative nature but not trading contracts.

305
Q

What are the disadvantages of a floating charge?

A

The fixed charge takes a priority over the floating charge over the same assets.

From the lender’s point of view, a disadvantage is that the chargor is able to deal with the assets. The company could sell its existing stock and not purchase new stock to replace it. One solution is for the lender to take a fixed charge over other assets in the business too. The more assets they have a charge over, the more chance they have of getting paid.

Preferential creditors have the right to claim money from the proceeds of sale of the assets covered by the floating charge if the company.LLP becomes insolvent before the charge holder itself gets the money.

In certain circumstances, a liquidator/administrator of an insolvent company may apply to have a floating charge set aside.

If this is done, this would have the effect of removing the floating charge holder’s priority over unsecured creditors of the company.

306
Q

What is the relevant time for a transaction at an undervalue?

A

Two years ending with the onset of insolvency.

The company must have been insolvent at the time of transaction or have become insolvent as a result of the company entering into the transaction.

307
Q

What is the purpose of pre-emption rights?

A

Pre-emption rights allow existing shareholders to preserve their percentage shareholding in the company. This is to avoid the dilution of their voting power.

308
Q

Can the debtor dispute the winding up petition in a compulsory liquidation?

A

If the company can show that there is a genuine and substantial dispute in relation to the money owed the petitioner will be prevented from proceeding with a winding up petition.

This will be difficult if the petitioner has already obtained judgment, unless the company can successfully argue to have the judgment set aside.

However the statute does make it clear that the court may still wind up the company if the company is unable to pay its debts so court has ultimate discretion.

If the company indicates that it will be able to pay the debt within a reasonable period of time, the court may adjourn the hearing to a later date.

309
Q

When can a derivative claim be brought?

A

May only be brought in relation to a cause of action arising from an actual or proposed act or omission involving negligence, default, breach of duty or breach of trust by a director.

310
Q

What form needs to be sent to CH to register a legal entity on the PSC register for the first time?

A

Form PSC02

311
Q

What are the administrative requirements post agreement for share buyback?

A

File return of purchase of own shares and notice of cancellation of shares within 28 days of completion

Keep copy of contract at registered office for ten years

Cancel the shares, update register of members and PSC register if required.

312
Q

How are groups of companies taxed?

A

Given that each company in a group of companies is a separate legal entity, each company will be charged to tax separately. However, tax legislation exists to try to ensure that operating as a group of companies is tax neutral; that is, it is no more or less advantageous from a tax point of view than trading as a single company.

  1. Group relief

This relief allows the company to transfer certain losses and expenses to another company within the same qualifying group.

The transferee will then use the loss or expense to reduce its taxable profit. Both companies must fit the definition of a group in order for this relief to be used.

1.1. What is a group for the purposes of group relief?

One company must be the 75% subsidiary of the other, or both companies must be 75% subsidiaries of a third company.

The test for being a 75% subsidiary essentially means that the holding company must own, directly or indirectly, 75% or more of the subsidiary’s ordinary shares.

1.2. How is this worked out?

A owns 80% of B and B owns 80% of C.

A and B are in the same group, and so are B and C. A and C are not in the same group because A does not own, directly or indirectly, 75% or more of C’s ordinary shares. A has an indirect shareholding of 64% in C (80% x 80% = 64%).

1.3. How does group relief work?

Once it has been established that two companies are in the same group, one company (the transferor) can surrender certain items, including trading losses and management expenses, to the other company (the transferee).

The loss or expense must have been incurred in an accounting period that overlaps with the accounting period of the transferee (who will be using the loss or expense from the other company to reduce its profits in that accounting period).

There are some restrictions on the application of the relief when the transferor has other profits, or the transferee has losses of its own. In addition, group relief does not apply to capital losses, only income losses.

313
Q

When happens to the debtor’s house when the bankruptcy order ends?

A

Three years after the bankruptcy order, ownership of the home transfers back to the bankrupt unless the trustee has:

a) Sold the property
b) Applies for an order for sale or possession or a charging order over the house
c) Entered into an agreement with the bankrupt regarding the home e.g. they can keep their interest in the house in exchange for payment

314
Q

What is the director’s liability if the company’s secretary fails to carry out their duties?

A

The directors are still liable for the company secretary failing to carry out their duties.

315
Q

Allotment of shares

If a company was incorporated under CA 2006, what needs to be considered with regards to any constitutional restrictions on allotment?

A

They will not have an ASC clause but they may have a similar clause in their articles to create an upper limit on the amount of shares that can be allotted.

If there is one, this will need to be removed by special resolution.

316
Q

What is tax avoidance?

A

Tax avoidance is using tax law to the taxpayer’s benefit to reduce their tax bill, in a way which is arguably lawful but not within the spirit of tax legislation.

This is different from tax evasion, which is using illegal methods to reduce tax payments, and which is targeted by domestic legislation and international treaties.

317
Q

What are the administration requirements for a loan to directors?

A

A memorandum setting out the terms of the loan and the company’s liability must be made available for inspection at the company’s registered office for 15 days prior to the general meeting at which the ordinary resolution will be proposed and at the general meeting itself.

If the board wishes to propose the ordinary resolution by written resolution, a copy of the memorandum must instead be sent out with the written resolution and does not need to be available for inspection at the company’s registered office.

318
Q

What is deductible expenditure used in calculating trading profits?

A

Deductible expenditure must be of an income nature and be incurred wholly and exclusively for purposes of trade.

Its deduction must not be prohibited by statute e.g. client entertainment and leasing cars with emissions over a certain level.

What is income in nature?

If the reason for incurring the expenditure is so that the business can sell the item at a profit, it is income in nature.

If the expenditure has the qualify of recurrence e.g. utility bills, it will be come in nature.

What does ‘wholly and exclusively for the purposes of trade’ mean?

Examples of commonly deductible expenditure:
a) Salaries
b) Rent on commercial
services
c) Utility bills
d) Stock
e) Contribution to an approved pension scheme for directors/employees
Interest payments on borrowings

319
Q

Following allotment of shares, what is the deadline for allocating share certificates?

A

Prepare share certificates within two months of allotment.

320
Q

What are trading accounts?

A

Businesses which buy and sell goods have a preliminary account called a trading account, in addition to their profit and loss account.

Businesses engaged in the provision of professional services, like law firms, do not need a trading account.

The trading account shows gross profit by subtracting the cost of sales (the cost of buying trading stock) from the income received from sales.

This figure is the business’s gross profit.

The gross profit is then transferred to the profit and loss account.

Any other income, and other expenses such as utility bills, will also be added to the profit and loss account and factored into the calculation of net profit.

Having a trading account as well as a profit and loss account means that businesses can see where the problem lies if they are not satisfied with their net profits.

The trading account will enable them to assess whether their business is buying stock at too high a price or whether it is its other expenses, shown on the profit and loss account, which are reducing its net profit by so much.

321
Q

How does one become a shareholder (post incorporation)?

A

a) The shareholder could obtain shares from an existing shareholder by i) buying shares from a shareholder, ii) receiving them as a gift, receiving the shares by way of transmission when a shareholder dies or becomes bankrupt.

b) Company allots new shares.

322
Q

What is the time period for challenging a preference (personal insolvency)?

A

The trustee can challenge any potential preferences within the six months prior to presentation of a bankruptcy petition or within the two years prior if the preference is in favour of an associate.

The bankrupt must have been insolvent at the time of the preference or became insolvent as a result of it.

323
Q

What are the advantages of an IVA for the creditor?

A

Low cost compared with bankruptcy, more straightforward, greater returns. But needs to consider whether the bankrupt can be trusted to honour the IVA. If not the creditor may be able to petition for the debtor’s bankruptcy at a later date.

324
Q

What is the deadline for filing accounts for public companies?

A

6 months from end of the accounting reference period for a public company.

325
Q

What are the exceptions to the duty to declare interest in a proposed transaction or arrangement?

A

Director is not aware of the interest or of the transaction or arrangement in question (and a director is treated as being aware of matters of which he ought reasonably be aware)

If the interest cannot reasonably be regarded as likely to give rise to a conflict of interest

The other directors are already aware of it (or ought reasonably be aware of it)

If it concerns the terms of the director’s service contract

326
Q

What is the default position under statute regarding the management of an LLP?

A

Every member is may take part in the management of the LLP.

Members are not entitled to remuneration for taking part in management.

327
Q

How can a sole director make decisions given the quorum rule?

A

Whilst the quorum for directors’ meetings is two, in companies with only one director, the director can still validly take company decisions because the MA allows them to make decisions without calling a board meeting.

328
Q

What administrative and filing provisions are required when an LLP creates a charge?

A

Register of charges:

LLPs need to keep a register of charges and a copy of every charge requiring registration at its registered office.

Any creditor or member of the LLP must be allowed to inspect the register without paying a fee.

Register the charge:

LLPs must also register the charge with the Registrar of Companies and which form to use depends on the nature of the charge.

329
Q

How is corporation tax paid on dividends?

A

When a company receives dividends from shares it has in another UK company, in principle they are taken into account when calculating its corporation tax liability.

However, there are a number of exceptions whose effect is to exempt most dividends from corporation tax, preventing double taxation.

Double taxation would otherwise arise on distributing profits that have already been assessed to tax on the company which paid the dividends.

330
Q

How is a moratorium commenced?

A

The directors must file the relevant documents at court and the proposed monitor must also confirm that it is likely that the moratorium will result in rescue of the company as a going concern.

331
Q

How is tax presented in the business accounts of a company?

A
  1. Unincorporated businesses

Tax is not shown in the accounts of unincorporated businesses because the businesses themselves do not have a tax liability.

It is the sole trader or partners who have a personal liability for income tax or CGT.

  1. Incorporated businesses

Companies have their own liability to corporation tax, and this is shown in the accounts.

Unless they are large companies, which pay two of their instalments of corporation tax during the accounting period to which the tax relates, companies do not have to pay corporation tax until nine months after the end of the accounting period.

This means that when the final accounts are prepared, at the end of an accounting period, the company will not yet have paid any tax on its profits from the accounting period. The profit and loss account will show that a certain sum is needed to pay the company’s corporation tax bill. It will do this by showing the company’s profits before and after tax.

The tax due to HMRC will appear on the balance sheet as a current liability.

When the tax is paid, the ‘provision for tax’ will disappear from the current liabilities section and cash will decrease in the current assets section, to reflect the fact that the company has used cash to pay its tax bill.

332
Q

What is the procedure in terms of company decision making of a company issuing debentures?

A

As it is usually the directors who negotiate borrowing of money in the company’s name, it is usually a board resolution that is required to authorise both the borrowing by the company and the grant of any security.

There may be problems over the ability of a director to vote and count in the quorum on the resolution to borrow and grant security, particularly if the directors have been asked to guarantee the loan personally.

The articles may prevent any director who has a personal interest from being involved in the decision to borrow. It may then be necessary to call a general meeting or to circulate written resolutions either to suspend any prohibition in the articles by ordinary resolution or to allow the directors to count in the quorum or vote or to change the articles by special resolution.

333
Q

What is roll over relief on incorporation of a business?

A

This is similar to rollover relief on replacement of business assets, in that the charge to CGT is postponed.

  1. When does it apply?

It applies, subject to certain conditions, when an individual sells their interest in an unincorporated business (ie a sole trader or partner) to a company.

The gain is rolled over into the shares which the seller receives as consideration for the sale of the assets to the company.

The CGT is payable when the individual disposes of the shares.

This relief encourages people to incorporate and expand their businesses, because if it were not for this relief, often taxpayers would not be able to find the money to pay CGT and invest in the new company.

  1. Conditions

a) The business must be transferred as a going concern, so after the disposal it must essentially be carried on as the same business but with a different owner.

b) The consideration must all be in shares issued by the company. If only part of the consideration was shares, for example, 25% of it, then only that percentage of the gain could be rolled over.

c) The business must be transferred with all of its assets, ignoring cash. If the taxpayer retains any of the assets, for example, if they keep the business premises, then the relief does not apply.

  1. How is the relief applied?

As with rollover relief on replacement of business assets, the gain is rolled over by notionally deducting it from the cost of acquisition of the new shares.

The taxpayer does not have to apply for this relief. HMRC applies it automatically unless the taxpayer chooses not to use it.

Note that the annual exemption cannot be used before the gain is rolled over, so the taxpayer loses the benefit of the annual exemption if they apply for rollover relief on incorporation.

334
Q

How do company accounts differ from other business accounts?

A

A company’s accounts are prepared on the same basis as the accounts of a sole trader or partnership.

Companies use the double entry system, and will prepare a trial balance leading to the preparation of annual final accounts.

However, company accounts will still look slightly different from the accounts of a sole trader or partnership, firstly because a company is a separate legal entity, and secondly because companies are more closely regulated by statue and accountancy rules than sole traders and partnerships.

It is important to remember that the accounts are still showing the same information that sole traders and partnerships show in their final accounts.

  1. Shares

The most obvious difference between unincorporated businesses and companies is that companies have shares.

  1. Issuing shares

Shareholders finance the company by paying money in return for shares. Whereas in unincorporated businesses, the introduction of money is recorded in a capital account for each contributor.

With companies, the payment of money in return for shares is recorded in a share capital account.

The share capital account records all of the shareholders’ contributions, rather than having an account for each shareholder, which could become complicated and unwieldy.

The section of the balance sheet which shows how much the shareholders have contributed is often headed ‘capital and reserves’, but is sometimes headed ‘Financed by’ or ‘Equity’.

Shares are either issued at par value – meaning that, for example, a shareholder will pay £1 for an ordinary £1 share – or the shareholder will pay a premium to reflect the fact that the company is profitable and its shares are now worth more than their £1 nominal value.

The premium will be shown separately, in the share premium account.

Companies may have more than one type of shares, for example, ordinary and preference shares. They will be shown separately on the balance sheet.

A company’s profit and loss account will look very similar to the accounts of a sole trader.

It will show trading income and expenses and the figures are calculated using the same accounting principles.

However, there are some small differences in the way expenses are shown on a company’s profit and loss account.

In a company, the owners (shareholders) and directors are a separate legal person from the company, so when the company pays its directors a salary, it is a company expense and will appear as an expense on the profit and loss account.

Contrast this with a partnership, where the partners might receive a salary, but the salary is just a name given to an arrangement where the partner receives some of the partnership’s profits before the profits are divided between the partners: a salary in this sense is just a name given to the way the profits are allocated to the partners.

335
Q

How are pre-emption rights disapplied if the company is a public company or private company with more than one class of share AND the authority to allot shares contained in s 551 ordinary resolution was in relation to a specific allotment?

A

The companies can disapply the pre-emption rights by special resolution, however there are some conditions.

The special resolution must be recommended by the directors of the company and before proposing a special resolution, the directors must make a written statement setting out:
a) The reasons for making the recommendation
b) The amount the purchaser will pay
c) The directors’ justification of that amount

The directors’ written statement must be circulated to the shareholders along with the notice of the general meeting or if the resolution is proposed as a written resolution, it must be sent out with the written resolution.

(Any company can use the procedure above if they want to but only those in this section are obliged to).

It is an offence to knowingly or recklessly authorise or permit the inclusion of any matter that is misleading, false or deceptive in a material particular in the directors’ written statement.

336
Q

What is a share transfer?

A

This is where the shareholder sells or gives shares to another shareholder or to a new shareholder.

The total number of shares does not change but the owner of the shares does.

There is no pre-emption right with share transfer and no prohibitions in the CA 2006.

337
Q

In a partnership, the firm may be liable for actions which were not actually authorised but which may have appeared to an outsider to be authorised.

What is the test for apparent authority?

A

a) The transaction is one which related to business of the kind carried on by the firm (OBJECTIVE)
b) The transaction is one for which a partner in such a firm would be usually expected to have authority (OBJECTIVE)
c) The other party to the transaction did not know that the partner did not have authority to act (SUBJECTIVE)
d) The other party deals with a person whom they known or believe to be a partner (SUBJECTIVE)

If a partner has acted with apparent authority, the firm will be liable to the third party under the contract.

In addition, the partner who has made the firm liable by virtue of their apparent authority is liable to indemnify their fellow partners for any liability or loss that they incur because they acted without actual authority.

338
Q

What is a tax return?

A

A taxpayer who has income that is liable to tax is required by law to notify HMRC of this within six months of the end of the relevant tax year. The penalty for default is a fine.

339
Q

What is the memorandum?

A

Under CA 2006, it consists of a statement that the subscribers wish to form a company and agree to become a shareholder and take at least one share each.

The memorandum must be signed by the subscribers.

It must be in a prescribed form and a copy of the correct memorandum will be on the CH website.

340
Q

What is a company seal?

A

Under CA 2006 a company seal can be used to execute a document. Under the MA, if a company seal is used to secure a document, it must also be signed by at least one person who is authorised to attest the signature in the presence of a witness who also signs. The company may disapply this MA and insert its own provisions.

341
Q

What claims can the liquidator/administrator bring on behalf of the company?

A

Avoidance of certain floating charges

Preferences

Transactions at an undervalue

Transactions defrauding creditors

Extortionate credit transactions

The liquidator/administrator can also bring a claim in misfeasance, wrongful and fraudulent trading against the director personally.

342
Q

What are the disadvantages of overdraft facilities?

A

An overdraft is known as an uncommitted facility which means that it will usually be payable on demand without notice. In practice the bank probably will not do this unless the business in question is in financial difficulties.

The business will have to pay a fee for the overdraft facility and the bank will also charge interest by reference to its base rate. It is therefore relatively expensive.

343
Q

What is the minimum and maximum number of shareholders for a private company?

A

Min = one
Max = none

344
Q

What is calculated at Step 3 of a corporation tax calculation?

A

In Step 3, you must add together the company’s income profits and capital gains. The resulting figure is the company’s total profits for the accounting period. By this point, certain reliefs have already been applied, but at Step 3 those reliefs which are applied against total profits must be considered.

Certain trading loss reliefs are deducted from total profits, namely carry- across/ carry- back relief, terminal carry- back relief and carry- forward relief.

Certain qualifying donations to charity are also deductible from total profits.

  1. Relief for trading loss

Sometimes, the calculation of trading income results in a loss rather than a profit. The company may be able to claim tax relief for this loss.

Where the company is eligible for more than one relief, the company may choose which to claim. If applying a particular relief does not absorb all of the company’s losses, the company can claim as much relief as is available under one relief and then claim relief for the balance of its loss under any other available relief. Companies cannot claim for the same loss twice.

Often companies claim relief under whichever provision is best for cash flow: out of the reliefs described below, carry- across/ carry- back relief and terminal carry- back relief may lead to a refund of tax already paid.

Carry- forward relief reduces the amount of corporation tax that the company will have to pay in the future.

1.2. Carry- across/ carry- back relief for trading losses

1.2.1. Carry across relief

Companies may carry across their trading loss for an accounting period and set it against total profits for the same accounting period.

1.2.2. Carry back relief

If, after setting the losses against total profits, there are still some losses remaining, the remaining losses can then be carried back and set against total profits from the accounting period(s) falling in the 12 months prior to the accounting period of the loss, on condition that the company was then carrying on the same trade.

If there are no profits at all for a particular accounting period, carry- back relief allows the company to set its losses for the current accounting period against total profits from the accounting period(s) falling in the 12 months prior to the accounting period of the loss, on condition that the company was then carrying on the same trade.

Applying these reliefs often leads to companies receiving a rebate of corporation tax they have already paid.

1.2.3. Deadline

Claim for the relief must be made within two years from the end of the accounting period in which the loss was incurred.

1.3. Terminal carry- back relief for trading losses

When a company ceases to trade, it can carry back any trading losses and set them against the company’s total profits from any accounting period(s) falling in the three years before the start of that final 12 months, taking later periods first.

1.3.1. Deadline

Claim for the relief must be made within two years from the end of the accounting period in which the loss was incurred.

1.4. Carry- forward relief for trading losses

A company may carry forward its trading loss for an accounting period and set it against subsequent profits in the next accounting period.

In order to benefit from this relief, the company must continue to trade and certain other conditions must be met.

Any remaining loss can be carried forward again and set against total profits in subsequent years.

1.4.1. Deadline

A claim for carry- forward relief must usually be made within two years of the end of the accounting period in which the company will apply the losses to reduce total profits.

1.5. Maximum claimed

The maximum amount that can be claimed under this allowance is £5 million, plus 50% of remaining total profits after deduction of the allowance.

If the company does not meet the conditions to set the loss against total profits, it can still carry forward the loss and instead set it against profits of the same trade.

345
Q

What are the two key functions of a liquidator?

A

a) Preserving and increasing assets of the company
b) Distributing those assets to creditors

346
Q

What is the statutory regulation on accounts?

A

Under the CA 2006, companies must keep ‘adequate accounting records’ and prepare accounts for the company for every financial year.

Directors must not approve accounts unless they are satisfied that they give a true and fair view of the assets, liabilities and financial position of the company. Directors can incur civil and criminal liability for untrue or misleading statements in the company’s accounts.

Companies must file their annual accounts with the Registrar of Companies and send a copy of their annual accounts and reports for each financial year to shareholders, debenture holders and anyone else who is entitled to receive notice of general meetings.

The final accounts of public companies must be put before the members in a general meeting.

Companies must usually have their accounts audited. Many small companies and dormant companies are exempt from this requirement, but shareholders holding 10% or more of the nominal issued share capital may require the company to have its accounts audited.

347
Q

When are shares considered to be allotted?

A

A company allots shares when a person acquires the unconditional right to be included in the company’s register of members in respect of the shares.

This will be when the shares have been transferred and paid for and the board has passed a resolution to register the transfer

348
Q

What are the conditions for a wrongful trading claim?

A

The company has gone into insolvent liquidation or insolvent administration,

Before commencement of the winding up of the company, the director knew or ought to have concluded there was no reasonable prospect that the company would avoid insolvent liquidation,

That person was a director of the company at the time.

349
Q

Which companies have to file a directors’ report and which companies are exempt?

A

Most companies must also prepare a director’s report to accompany the accounts.

Exemption for preparing a director’s report:

Private companies classed as a small company or micro-entity.

Small company = A small company is a company with a balance sheet of not more than £5.1 million, a turnover of not more than 0.2 million and no more than 50 employees in a particular financial year.

Micro-entity = A micro-entity is a company with a balance sheet total of not more than £632,000 and no more than 10 employees in a particular financial year.

350
Q

What other types of security other than floating charge or fixed charge are there?

A

Personal guarantee:

A director or partner gives a personal guarantee risks losing their personal assets if the business fails.

Pledge:

Arises where an asset is physically delivered by the debtor to the creditor to serve as security until the debtor has paid their debt.

The creditor has the right to sell the asset to settle the debt owed, provided they give sufficient notice (which may be agreed in advance by the parties).

Lien:

Gives a creditor right to physical possession of the debtor’s goods or assets until the debt is paid.

Retention of title:

On sale of goods, the buyer does not get full title to goods until they pay the full price to the seller.

If the buyer defaults then the goods are repossessed by the seller.

351
Q

What is the effect of crystallisation of a floating charge?

A

The chargor can no longer deal with the assets covered by the charge.

352
Q

In the context of an unfair prejudice claim, what would the court consider to be prejudicial conduct?

A

Diverting opportunities to a competing business in which the majority shareholder holds an interest.

Awarding excessive pay to directors.

Excluding a shareholder from management of the company where when the company was incorporated the shareholders negotiations led them to believe that they would participate in management.

Removal of auditor by shareholders on the grounds of divergence of opinion on accounting or audit procedures is unfairly prejudicial.

353
Q

What is a CVA?

A

It is a written agreement between the company and usually of its creditors to agree to wait a period of time to be paid back their debts or to accept part payment of debts or both.

354
Q

What is liquidation?

A

The process whereby the business stops trading, its assets are sold and the company ceases to exists aka winding up.

When liquidation proceedings begin a liquidator is appointed, the directors’ powers cease and the liquidator runs the company.

The liquidator may review the company’s past transactions and see if any of them can be challenged in order to obtain more money for the company’s creditors.

The liquidator will then distribute the assets of the company to the creditors in an order set down by statute and the company will be dissolved at CH within a few months.

355
Q

What is a statement of compliance?

A

This is completed once the IN01 is filled out.

The statement of compliance in Part 9 of the form needs to be filled out to confirm that all the requirements of CA 2006 have been complied with.

This is when CH will issue a certificate of incorporation and the company will exist as a legal entity.

356
Q

What should directors do after the board meeting to suggest a share buyback?

A

Either i) or ii)

i) Circulate written resolution

This will be in the form of a written resolution to authorise a buyback contract and has to be circulated with the contract.

Holders of the shares being bought are not eligible to vote.

ii) General meeting

Pass the OR to authorise the buyback contract.

The votes of the shareholders who are eligible to vote will not be counted if those votes make the difference between the resolution passing or not.

The buyback contract must be available at the GM.

357
Q

What are the penalties of failure to comply with VAT legislation?

A

Failure to comply with VAT legislation can lead to a range of criminal and civil penalties as well as being required to pay any unpaid tax with interest.

358
Q

What are a shareholder’s membership rights?

A

This includes their voting rights and their right to share in the company’s profits by receiving dividends.

359
Q

What should be included in a PA regarding work roles?

A

Set out the roles and any restrictions on roles e.g. the amount up to which they can enter into a contract.

If a partner breaches this they will be in breach of the partnership agreement.

Failure to comply with role may be a grand for exclusion.

360
Q

Who can enter into contracts on behalf of a company?

A

Contracts can be entered into by a company using the company seal or on behalf of the company by a person acting under its authority whether express or implied.

This is often a director but can be another employee if entering into contracts is part of their role.

361
Q

What can be included in a written resolution?

A

A statement of up to a thousand words can be included on the subject matter of the resolution.

362
Q

How are company secretaries appointed?

A

The first company secretary will usually be appointed on the IN01 form.

If appointed after incorporation, they will be appointed by board resolution.

The power to appoint a company secretary will be expressly stated in the company’s articles but there are no MA for this.

363
Q

What does a private company need to insert on the IN01 form if they choose to have a company secretary?

A

If they do choose to have one they need to add their name and service address on the form.

364
Q

Why might a company want to buyback shares?

A

There may be restrictions on a share transfer but not enough of the shareholders would support a special resolution to change the articles but there may be enough shareholders in favour of a share buyback which is authorised by an ordinary resolution.

365
Q

What is the effect of automatic dissolution of a partnership?

A

Unless the partners agree otherwise, all of the assets must be sold/partnership sold as a going concern and the outgoing partner has to receive their share.

An outgoing partner can insist that the business be sold, so remaining partners do not necessarily have the option to continue the business.

Needs to be stated explicitly in a partnership agreement that if a partner leaves, the remaining partners continue the partnership. This is known as partial dissolution.

366
Q

What is a chargeable asset?

A

‘Chargeable asset’ includes ‘all forms of property’, including debts, options and incorporeal property (a legal right in property having no physical existence, for example, a patent or a lease).

It does not include sterling, so disposal of cash in sterling is not chargeable to CGT.

367
Q

What adjustments may need to be made to the balance sheet?

A
  1. Outstanding expenses

Expenses will be entered in a trial balance when they are paid. The profit and loss account and balance sheet will show expenses which have been paid, for example, a utilities bill.

1.1. How are they included in the final accounts?

However, when preparing final accounts, additional, unpaid bills will also need to be included as an adjustment.

They will appear in the profit and loss account as an expense, and in the balance sheet as an additional current liability, under ‘accruals’.

  1. Prepayments

Prepayments are the opposite of outstanding expenses. A prepayment is a payment which the business has made in advance. Perhaps the business has bought something this year to use in the next financial year, or perhaps it is required to pay for something in advance, which it will not benefit from until the next financial year, for example, rent.

2.1. How are they included in the final accounts?

Prepayments are added to the balance sheet as current assets, labelled ‘Prepayments’. A prepayment will also reduce the value of the prepaid item in the expenses section of the profit and loss account.

  1. Work in progress (professional services charging)

Work which solicitors have carried out but for which they have not submitted a bill is called work in progress.

Law firms try to ensure that they issue bills as quickly as possible to reduce outstanding work in progress, but there will almost always be some work in progress from the current year carried over to the next financial year.

This is classed as an asset, because once it is billed to the client, it should result in the business receiving a cash payment. It will appear as additional income (known as profit costs) in the profit and loss account for the current year and as an additional current asset on the balance sheet.

  1. Closing stock

Unless they are in their first year of trading, businesses will begin their accounting year with some stock already in their warehouse, which they did not sell during the previous accounting period.

They will then purchase stock during the accounting period, and there will undoubtedly be stock remaining at the end of that accounting period. This stock is called closing stock.

Closing stock will be an additional item in the current assets section of the balance sheet, to show that the business will be starting the next accounting year with some stock already purchased.

  1. Bad and doubtful debts

A figure for debtors will appear in the balance sheet. Debtors are classed as a current asset, but not every debtor will pay.

They may be insolvent, or may refuse to pay, and the business may not think it is worth the time and money to pursue the debt.

5.1. Bad debts

The business should review its debts on a regular basis, and include those which it believes will never be paid (so- called ‘bad debts’) in the profit and loss as an expense.

The effect of ‘writing off’ the debt in this way is that the debt will no longer be shown on the balance sheet: it will be deducted from the debtors figure.

5.2. Doubtful debts

Doubtful debtors – those who may not pay – are entered in the expenses section of the profit and loss account as ‘provision for doubtful debts’ and are also subtracted from the debtors figure on the balance sheet.

  1. Depreciation

Most assets depreciate over time. After several years, they will be worth nothing, or almost nothing. For example, a car or piece of machinery will usually be worth more when it is new than when it is five or ten years old.

This is called depreciation, which is shown in the profit and loss account as an expense item which reduces net profit. Usually the directors will determine the amount of depreciation as a percentage of the value of the asset.

  1. Revaluation

Some assets do not tend to depreciate, and they will appear in the accounts at the acquisition value.

This figure will become less accurate the longer the asset is owned. Business owners may, in these cases, have assets revalued, and show the up- to- date value in the accounts. An example is property.

Property tends to increase in value over time, and a property valuation from ten years previously gives an inaccurate picture of business assets, as it will almost undoubtedly have increased substantially in value over the course of ten years.

  1. Disposing of assets

If a business disposes of a fixed asset, it will usually receive cash by way of consideration.

Therefore its total assets do not change, but its balance sheet will.

The fixed assets figure will reduce, and the cash figure will increase. Usually, however, assets do not sell for exactly the value at which they are recorded in the accounts. If the asset is sold for more than the figure recorded in the accounts (‘book value’), it will be shown in the profit and loss account as profit. If it sells for less than its book value, it will be recorded in the profit and loss account as a reduction of net profit.

368
Q

Who can demand a poll vote?

A

The chair of the meeting

The directors

Two or more people who have the right to vote in a resolution

A person/people not representing less than one tenth of the total voting rights of all the shareholders having the right to vote on the resolution.

369
Q

Is a company secretary necessary for a private company?

A

No

370
Q

How is taxable income worked out?

A

To work out taxable income, various reliefs and allowances are deducted from the income the individual earns.

The allowances and rates depend on the type of income, so when calculating income, it is important to separate out the income into the three categories so that the types of income can be treated differently.

371
Q

Comparing debt and equity finance, how does payment of income compare?

A

Equity finance

The company is only able to pay a dividend to its members if there is sufficient distributable profits.

Even if the company is sufficiently profitable, the directors usually have complete discretion as to whether a dividend should be paid (depending on the type of share).

Debt finance

The interest must be paid in accordance with the facility agreement, whether or not the company has the profits available.

If there are no profits, the company must use capital to make the interest payments.

If the company fails to make a payment of interest, the lender may be entitled to enforce the terms of the debenture by the appointment of a receiver or an administrator.

372
Q

What are the grounds for disqualification from directorship?

A

Court may disqualify a person from being a director for between two and fifteen years. Case law has established that the length of disqualification depends on the director’s behaviour in relation to the current offence and previous behaviour.

a) Conviction of an indictable offence
b) Persistent breaches of companies legislation
c) Fraud on a winding up
d) Summary conviction for failure to file a required notice or document
e) Being an unfit director of an insolvent company (most common)
f) Following an investigation and finding of unfitness
g) Fraudulent or wrongful trading
h) Breach of competition law

373
Q

What are chargeable receipts used in calculating trading profits?

A

Chargeable receipts is money received for the sale of goods and services.

The receipts must come from the business’s trade and be income rather than capital.

374
Q

Who pays the expenses of a shareholder written resolution?

A

The shareholders who requested the circulation of the resolution must pay the company’s expenses in complying with the request.

375
Q

How can the declaration under the director’s duty to declare interest in a proposed transaction or arrangement be made?

A

It can be made at a board meeting or by general notice in writing to the directors. Other forms are accepted, the point is more that a declaration is made rather than the form it takes.

376
Q

What is the benefit of selling a partnership as a going concern?

A

This is the business’s reputation and the value of its clients and contacts.

When a business is sold as a going concern, part of the purchase price will be for the business’s goodwill because the business is up and running and can continue as before, retaining its clients and reputation.

Goodwill is commonly valued as two years’ profit.

If the partnership assets are sold individually to be used elsewhere, then the goodwill will not be part of the equation.

It is a huge advantage to sell a business as a going concern because of goodwill.

Partner’s can increase the likelihood of the partnership being sold as a going concern when the partnership is to be completely dissolved by including terms in the partnership agreement that build in some time for partners to find buyer rather than having to sell the assets individually to raise money quickly.

377
Q

Is there a requirement of insolvency to deem a transaction a preference?

A

The company must have been insolvent at the time of the preference or have become insolvent as a result of giving the preference.

The liquidator or administrator will produce financial information such as company’s balance sheet and evidence of court proceeds against the company or correspondence to show that the company was insolvent at the time or became insolvent as a result of the transaction.

378
Q

Dependant on specific facts, what entries does a company need to make in its own register?

A

Amend register of members within two months

Amend PSC register if necessary

379
Q

What information is shown in a profit and loss account?

A

The profit and loss account shows how profitable the business is, that is, how successful it is in its day- to- day operations, whether those operations are trading or providing services.

The balance sheet shows what the business is worth, taking into account all of its assets and all of its liabilities.

All of the information in the profit and loss account and balance sheet are from the trial balance, but the information is presented in such a way that it is easy to understand the business’s profitability and worth from looking at the final accounts.

The profit and loss account tells us how profitable a business is, through a simple calculation:
Income – Expenses = Profit

If the business’s income exceeds its expenses, the business has made a profit.

If the business’s expenses exceed its income, the business has made a loss.

Only those items which are classified as income and expenses will appear on the profit and loss account.

It is important to identify income and expenses accurately, and not to confuse them with the business’s assets and liabilities (which will instead appear on the balance sheet). There is no place for assets and liabilities on the profit and loss account – knowing, for example, the value of the business’s fixed assets does not tell you anything about how successful the business is in its day- to- day trading.

On the profit and loss account, all of the income figures are grouped together, followed by the expenses. Then the expenses are deducted from total income to calculate profit. The items are set out vertically, so the layout of the profit and loss account is:

Income
LESS
Expenses
EQUALS
Profit

380
Q

Regarding the additional requirements to authorise buyback out of share capital, why is an auditor’s report required?

A

The statement of solvency must be annexed to an auditor’s report confirming that the auditors are not aware of anything to indicate that the director’s’ opinion is unreasonable.

381
Q

When is a bankruptcy order discharged?

A

It is discharged automatically after one year unless discharge is suspended.

This means that after the bankruptcy ends, the bankrupt is released from most of their debts.

They may still be subject to a bankruptcy restriction order (BRO) or a bankruptcy restriction undertaking (BRU).

The property which was vested in the trustee is not returned the bankrupt apart from the matrimonial home in some cases.

The trustee may not have realised all of the bankrupt’s assets by this point and the former bankrupt must still assist the trustee with this even once the bankruptcy order is discharged.

Sometimes automatic discharge is suspended if the bankrupt is uncooperative or dishonest during the bankruptcy. However, it is more likely that they will apply for a BRO or enter into a BRU.

382
Q

Which form must be used to file a change of registered office?

A

AP01

383
Q

What happens if any of the personal allowance for income tax is not used?

A

Any unused personal allowance (for those whose income is less than £12,570) cannot be carried forward for use in future years, unless the marriage allowance applies.

384
Q

What is a preference?

A

A preference is a transaction that, within the relevant time period, puts the other person in a better position than they would have been otherwise in the event that the company went into insolvent liquidation or administration.

385
Q

What kind of assets will a lender want a mortgage over?

A

A lender will seek to take a mortgage over high quality assets owned by the borrower e.g. land, buildings, machinery, aircraft, ships, shares in other companies.

386
Q

What do lenders need to pay income tax on?

A

A lender must pay income tax on interest received in relation to a loan.

If the lender/debenture-holder is a company, interest received is income, chargeable to corporation tax.

387
Q

What resolution is required to authorise a loan to directors?

A

Ordinary resolution

If the director receiving the loan is also a director of the company’s holding company, the holding company must also pass an ordinary resolution to authorise the loan.

388
Q

In a share buyback, what must the directors do at the board meeting?

A

Decide the method of finance

Resolve to approve the draft terms of purchase

Resolve to call a GM/propose a WR

Make contract/memorandum available to members. If WR used, memorandum must be circulated with WR. If GM used, memorandum must be at registered office for at least 15 days before GM and at the GM

389
Q

How many directors does a public company need to have?

A

2

390
Q

What does a company secretary do?

A

They are responsible for administrative tasks such as filing documents at CH and keeping board minutes.

391
Q

How can a debtor apply for their own bankruptcy petition?

A

They must apply online. An adjudicator will decide whether the make a bankruptcy order.

What is the ground for the application?

The ground is that they are not able to pay their debts.

As well as fees for the application, the debtor needs to pay a deposit in respect of the OR administration fees.

When must a decision about a bankruptcy order be made?

The adjudicator needs to decide whether to grant a bankruptcy order or deny one within 28 days of the application.

If the adjudicator requires more information, this can be extended to 42 days.

392
Q

When will the registered office be deemed to have changed?

A

The change of registered address changes when the registrar changes it but documents can still be sent to the other address up to 14 days after the change and they will be deemed to have been sent to the registered office.

393
Q

What happens to a general partnership in insolvency?

A

An insolvent partnership can be wound up as an unregistered company or may use the rescue procedures available to companies such an voluntary arrangement with creditors or an administration order of the court.

The individual partners may be made bankrupt if an obligation is enforced against against their personal assets and there is still not enough to meet the liabilities.

394
Q

Comparing debt and equity finance, how does this impact the existing capital structure?

A

If the company already has a high debt, it may not be possible for the company to obtain more finance other than shares. It is described as having a high gearing.

Gearing is the ratio of borrowing to shareholder funds.

High gearing means a greater burden of borrowings and the greater possibility of insolvency.

395
Q

During what period of time are trustees allowed to investigate transactions for the purpose of identifying transactions at an undervalue (personal insolvency)?

A

The trustee can investigate transactions during the five years prior to the presentation of the bankruptcy petition.

If the transaction was made in the the 0-2 year period in the 5 year investigate period, insolvency does not need to be proven. If the transaction was made in the 2-5 year period, then insolvency will need to be proven.

(The trustee does not have to show that the bankrupt was insolvent at the time of the transaction or as a result of the transaction unless the transaction was made more than two years before the petition)

396
Q

How is it agreed to hold a GM?

A

In order to call a GM, the board must hold a board meeting and agree to hold a GM by passing a board resolution. So over half the directors must agree to hold a GM.

Public companies MUST hold a general meeting every year but private companies are not obliged to.

397
Q

What are the three main types of loans?

A

Overdraft facility

Term loan

Credit facility

398
Q

Comparing debt and equity finance, what is the impact of existing restrictions on either?

A

Equity finance

The articles may restrict the company’s ability to borrow.

Debt finance

The terms of the existing facility agreements may restrict the taking of new loans or debt, or at last without the lenders consent.

399
Q

What is an IVA?

A

It is a binding agreement between unsecured creditors, setting out how much each creditor will receive from the bankruptcy in settlement of their debts.

This is created with the help of an insolvency practitioner who will effectively formulate the proposals and supervise their implementation if the creditors approve the process.

The insolvency practitioner is initially known as the debtor’s nominee and will only agree to act if there are sufficient assets to pay their fees.

Once the IVA is approved, they are renamed as supervisor of the arrangement.

400
Q

What is a claim in fraudulent trading?

A

A director will be liable if in the course of the company being wound up the company’s business has been carried out with the intent to defraud creditors of the company or creditors of any other person or any fraudulent purpose.

If this is found, the court may declare that anyone who is knowingly party to the carrying on of the business in such a manner is liable to make contributions to the company’s assets as the court thinks fit.

These claims are uncommon as the liquidator or administrator will need to show intention to defraud in order for the definition of fraudulent trading to be met and it is usually difficult find evidence of this.

Given that wrongful trading can arise on the same facts it is common for a liquidator/administrator to attempt a claim in both knowing that if the fraudulent trading claim fails, they may still succeed with the wrongful trading claim.

Any director who is found liable for fraudulent trading therefore also risks a criminal conviction.

401
Q

What do designated members do in an LLP?

A

The various legal and administrative matters are:
a) Signing and filing the annual accounts with the Registrar
b) Appointing, removing and remunerating the auditors
c) Filing the annual confirmation statement
d) Sending notices to the Registrar of Companies e.g. concerning a member leaving or joining the LLP
e) Winding up the LLP

They must carry out their functions with the core fiduciary obligation that every member owes to the LLP itself.

Designated members also owe a duty of reasonable care and skill to the LLP.

402
Q

How does the duty to avoid conflicts of interest and the duty to declare interest in a proposed transaction or arrangement intersect?

A

S 177 concerns a personal interest in a transaction or arrangement with the company.

S 175 is where the duty does not apply to a conflict of interest arising in relation to a transaction or arrangement with the company.

403
Q

When is a creditor entitled to bring a bankruptcy petition at court?

A

If they are owed £5,000 or more. But creditors owed less than £5,000 may join other creditors and petition together, provided that the total amount owed to all the petitioners is not less than £5,000. The creditors must pay a deposit to meet the costs of the trustee in bankruptcy and the court fee,

This must be liquidated.

The creditor must show that the debtor is unable to pay their debt or has little prospect of being able to pay their debt which is presumed if the creditor has used one of the methods e.g. statutory demand or court judgment.

404
Q

If MA 21 is included in the articles what does this mean?

A

This is important to note following the allotment of shares.

Under MA 21, all shares in a company must be fully paid so the buyer must pay for the shares when they receive them.

If there is no MA 21 in the articles, then the shares can be issued partly paid but the shareholder must pay the remainder when contractually obliged to do so or if the company is wound up.

405
Q

What should be included in a PA regarding restraint of trade?

A

This seeks to restrict the outgoing partners in their business dealings once they have left the partnership. This has to be an express clause in a PA, no default clause in PA 1890.N

It will only be enforceable if it protects a legitimate business interest e.g. business contacts and confidential information and it is no wider that is reasonable to protect that interest.

A restraint of trade clause encompasses non-compete, non-solicitation and non-dealing closes.

A non-solicitation clause does not stop customers or employees from approaching the former partner and entering into a contract but they cannot approach them.

Non-dealing clauses are more restrictive - the partner cannot enter into contracts with former clients or employees.

406
Q

When are members’ voluntary liquidation used?

A

Used when companies are dormant e.g. where there is a group of companies and some of the companies within the group are no longer used or when the directors in an owner-managed company want to retire and cease trading.

407
Q

How does business property relief impact IHT on death?

A

On death, there is a deemed transfer of value by the deceased of their entire estate immediately before death, and potentially exempt transfers made by the deceased in the seven years preceding their death become chargeable transfers as a result of their death.

Business property relief (‘BPR’) operates to reduce the value transferred by a transfer of value of relevant business property by a certain percentage.

This relief should be considered after applying any spouse/ civil partner or charity exemption.

  1. Reduction of 100%

A reduction of 100% of the value transferred is allowed for transfers of value where the value transferred is attributable to certain defined types of ‘relevant business property’ (meaning that there will be no charge to IHT in respect of those assets).

1.1. Relevant business property

a) a business or an interest in a business (including a partnership share);
b) company shares that are not listed on a recognised stock exchange (AIM is not included in the definition of stock exchange for these purposes). Only the value of the shares attributable to business/ trading activities is eligible for the relief.

  1. Reduction of 50%

A reduction of 50% of the value transferred is allowed for transfers of value where the value transferred is attributable to any other relevant business property.

2.1. Relevant business property

a) company shares that are listed on a recognised stock exchange if the transferor had voting control of the company immediately before the transfer;
b) land, buildings, machinery or plant owned by the transferor personally but used for business purposes by a partnership of which they are a member, or by a company (whether quoted or unquoted) of which they have voting control (the ability to exercise over 50% of the votes on all resolutions). In assessing whether or not a person has voting control, separate shareholdings of spouses or civil partners can, in certain circumstances, be taken as one, so that if the combined percentage of the votes gives the couple voting control then the test will be satisfied.

  1. Relevant time

a) must have been owned by the transferor for at least two years at the time of the
transfer; or
b) broadly, must be a replacement for relevant business property where the combined period of ownership is two years. This would include the situation where a sole trader or individual partner incorporated their business – the shares in the company that they received in return would be relevant business property.

3.1. Property inherited from a spouse

If property is inherited from a spouse or civil partner, the surviving spouse/ civil partner is deemed to have owned the property from the date it was originally acquired by the deceased spouse/ civil partner (but this rule does not apply to lifetime transfers between spouses/ civil partners).

3.2. Not whole of the interest

For BPR to apply, whether at the rate of 100% or 50%, the transfer does not need to be of
the transferor’s entire interest in the business or shareholding.

3.3. Sale of the business interest instead

Where a person has entered into a contract for sale of their business or company shares, their interest is taken to be in the proceeds of sale. So, because cash is not relevant business property, no relief will be available where there is a binding contract for sale (for example, when a shareholders’ agreement provides that if a shareholder dies, the shareholder’s PRs will sell to the remaining shareholders who will buy those shares).

The problem can be avoided by the use of an option to sell or purchase.

408
Q

What makes up a company’s constitution?

A

Company memorandum of association

Articles of association

Certificate of incorporation

Current statement of capital

Copies of court orders

Legislation altering the company constitution

Shareholder’s resolutions affecting the constitution

Certain agreements involving shareholders

409
Q

What are partners’ responsibilities of a partnership under under the PA 1890?

A

a) Partners must be completely open with each other regarding any relevant information regarding the partnership
b) Must account to the firm any private profits they have earned without the other partner’s consent from transactions concerning the partnership
c) Must not compete with the firm: this is classed as carrying on the business of the same nature as and competing with that of the firm. If the partner does so without the other partner’s consent, that partner must account for and pay over to the firm all profits made by them in that competing business,
d) Partners must bear a share of any loss made by the business, in accordance with the terms of the partnership agreement
e) Partners must indemnify fellow partners who have borne more than their share of liability or expense in connection with the partnership.

410
Q

Allotment of shares

If a company is a private company with one class of share incorporated under CA 2006, what needs to be considered with regards to the director’s authority to allot shares?

A

If the private company has one class of share both before and after the allotment, the directors have authority to allot shares without shareholder permission under s 550.

All that is needed is a board resolution.

411
Q

How are bankruptcy proceedings started?

A

Bankruptcy begins by a creditor presenting a petition at court or by the debtor applying for their own bankruptcy online.

412
Q

What needs to be submitted to Companies House in order to incorporate a company?

A

The applicant must complete a Companies House form IN01.

Submit a memorandum of association and companies articles of association (if they are not planning on using the Model Articles).

Submit applicable fee.

413
Q

How is a board resolution carried out?

A

Show of hands where each director has one vote.

If there is a chairperson they will have a casting vote. The chair will only need to use the casting vote if they are in favour of the resolution because if there is a tie, the resolution will not be passed.

It is possible to pass board meetings in writing or any other method that shows that the directors all agree in a situation where all directors agree unanimously.

414
Q

What is the process of compulsory liquidation?

A

Petition filed at court and served on the company - the start of the winding up. The company cannot dispose of any assets.

Petition is advertised in the Gazette.

Court hearing - if a winding up order is made, the Official Receiver appointed as liquidator and directors’ powers cease.

OR advertised order in the Gazette and notifies Registrar of Companies.

Liquidator investigates and reports to creditors.

Creditors may appoint alternative liquidator if the majority are in favour of that.

Liquidator collects in assets and realises if necessary and then distributes in the statutory order.

Final accounts sent to creditors and/or members.
Final return filed with court and Registrar of Companies.

Company dissolved after three months.

415
Q

Allotment of shares

If a company is a public company or a private company with more than one class of share, what needs to be considered with regards to the director’s authority to allot shares?

A

Under s 551, they must obtain the permission of the company’s shareholders before they can allot shares which requires an ordinary resolution or the articles of association could have included this from incorporation.

Content of the ordinary resolution:

Under s 551, the ordinary resolution must state the maximum number of shares the directors may allot and the date on which the authority will expire, which must not be more than five years from the date that the ordinary resolution was passed.

Expiry of the authority to allot:

It may be renewed by ordinary resolution of the company for a further period not exceeding five years.

The ordinary resolution must state the maximum number of shares the directors may allot and the date on which the authority will expire, which must not be more than five years from the date that the ordinary resolution was passed.

Inclusion in the articles:

The article must state the maximum number of shares that may be allotted under it and must specify the date on which it will expire which must not be more than five years from incorporation.

Filing at companies house:

The ordinary resolutions must be filed at CH.

416
Q

What is the deadline for issuing share certificates?

A

Two months of the transfer of shares being lodged with the company.

417
Q

What are the exceptions under the duty to declare interest in existing transaction or arrangement?

A

If the director is not aware of the interest or of the transaction or arrangement in question (and a director is treated as being aware of matters of which they ought reasonably be aware of)

If the interest cannot reasonably be regarded as likely to give rise to a conflict of interest

If or to the extent that the other directors are already aware of it (or ought reasonably be aware of it)

If it concerns the terms of the directors service contract

418
Q

What needs to be filed following a shareholder resolution to change articles?

A

A copy of the amended articles must be filed within 15 days of the amended articles taking effect.

It must also file a copy of the special resolution to amend the articles. This must be filed within 15 days of the special resolution taking effect.

419
Q

How do shareholders ratify a director’s breach of duty?

A

The shareholders can ratify a breach/potential breach of director’s duty by ordinary resolution. The directors can alternatively go ahead with an act on the condition that the shareholder’s or liquidator in case of insolvency, will ratify the breach.

Where the ordinary resolution is proposed as a written resolution and the director in question is also a shareholder, they cannot vote for the purposes of the written resolution. The same approach is taken for a general meeting.

420
Q

What clauses should be included in a PA?

A

Name of partnership

Place and nature of business

Commencement and duration

Work input

Roles

Decision making

Financial input

Drawings and salaries

Ownership of assets

Expulsion

Dissolution

Goodwill

Distribution of proceeds of sale
Restraint of trade

Dispute resolution

421
Q

What are the three categories of income?

A

(a) non- savings, non- dividend income (‘NSNDI’). This is essentially all sources of income apart from income from savings and income from dividends;
(b) savings income, which is interest from various sources, such as interest on money held in a bank account;
(c) dividend income.

422
Q

What must directors consider in a share buyback?

A

They must consider their director’s duties when buying back shares and whether it is a good decision for the company in the long run.

The board will need to make the decision with due skill, care and attention.

However, companies can justify a buyback on the basis that it is better for the company in the long run to buy out a vexatious shareholder, especially if that shareholder is also a director and their resignation was conditional on buying out their shares.

423
Q

What are the key accounting principles in the UK?

A

The Financial Reporting Standard, and it is applicable in the UK and Republic of Ireland. It is aimed at achieving consistency in the preparation of accounts and is sometimes referred to as ‘UK GAAP’ or ‘New UK GAAP’, to distinguish it from the similar systems of other jurisdictions.

  1. What does the Financial Reporting Standard do?

It specifies certain methods of calculation and sets out detail which must appear on the profit and loss account and balance sheet.

The International Accounting Standards Board (IASB) issues its own International Financial Reporting Standards (IFRS), which can be adopted in different jurisdictions. The IFRS have different rules from FR102, but cover the same points as FR102.

  1. UK format

The CA 2006 requires companies to prepare either ‘Companies Act Individual Accounts’ or ‘International Accounting Standards Individual Accounts’, which essentially comply with FRS 102 and the IFRS respectively.

Note that both require the preceding financial year’s figures to be shown alongside the equivalent figures for the current financial year, which enables comparisons to be made between financial years so that directors can spot changes and patterns emerging in the accounts.

2.1. The Companies Act Individual Accounts

The CA 2006 provides that Companies Act Individual Accounts must include a profit and loss account and a balance sheet, and must comply with any relevant regulations.

The main regulations which cover company accounts formats are the Large and Medium- sized Companies and Groups (Accounts and Reports) Regulations 2008 (SI 2008/ 410) and their equivalent for small companies, SI 2008/ 409 (together, ‘the Regulations’).

The Regulations contain templates showing the way the profit and loss account and balance sheets must be drawn up. Some variation is permitted, so you will encounter slightly different formats in practice.

2.2. Profit and loss account

The first point to note is that company profit and loss accounts separate income, expenses and profit generated by their trading/ operating activities from other items such as finance costs.

This enables them to see immediately how well they are performing in terms of trading activities.

The profit and loss account for a company does not itemise every expense; rather, it groups them together into categories such as ‘Administration Expenses’.

This stops the profit and loss account from being too complicated to interpret quickly. There will be notes accompanying the accounts, which give a more detailed breakdown and explanation of the figures. The names of the categories of expenses are dictated by accounting standards. You will also note some differences in terminology. An example is that sales are usually called ‘turnover’.

Finally, the company’s corporation tax liability will be shown at the end of the profit and loss account. Balance sheets may also be called statements of financial position.

424
Q

Can sole traders and general partnerships grant float charges?

A

They cannot grant floating charges. They can only grant fixed charges which must be registered at LR if they are over land.

LLPs can grant floating charges as well as fixed charges and the registration process is similar to that for companies.

425
Q

Comparing debt and equity finance, how does cost differ?

A

Equity finance

The cost of equity finance concerns the impact of calling dividends, capital appreciation and share buybacks.

The above returns are a cost to the original shareholders as their share of future dividends or capital growth is decreased due to the presence of new shareholders.

Dividends are not deductive for corporation tax purposes.

Debt finance

The cost of debt is the interest rate charged by the lender to the borrowing company.

The rate depends on commercial factors such as the security offered by the company, how much is borrowed, how long it is borrowed for, the company’s creditworthiness and general economic conditions.

The tax system favours debt financing because it is tax deductible.

Tax savings should be considered when assessing the cost of borrowing.

426
Q

What are the advantages of an IVA for the debtor?

A

They avoid the stigma of bankruptcy, bad publicity, avoid a public examination in court and the disabilities and disqualifications a bankrupt is subject to.

427
Q

How is CGT calculated in an LLP?

A

Treated for most purposes in the same way as an ordinary partnership as far as CGT is concerned.

However, when an LLP ceases to trade, it may be treated as a body corporate rather than a partnership for the purposes of capital gains.

428
Q

What form needs to be sent to CH to remove a PSC from the register?

A

Form PSC07

429
Q

What should the company consider initially when borrowing money and granting a security over a loan?

A

a) Ensure that the company has the power to borrow
b) Directors need to make sure that they have the power to grant security over the assets before entering into any security contracts on the company’s behalf, otherwise they are acting outside of their power and are in breach.

Company formed under CA 2006:

The company has unrestricted objects unless specifically restricted by the company’s articles of association.

The MA do not place restrictions on granting a security.

Company formed pre CA 2006:

The objects of the company were set out in its old style memorandum. If the company’s articles have not been updated, the memorandum must be checked to see that there is no restriction on the company granting security.

Restrictions on granting security (both types of company):

The company’s shareholders must first pass a special resolution to amend the articles.

Director authority:

The directors also need to check whether they have the necessary authority to enter into a security contract on behalf of the company.

For a company with MA, the directors’ authority is contained in MA 3.

If the company has amended or bespoke articles, they must be checked for any restrictions on the directions acting in the situation.

430
Q

What are the conditions for a public company to be considered ‘public’?

A

The constitution must state that it is a public company

The words public company or plc must be in the company’s name

The company’s owners must invest a specified minimum amount of money for use by the company - £50,000. Each allotted share must also be paid up to at least a quarter of its nominal value plus the whole of any premium on it.

431
Q

Can public companies pass shareholder resolutions through the written resolution procedure?

A

This is only allowed for private companies NOT public.

432
Q

What additional requirements are needed for a company to be registered as a public company?

A

The constitution must state that it is a public company

The words public limited company/plc need to be included in the company name

The company’s owners must invest a specified minimum (£50,000) and each allotted share must be paid up to at least a quarter of its nominal share

433
Q

How are pre-emption rights disapplied if the company is a public company or private company with more than one class of share AND an ordinary resolution under s 551 gave the directors a general authority to allot ?

A

The company can remove the pre-emption rights just by passing a special resolution.

Disapplication will last as long as the director’s authority under s 551.

434
Q

What is a transmission of shares?

A

This is an automatic process where if a shareholder dies, their shares automatically pass to their personal representatives or if a shareholder is made bankrupt their shares automatically vest in their trustee in bankruptcy.

435
Q

What administrative requirements are there when a member leaves an LLP?

A

LLP need to file form LL TM01 for an individual member or LL TM02 for a corporate member at Companies House within 14 days.

436
Q

How does a company notify Companies House that they would like to use a SAIL address instead of their company’s register to store their statutory records?

A

Send from AD02 to CH

437
Q

How many designated members are required in LLP?

A

Need to have two designated members (are in charge of filing documents at Companies House).

438
Q

What is a defence to a wrongful trading claim?

A

If they took every step with a view to minimising the potential loss to the company’s creditors as they ought to have taken.

439
Q

What should be included in a PA regarding dispute resolution?

A

Useful to include a provision that indicates how the event of a dispute should be handled between partners i.e. arbitration or another form of ADR rather than formal court proceedings.

440
Q

What are the types of receivership?

A

a) Law of Property Act receivers

This is when a receiver is appointed by a fixed charge holder.

The power to appoint this type of receiver is usually in the charge document.

The aim of the LPA receiver is to sell the charged property so that the creditor an be repaid.

If the sum realised is not enough to pay what the creditor is owed, the creditor will become an unsecured creditor for the remainder.

If the sale realises more than the amount owed to the creditor, the surplus will be returned to the company and will be available to unsecured creditors.

LPA receivers do not need to be licensed insolvency practitioners.

b) Administrative receivers

These are appointed by floating charge holders when the floating charge is over the whole of the company’s undertaking.

They are only used for floating charges created before 15 September 2003. Charges created on or after the 15 September 2003 use the administration route instead.

441
Q

Where is the PSC kept?

A

Registered address or on central register at Companies House.

442
Q

What are the administrators powers and duties?

A

a) Removing and appointing directors,
b) Paying creditors, but only with the court’s permission if the payment is to an unsecured creditor
c) Calling a meeting of creditors or shareholders
d) Dealing with property that is subject to a floating charge
e) Dealing with property that s subject to a fixed charge with permission of the court
f) Investigating and applying to have the company’s past transactions set aside or challenged
g) Commencing fraudulent or wrongful trading proceedings against directors
h) Power to do anything necessary or expedient for the management of affairs, business or property of the company.

443
Q

What should directors do if they have an interest in a transaction?

A

If a director has an interest in a transaction they should declare the nature and extent of this to the board.

444
Q

What is carry back of terminal trading loss relief?

A

Any loss incurred by the taxpayer in the final 12 months of trading can be carried across and set against trading profit in the three years preceding the year of loss, starting with the year preceding the year of the loss and moving back year by year until the loss is fully absorbed or the three-year limit is reached whichever is first.

No cap on the amount that can be relieved under this provision.

(There can be a loss and a profit in the same year as the rules allow other sours of income which are connected to the trade but not actually profits of the trade to be treated as trading profits for the purposes of this relief).

The use of this relief may mean that the tax payer gets a tax rebate, because their trading profits from previous years will be reduced and it is likely that their tax liability will reduce as a consequence.

It is important to note that this relief can only be applies to trading income.

A claim for carry-back of terminal trading loss must be made no more than four years after the end of the tax year to which the claim relates

445
Q

What is the process of creditors voluntary liquidation?

A

Directors agree by a majority that the company is insolvent and needs to be placed into liquidation.

Members pass special resolution to liquidate the company and may consider a liquidator. Directors powers effectively cease and this marks the start of the winding up.

Petition advertised in the Gazette.

Directors must before the end of the 7 days beginning with the day after the day the company passes a resolution for CVL: a) make out a statement in the prescribed form as to the affairs of the company, send the statement to the company’s creditors, c) seek nomination from the company’s creators for a person to be liquidator. Creditors considering the appointment of a liquidator must seek approval by the new deeded consent procedure.

Once the resolution to wind up the company has been passed by the shareholders, the liquidator must file a copy at Companies House and it must be advertised in the Gazette.

Liquidator investigates and reports to the creditors.

Creditors may appoint alternative liquidator if the majority are in favour and a resolution is passed to that effect.

Liquidator collections in assets and realises if necessary and then distributes in the statutory order.

Final accounts sent to creditors and members.

Final return filed with Registrar of Companies.

Company dissolved after three months.

446
Q

What is the deadline for filing accounts for private companies?

A

9 months from end of accounting reference period for a private company.

447
Q

What are the fiduciary duties and responsibilities of members of an LLP?

A

Members also owe fiduciary duties to the LLP as its agents.

This includes a duty of good faith, a duty to account for money received on behalf of the LLP and duty to the other members to render true accounts and full information on matters concerning the LLP.

448
Q

What comprises a liquidator’s duty to preserve and increase assets of the company?

A

Liquidators and administrators are under a duty to maximise the assets available to creditors.

They have the power to investigate company affairs and to investigate the directors’ actions before liquidation or administration.

The liquidator/administrator can bring a number of claims on behalf of the company. The money awarded to the company will be paid to the creditors.

449
Q

What is a poll vote?

A

This is where the shareholders vote in a general meeting own the basis of one vote for each share that they own.

The more shares a shareholder has the more power they have when voting at a GM.

This can be seen as a way of ensuring a fairer outcome for those shareholders who have invested more money in the company.

450
Q

What are the advantages of a public company over a private company?

A

Can raise money by offering shares to the public

Public companies can apply to join the stock market - can raise large sums of money enabling investors to buy the company’s shares quickly and easily

451
Q

In step 2 of a CGT calculation, how is the gain calculated?

A

Take the initial consideration for sale and subtract any of the following expenditure.

  1. Initial expenditure

The cost price of the asset (or its market value or probate value if the asset was given as a gift or the taxpayer inherited it);

Any incidental costs of acquisition, for example conveyancing fees in relation to the purchase of a property, or other legal fees, valuation fees and stamp duty; and

Any expenditure wholly and exclusively incurred in providing the asset, for example, the cost of building a property.

  1. Subsequent expenditure

Expenditure wholly and exclusively incurred in establishing, preserving or defending title to the asset. An example of this would be legal fees incurred to resolve a dispute regarding the title to the property, for example a boundary dispute; and

Expenditure wholly and exclusively incurred to enhance the value of the asset, which is reflected in the value of the asset at the time of disposal. An example of this would be the cost of building an extension to a house.

The cost of normal maintenance, repairs and insurance is not deductible.

  1. Incidental costs of disposal

These include legal fees for the sale and the estate agent’s fees or commission.

  1. Losses

Sometimes a disposal can result in capital loss, not a gain.

452
Q

What happens if the debtor does not comply with the IVA?

A

If the debtor does not comply with the arrangement or if it is discovered that the creditors accepted the debtor’s proposals based on false and misleading information, the supervisor/any of the creditors who are party to the IVA, can petition for bankruptcy.

This may happen if the debtor has made a transaction at an undervalue or a preference just before entering into the IVE which has not been disclosed.

Only the trustees can apply to court to set aside these transactions, the supervisor is not able to.

453
Q

Who can enter into deeds on behalf of a company?

A

A company can execute a deed by:
a) Affixing its seal
b) By signatures of:
I) Two authorised signatories (a director or company secretary) or
ii) A director of the company in the presence of a witness who attests the signature

The document must be delivered as a deed i.e. it must be clear on the face of it that it is a deed.

Execution by company seal is more complicated. MA 49 states that if a company wishes to use its seal to execute a document, the documents must also be signed by at least one authorised person (director, company secretary or other authorised person) in the presence of a witness who attests the signature).

454
Q

Which factors count against a director when they may be disqualified from office?

A

a) Using money meant for VAT, PAYE or national insurance contributions as the company’s working capital
b) Paying excessive directors’ remuneration
c) Recklessly trading while insolvent

455
Q

When can a fraudulent trading claim be brought?

A

Such claims can only be brought when a company is in insolvent liquidation or insolvent administration.

456
Q

How is a taxpayer’s total income worked out?

A

This is step 1 in an income tax calculation.

To work out the taxpayer’s total income, you must find out what sources of income the taxpayer has, calculate the income arising under each source and then add all of the sums together. The resulting figure constitutes total income.

457
Q

Which debts still have to be paid during the moratorium?

A

Employee’s wages or salary under a contract of employment, the monitor’s remuneration or expenses, goods or services supplied during the moratorium.

458
Q

What sources of income (non-comprehensive) do income taxes NOT have to paid on?

A

a) interest on damages for personal injuries or death,
b) interest on savings certificates,
c) certain state benefits,
d) premium bond winnings and income from investment in an individual savings account.

459
Q

How do shareholders make money from their investments?

A

Value of the shares increased:

If a company begins trading with 100,000 shares of £1 each, then makes profit in its first year so that the company is worth £150,000, each share is now worth £1.50, instead of £1.

Receipt of dividends:

A company can pay dividends if it has profits available for the purpose.

A company’s available profits are its accumulated, realised profits less its accumulated, realised losses.

If in a particular financial year, the company has not made any profit, it can use profits from previous years to pay a dividend if it wishes.

Under MA 30, it is the directors who decide whether or not to recommend that a dividend be paid and how much it should be.

The shareholders must then pass an ordinary resolution for this to be approved.

460
Q

What information do companies need to keep about directors?

A

Companies must keep a register of directors containing info such as the director’s birth date and address and the company’s registered office.

This must be kept available for inspection without charge by shareholders or by other individuals following payment of a fee at the company’s registered office.

The company can elect to keep it on the central register at CH instead.

461
Q

What taxes do sole traders have to pay?

A

Sole traders are almost always subject to income tax, and may also have to pay capital gains tax (‘CGT’) or inheritance tax (‘IHT’), depending on their circumstances and the nature of their trade.

If they make chargeable supplies exceeding £90,000 in any period of 12 months, sole traders will also be required to register for VAT.

Calculating the amount of tax a sole trader must pay requires an understanding of how trading profit is assessed.

462
Q

How many members are required in LLP?

A

Must have atleast two members on incorporation.

If at any time the number of members reduces to one and this carries on for more than six months, that person is jointly and severally liable for any of the LLPs debts incurred during the period from the sixth-month point onwards.

463
Q

What are the advantages of a floating charge?

A

It allows the chargor to deal with the secured assets on a day to day basis.

As a form of security that attaches to assets that are unsuited for a fixed charge or a mortgage, it allows the chargor to maximise the amount they are able to borrow.

A floating charge can be taken over the whole of a company/LLP business.

464
Q

Under PA 1890, how are the proceeds of sale of a partnership applied?

A

a) Creditors of the firm must be paid in full. If there is a shortfall, the partners must pay the balance from their private assets. They will share the losses in accordance with their partnership agreement.
b) Partners who have lent money to the firm must be repaid the amount of the loan that is outstanding, including interest.
c) Partners must be paid the share of the partnership’s capital to which they are entitled.
d) Any surplus is shared between the partners in accordance with the terms of their partnership agreement.

465
Q

How is CH notified of the movement of company records to the SAIL address?

A

Form AD03 sent to CH

466
Q

What kind of express covenants will the lender require from the borrower?

A

a) Limitation of dividends

Companies must ensure that dividends and other distributions to shareholders do not exceed a specified percentage of the net profits.

b) Minimum capital requirements

The business must ensure that current assets exceed current liabilities by a specified amount of money or a specified percentage.

c) No disposal of assets or change of business

The business must not dispose of assets without the lender’s consent or change the scope and nature of the business.

d) No further security over the assets

The business must not create any further security over the whole or part of the undertaking without the lender’s consent

e) Provision of information on the business

e.g. annual accounts

Whilst the lender has the bargaining power on their side, they need to be careful not to put too many covenants in place to avoid the risk of being a shadow director to the borrowing company.

467
Q

When are written resolutions passed?

A

They are passed when the required majority of eligible members have signified agreement to the resolution.

468
Q

What happens if there is no PA provision that addresses the issue of an outgoing partner’s share of the partnership?

A

If the partnership agreement does not address the issue of payment for the outgoing partner’s share, the outgoing partner is entitled to either interest at a rate of 5% per annum on the value of their partnership share until they receive their share from other partners or such sum as the court may order representing the shares of the profits made which is attributable to the use of their share.

469
Q

What are the test for insolvency?

A

A company is insolvent when:
a) A creditor has served a statutory demand for an outstanding sum of £750 or more, and the company does not pay or come to an arrangement with the creditor within 21 days of service of the statutory demand,

b) A creditor has obtained judgment against the company and has tried to enforce the debt against the company but the debt still has not been paid in full or at all,

c) Cashflow test

It can be proved to the court that the company is unable to pay its debts as they fall due,

d) The balance sheet test

It can be proved to the court that the company’s liabilities exceed its assts.

470
Q

What is the purpose of a chair person?

A

They run the meeting and have the casting vote at board meetings. If a vote is a tie, they can use their casting vote to ensure that the resolution is passed by a simple majority.

If the board of directors does have a chair, this person will also chair general meetings if they are present and willing to do so.

471
Q

Can a company make a share buyback out of capital instead of distributable profits?

A

This can be considered if the company does not have enough distributable profits. They must have excused their distributable profits before considering this.

Only private companies that do not have a restriction on the use of capital for buyback in the articles are able to do this.

Public companies are not allowed to do this.

The same conditions as for buyback out of distributable profit must be met with additional conditions.

472
Q

Can an LLP charge its property?

A

LLPs can issue debentures and both fixed and floating charges.

473
Q

Comparing debt and equity finance, what is the capital value of the investment?

A

Equity finance

The capital value of a private company’s shares may increase or decrease depending on the company’s success.

Many shareholders invest in shares hoping they will increase in value rather than by way of dividend.

Debt finance

The capital value aka the value of the loan stays constant as there is no possibility of capital appreciation or depreciation.

This is not the purpose of the investment (which is receipt of income from regular interest payments).

474
Q

What forms need to be sent to CH with regard to removal of a director?

A

Forms TM01 (for human directors) and TM02 (corporate directors) are used to notify CH of the resignation or removal from office of a director.

This must be filed within 14 days of the resignation or removal from office.

475
Q

What is the capital allowance used in calculating trading profits?

A

A capital allowance allows businesses to deduction a proportion of the costs of most capital items from chargeable receipts. This is done to encourage businesses to invest in key plant and machinery that will make them grow.

The main types are plant and machinery. This includes whatever apparatus business props use to carry on their business i.e. goods and chattels which are kept for permanent use in their business but not stock in trade.

Includes manufacturing equipment, tools, computers and office equipment.

Each financial year, a business is entitled to a writing down allowance to account for the reduction in value over time.

The writing down allowance is 18% of the total value of the business’s plant and machinery, valued at the start of the financial year.

Each financial year, the business’s plant and machinery will be valued and 18% of its total value will be deducted from chargeable receipts when calculating trading profits for that accounting period.

All of the plant and machinery is pooled and the written down value is usually calculated each year on the basis of the value of the whole pool. If an asset is sold, the proceeds of sale are dedicated from the value of the whole pool not the individual item.

In addition to the written down allowance, the businesses are entitled to an annual investment allowance.

This allows businesses to deduct the whole cost of plant and machinery purchased in that particular accounting period from the chargeable receipts.

Due to be £1,000,000 until 31 March 2024.

So 1 million of the fresh qualifying expenditure on plant and machinery incurred in an accounting period is wholly deductible.

A group of companies only has one annual investment allowance but it can be allocated within the group as is seen fit.

This applies to all businesses and new, second hand and refurbished plant and machinery.

476
Q

What is the quorum for board meeting?

A

A quorum of two directors must be present at all times during a board meeting.

477
Q

In bankruptcy proceedings, what happens if someone else has a legal or equitable interest in the house or a right of occupation?

A

The bankrupt cannot be evicted straight away and the trustee needs a court order to sell the house.

The court will weigh up all the relevant circumstances including the interests of the creditor, conduct and needs of current or former spouse or civil partner and the needs of the children before agreeing to a court order.

After one year of bankruptcy, the creditors’ interests outweigh those of anyone living in the house unless the circumstances are exceptional and the trustee is likely to be able to obtain an order for sale.

478
Q

What happens if the shareholders do not take up the offer to buy pre-emption rights?

A

The directors can offer the shares to other buyers.

479
Q

How long must statutory records (record of board meetings and general meetings) be kept at either the SAIL, company registered office or central register of the CH?

A

10 years

480
Q

In a general partnership, who can a claimant sue to enforce their liabilities?

A

a) Partner(s) with whom they made the contact because of privity of contract
b) Anyone who was partner at the time when the debt was incurred. That partner can then claim an indemnity from their partners is that the partners share the liability between them (under the PA 1890 or PA)
c) Sue the firm in the firm’s name. Means that anyone who was a partner at the time when the debt was incurred is jointly liable to satisfy the judgment.

The best option is to sue all of the partners of the firm as the judgment can be enforced against the partnership assets and against the partners’ personal assets.

The court may order another partner to pay a just and equitable contribution to the debt so if the claimant does sue one party only, the others can be made to contribute some or all of the judgment amount.

481
Q

In a share buyback, what must the directors do before the board meeting?

A

Check there is no limit in the company’s articles on its power to buy back the shares

Prepare accounts to ascertain available profits and whether the shares are fully paid

482
Q

In bankruptcy proceedings, what happens to the debtor’s physical assets?

A

The bankrupt can keep some assets which are needed for day-to-day living, such as items needed for work and everyday household items such as clothing or furniture.

But if any of these items are of high value the trustee can sell them and replace them with a cheaper source.

483
Q

Who does the CVA not affect?

A

The rights of secured and preferential creditors, unless they argued to it.

484
Q

What can income tax be charged on?

A

Salaries, fees and other benefits paid to employees are deductible expenditure when calculating trading profits.

Businesses’ trading receipts will frequently be reduced to nil or almost nil once salaries have been deducted, because they are one of the most significant types of expenditure for any business.

Whilst this will result in the business ultimately paying less tax, the recipients of the salaries or fees will instead pay tax on the income.

Employees, including directors, pay income tax on employment income, pensions income and Social Security income.

Here, we will focus on employment income.

  1. Employment income

Employment income includes ‘earnings’, which means all benefits received by the employee (or director) which derive from their office or employment as a reward for their services.

This is whether they are paid by the employer or by a third party.

1.2. Taxable employment income

a) Salaries
b) Non-cash benefits (company car, private medical insurance - value of this will be the monetary value, share schemes)
c) Bonuses and tips
d) Lump sums received at the beginning and end of a person’s employment
e) Compensation for unfair dismissal and damages for wrongful dismissal (although the dismissed employee receives the first £30,000 of the compensation or damages free of tax).
f) Rent free or low rent accommodation unless necessary for the employee to live on the premises in order to perform duties or the accommodation is provided so that they can perform their duties better.

1.3. Non taxable employment income

a) Personal gifts
b) Special rate loans from employer when the total amount outstanding on any loan to that employee does not exceed £10,000 at any time in the tax year.
c) Employer’s pension contributions if they are paying into an HMRC approved scheme

Note that employees also have to pay National Insurance, which is a tax on earnings and
profits of the self-employed.

Individuals pay National Insurance to qualify for certain benefits and the state pension.

There are different types of National Insurance called classes, and which class an individual is in depends on their employment status and how much they earn.

It is deducted from the employee’s salary by employers. HMRC produces tables so that employers can check how much an employee should pay.

  1. Deductible expenditure

An employee can deduct from their income expenditure which is incurred wholly, exclusively and necessarily in the performance of their duties.

This is a stricter test than that for deductible expenditure when calculating trading income.

The requirement for necessity means that it must be shown that the employee could not perform their duties without the expenditure in question.

The strictness of the test means that few items of expenditure will satisfy it, so it is modified in relation to travelling expenses and pension contributions.

The employee’s contributions to an occupational pension scheme or personal pension scheme are deductible and travelling expenses need not be wholly and exclusively incurred in the performance of their duties. This means that travelling from one place of work to another would generally be deductible, but travelling to work would not, because an employee only commences performing their duties when they arrive at work.

The effect of the rules on deductible expenditure is that any of the employee’s expenses which are deductible will be subtracted from total income so that less of the employee’s income is charged to tax. The employee is not permitted to deduct any expenditure which has been reimbursed by their employer.

485
Q

What may the court decide the after the second stage hearing of a derivative claim?

A

The court may decide to grant permission to the shareholder to continue the claim on terms that the court thinks fit or adjourn the proceedings.

Only at this stage will the court give directions for the trial of the issues raised in the claim.

486
Q

What rebuttable presumption is made in relation to a preference (personal insolvency)?

A

There is a rebuttable presumption of the intention to prefer if the preference is in favour of an associate (essentially a close relative or business associate).

487
Q

Why is it important to identify a person with 75% or more of the company’s shares/voting rights?

A

Can pass special resolutions and ordinary resolutions on their own. Can block both ordinary and special resolutions.

488
Q

Which kind of resolution is required to change the registered office of a company?

A

Board resolution

489
Q

Are company secretaries remunerated?

A

The board of directors will decide the contractual terms upon which the company secretary will hold office.

In smaller private companies, there will be no remuneration

490
Q

What authority do members of an LLP have?

A

Members of an LLP are its agents.

Limitations can be placed on a partner’s actual authority the partnership agreement or orally.

LLPs may still be liable to a third party even if a partner has acted without actual authority because of apparent authority.

491
Q

What are participating shares?

A

They have a further right to receive profits or assets in addition to their other preference share rights.

E.g. if the ordinary shareholders receive a dividend over a specified amount, the participating preference shareholders have the right to an additional payment over and above their usual entitlement.

492
Q

Once the savings income has been worked out from taxable income for step 4 of income tax, what needs to be done with the savings income figure?

A

To tax savings income, the PSA must first be deducted from the savings income figure. This will
give the amount of savings income which will be taxed at the savings rates set out below.

Savings income less PSA (taxed at 0%) = ‘remaining taxable savings income’

  1. Personal savings allowance (‘PSA’)

1.1. The PSA can be set against savings income, so that up to the first £1,000 of savings income will be tax free.

1.2. The amount of the PSA to which a taxpayer is entitled depends on whether the taxpayer is a basic or higher rate taxpayer. This is calculated by reference to the taxpayer’s taxable income, as calculated at Step 3. (Allowances will be made for allowable pension contributions and gift aid.)

1.3. Additional rate taxpayers do not receive a PSA.

1.4. Basic rate taxpayer:

1.4.1. Taxable income band = £0– £37,700
Allowance = £1,000 tax free

1.5. Higher rate taxpayer:

1.5.1. Taxable income band = £37,701– £125,140
Allowance = £500 tax free

1.6. Additional rate taxpayer:

1.6.1. Taxable income band = over £125,140
Allowance = No allowance

1.7. Remaining taxable savings income:

1.7.1. The remaining taxable savings income will then be taxed at the savings starting, basic, higher and additional rates. For the tax year 2023/24 the rates are as follows:

Starting rate for savings of 0% = 0 - 5000
Savings basic rate of 20% = 5,001 - 37,700
Savings higher rate of 40% = 37,701 - 125,140
Savings additional rate of 45% = over 125,401

Need to add the PSA to their NSNDI.

Let’s say this gives a figure of £36,000 and the individual has remaining taxable savings income of £3,000. The individual will pay tax on £1,700 of the remaining taxable savings income at 20%, and will pay £1,300 at 40% because this portion of the remaining taxable savings income is above the basic rate threshold of £37,700.

493
Q

What must the s 303 request state?

A

The request must state the general nature that needs to be dealt with at the meeting.

494
Q

Avoidance of certain floating charges

Who is a person connected with the company?

A

a) Director or shadow director of the insolvent company

b) Someone who is a close relative or business associate of a director or a shadow director

c) An associate of the company aka a company in the same group as the company or which is controlled by a director of the insolvent company

495
Q

How is retained profit presented in the business accounts of a company?

A

The balance of the net profit, after tax and dividends, is retained in the business. These retained profits are generally known as the profit and loss reserve. Because the profit and loss reserve consists of profit the business has made while trading, it does not form part of the capital of the company.

This means that it can be distributed to shareholders if the directors decide to recommend a dividend.

Contrast this with share capital, which forms part of the capital of the company because it derives from the money shareholders have paid for shares.

It is not usually available to shareholders because of the principle of maintenance of capital.

496
Q

What is a derivative claim?

A

This is a claim instigated by a shareholder for a wrong done to a company which has arisen from an act or omission of a director.

Allows the shareholder to instigate an action on behalf of the company because the board is refusing to do so.

497
Q

Which grounds for partnership dissolution can be disapplied by the PA?

A

Dissolution of partnership on expiry of fixed term, death/bankruptcy of partners and notice + charge of share can be disapplied in the PA.

498
Q

After having investigated the bankrupt’s assets, what actions can the trustee take to preserve and increase the bankrupt’s assets?

A

a) Disclaim onerous property
b) Apply to set aside transactions at an undervalue
c) Apply to set aside preferences
d) Apply to set aside transactions defrauding creditors
e) Avoid extortionate credit transactions

499
Q

What kind of specific provisions could be useful when a partner wishes to leave the partnership?

A

Examples:
a) Whether other partners must buy the outgoing partner’s share or whether they just have the option to do so
b) How the partnership share of the outgoing partner should be valued and when this should be paid
c) Useful to include a provision stating that the outgoing partner will be paid in instalments so it is more likely that the partners can afford to buy them out
d) Provision containing an indemnity in favour of the outgoing partner if their liabilities were taken into account when their partnership share was valued.

500
Q

Other than payment of over £200 for compensation for directors for loss of office, which other payments to individuals require authorisation by ordinary resolution?

A

Also need an ordinary resolution for payments to:
a) Past directors
b) Payments to a person connected with a director
c) Payments to any person at the direction of or for the benefit of a director or a person connected with a director

‘Payment’ also catches a situation where the director is also selling their shares in the company and the price is in excess of the price other shareholders could have obtained.

501
Q

What are final accounts?

A

This is when trial balance is translated into final accounts.

Businesses need and want to know how well their business is performing financially, and what it is worth.

The final accounts, which comprise the profit and loss account and the balance sheet, give them this information.

502
Q

How do partnership accounts differ from other business accounts?

A

The fact that partnerships have two or more owners means that partnership accounts will need to show more information than the accounts of sole traders.

Partnership capital is contributed by partners and the firm’s profit is owed to those partners.

The partnership will need to keep separate records for each partner, showing how much capital they contributed, how much profit is owed to them, and how much they have withdrawn during the year.

Each partner will also have an account in their own name, called a capital account, which shows the amount contributed by each partner.

  1. Share of profits

Partners will share profits and losses in accordance with their partnership agreement (or, if there is no agreement, in equal shares (Partnership Act (PA) 1890 or the LLP Regulations 2001).

Sometimes one or more partners may receive a salary to reflect the fact that they carry out more work for the business, or they may receive interest on capital contributions.

Any remaining profit will then be divided between partners in the proportions set out in the partnership agreement, PA 1890 or LLP Regulations 2001.

It is important to remember that salary and interest in this context are not separate from profit; partners are not employees and so ‘salary’ in this context just means that some partners are entitled to a certain amount of the profits before the profits are divided up between the partners, and awarding interest is just a way of awarding a greater share of the profit to those who have contributed more capital.

  1. The appropriation account

Partnership and LLP profit and loss accounts are prepared in the same way as the profit and loss accounts of sole traders, but there will also be an appropriation account showing how net profit is divided between the partners.

  1. Current account

The net profit is owed to the proprietors of the business.

Where there is only one proprietor, the profit is usually credited directly to the capital account.

In the case of a partnership, it is usual to have a separate current account for each partner to which the appropriation of net profit (including salary and interest) is added and from which drawings are deducted.

The reason for separating capital and current accounts is that, as we have seen, partners are often entitled to interest on capital contributed. It is therefore desirable to keep the original capital contribution of each partner readily identifiable and unaffected by subsequent appropriations of profits and drawings.

  1. Balance sheet

The capital and current account balances of each partner are shown separately on the balance sheet in the capital employed section.

The partnership’s balance sheet would be very complicated if it included full details of salaries, interest on capital, profit shares and drawings. Usually, then, an appendix is provided to show movements on current accounts, and just the balance of the current account is inserted on the balance sheet.

  1. Changes in partners

If a partner leaves or joins part- way through an accounting period, the net profit for that year will be apportioned between the period before and the period after the change.

Two appropriation accounts will be prepared. The first will show the allocation of profit earned before the change in membership, in accordance with the partnership agreement before the change.

The second will show the allocation of profit earned after the change, in accordance with the shares set out in the partnership agreement in force after the change in membership.

  1. Taxation of partnerships

Partners submit their own tax returns, claiming their own personal allowances.

503
Q

What is the duty to act within powers?

A

A director of a company must:
a) Act in accordance with the company’s constitution* and
b) Only exercise powers for the purposes for which they are conferred

Need to check the company constitution (articles, special resolutions and other agreements).

Any director who is voting or acting to promote their own interests rather than those of the company would not be using their own powers for the purpose for which they are conferred.

504
Q

What are the CGT tax rates for residential property?

A

If the chargeable asset is residential property which is not the taxpayer’s main residence, the gains are subject to a surcharge of 8% for basic rate tax payers and 4% for higher rate taxpayers on top of the normal rates of 10% or 20%.

This means that any gains which are below the basic rate threshold are taxed at 18% (ie the normal rate of 10% plus the 8% surcharge) and any gains which exceed the basic rate threshold are taxed at 24%, not 20%.

505
Q

Allotment of shares

What needs to be done if there is a restriction on the power to allot shares under s 550?

A

A special resolution needs to be passed to amend the articles to remove this restriction.

506
Q

Why is it important to identify a person with more than 50% but less than 75% of company’s shares/voting rights?

A

Can both block and pass ordinary resolutions on their own because they have more than 50% of shares but they cannot pass special resolutions on their own.

507
Q

Can all directors count as part of the quorum in all board meetings?

A

The director may not count in the quorum or vote if a proposed decision of the board is concerned with an actual or proposed transaction or arrangement with the company in which the director is interested.

This can be disapplied by the MA.

But they can still count in the quorum of the meeting for other resolutions to be passed, it is just that specific one where they have an interest.

508
Q

What is the effect of contravention of a disqualification order?

A

This is a criminal offence and the director could be fined or sentenced to up to two years in prison.

A director who is disqualified is personally responsible for the debts of the company if they are involved in the management of the company while disqualified.

509
Q

When is a partnership liable to third parties?

A

Contractual liability

Actual authority

Apparent authority

510
Q

What information about first shareholders needs to be included in the IN01 form?

A

They are called subscribers and their names, addresses and details of their shareholding need to be included.

511
Q

What are partners’ responsibilities of a partnership under common law?

A

Partners owe a duty of utmost fairness and good faith towards one another.

512
Q

What are the administrative requirements post appointment of a new director?

A

Company must notify the Companies House within 14 days of the appointment.

On form AP01 (individual director) or form AP02 (corporate director).

The company must also enter the director on its register of directors and register of directors’ residential addresses.

513
Q

What are the conditions for being a limited partner in a limited partnership?

A

The limited liability of the partner is conditional, as they must not:
- Control or manage the LP
- Have the power to take binding decisions on behalf of the LP
- Remove their contributions to the LP for as long as it is in business

514
Q

With regards to sole traders and partnerships claiming trading loss relief, what happens if the taxpayer is eligible for more than one relief?

A

When the tax payer is eligible for more than one relief, they may choose which relief to claim.

515
Q

How is CGT calculated if there is a disposal between spouses?

A

When spouses or civil partners are living together, and one makes a disposal of an asset to the other, there is deemed to be no gain or loss so they do not need to pay any CGT.

The spouse will also receive the asset at the value of the original consideration not the market value.

So when they do eventually sell the asset they end up paying more CGT as they are acquiring it at the original consideration whereas under the normal rules they would acquire it at market value.

It is only deferred, and not wiped out entirely.

This rule gives spouses the opportunity to save tax by transferring assets to the other person.

If the husband or wife wishes to dispose of a chargeable asset but has already used up all of their annual exemption for the tax year, they can transfer the asset to their spouse or civil partner without paying tax.

Then the spouse or civil partner can dispose of the asset and use their annual exemption instead so that between them, they have exempt gains of up to £6,000, rather than £3,000.

An inter- spouse transfer would also be beneficial if one spouse paid tax at a higher rate than the other, so that the tax is paid by the spouse or civil partner who pays tax at the lower rate.

516
Q

What happens if a preference is proven?

A

If a preference is proven, the court may order the release of any security given by the company, the return of any property transferred as part of the transaction or the payment of proceeds of sale of property forming part of the transaction to the company.

517
Q

In terms of an alternative to bankruptcy, what is the disadvantage of negotiating with creditors as opposed to another alternative?

A

This will be an informational arrangement but as it is informal it would not stop another creditor from issuing a bankruptcy petition.

518
Q

What is business asset disposal relief?

A

Available on gains made by individuals on the disposal (ie sale or gift) of certain assets.

  1. What is the tax reduced to?

If business asset disposal relief applies, the rate of tax will be reduced to a flat rate of 10%.

  1. What conditions are there on the qualifying business asset?

2.1. Sole trader or partnership:

a) If the the business, or part of it, is disposed of as a going concern; or
b) assets are disposed of following end of the business (provided the assets were used
in the business at the time of cessation of business).

To be a qualifying disposal of part or the whole of a business, the interest in the business as a whole, not just one or more assets, must have been owned either:

a) throughout the period of two years ending with the date of disposal; or
b) throughout the period of two years ending with the cessation of the business, provided that the disposal is within three years after cessation of business.

Only assets used for the purposes of the business carried on by the individual or partnership are eligible for relief. Company shares, securities and other assets held as investments are not eligible.

  1. Company shares

3.1. What qualifies for this relief?

A disposal of company shares (including securities) may qualify for relief if:

a) the company is a trading company; and
b) the company is the taxpayer’s ‘personal company’ (which means that the taxpayer must hold at least 5% of the ordinary share capital in the company and that holding must give them at least 5% of the voting rights in the company), and either or both of the following two conditions are met:
i) the taxpayer must be beneficially entitled to at least 5% of the profits available for distribution to equity holders and at least 5% of the assets available on a winding up; or
ii) the taxpayer would be beneficially entitled to at least 5% of the proceeds of sale if the whole of the ordinary share capital of the company were disposed of; and
c) the taxpayer is an employee or officer of the company.

3.2. What is the deadline for qualifying for this relief?

The above requirements must have been satisfied throughout the period of two years:

a) ending with the date of disposal; or
b) ending with the date the company ceased to be a trading company (and the disposal
must be within three years of cessation of trading).

3.3. What is classed as a trading company?

To be classed as a ‘trading company’, the company must not have activities that include ‘to a substantial extent activities other than trading activities’.

This means that companies which hold substantial cash reserves or investments may fall outside the definition.

  1. What is the effect of the relief?

If the conditions for business asset disposal relief are met, the taxpayer will pay a flat rate of 10% tax on the gains.

This is subject to a lifetime cap of £1 million of qualifying gains.

Once the taxpayer has made £1 million of qualifying gains in their lifetime, any gains over and above this amount will not benefit from business asset disposal relief.

  1. When must the claim be made by?

The taxpayer must claim business asset disposal relief on or before the first anniversary of the 31 January following the tax year in which the qualifying disposal was made. So for a disposal made in 2023/24, the claim must be made by 31 January 2026.

519
Q

What are the duties and responsibilities of members of an LLP under an LLP agreement/statute?

A

The mutual rights and duties of the LLPs members are governed by an agreement between members or between the LLP and its members.

If there is any matter which the parties have not agreed, the LLP regulations 2001 will apply.

520
Q

How long do minutes/written resolutions of general meetings and written resolutions need to be kept after a share buyback has been agreed?

A

Need to keep minutes/written resolutions for ten years.

521
Q

Comparing debt and equity finance, what is the repayment of capital?

A

Equity finance

Companies do not repay the shareholders the capital unless the company is wound up although the shareholders could recoup the capital by selling their shares to a third party.

Debt finance

Loan capital must be repaid at some date in the future, possibly on demand.

Consequently, the directors of the company must make provision for this and ensure that funds are available to repay the loan whenever it falls due or the company will be in default.

522
Q

What are the articles of association?

A

This is essentially the company’s rule book. There is a set of standard articles, the Model Articles for both private and public companies.

The MA apply by default if the applicant does not provide a bespoke set of articles when applying to incorporate a company.

523
Q

How can a debtor apply for a breathing ace?

A

To obtain a breathing space, the debtor must apply to a debt advice provider authorised by the Financial Conduct Authority.

To grant the breathing space/moratorium, the debt adviser must be satisfied that the debtor cannot pay some or all of their debt as it falls due.

The debtor cannot be an undischarged bankrupt and must not be in an IVA or subject to a debt
relief order.

The debtor must also not have already had another breathing space in the previous 12 months. The debt must also be a qualifying debt, but nearly all debts will be.

The debt advice providers will use an electronic service provided by the Insolvency Service to contact creditors and maintain a database of debtors who are in a breathing space.

524
Q

What are the advantages of a security of a loan for a borrower?

A

The borrower may also benefit from the security as the lender will often allow the business to borrow at a lower interest rate if they have given security.

525
Q

How is the company’s name changed following purchase of a shelf company?

A

Need special resolution and then file form NM01 at CH along with the special resolution and the applicable fee.

As long as name is eligible, CH will issue a new certificate of incorporation.

526
Q

Do trustees pay income tax?

A

Trustees pay income tax on income produced by the trust.

527
Q

Who can be an auditor?

A

The auditor must be someone who is qualified (certified or chartered accountant) and independent (not connected to anyone in the company).

Usually companies appoint a firm of accountants to be the company’s auditor so any qualified member of the accountancy form can undertake the audit.

528
Q

What should be included in a PA regarding decision making ?

A

Apart from three exceptions, all decisions in a partnership must be taken by majority.

The three exceptions where decisions need to be made unanimously:
a) Changing the nature of the business
b) Introducing a new partner
c) Changing terms of a PA

Responsibility for certain decisions can be delegated to certain partners or employees.

The partners could agree to include more provisions which need to be decided by unanimous vote. But they need to be careful, as this would make decision more cumbersome.

529
Q

What is a transaction defrauding creditors (personal insolvency)?

A

This is a transaction at an undervalue with the company has entered into to put the assets beyond the reach of someone making a claim against it or to prejudice the interests of that person in relation to any claim that might make.

It is very difficult to show intention to put assets beyond someone’s reach or to prejudice their interests.

The claim is usually only made when the liquidator or administrator cannot bring a claim relating to a transaction at an undervalue because the time limit has expired.

530
Q

How are chargeable gains calculated for corporation tax?

A

The next step in the corporation tax calculation is to calculate chargeable gains. These are calculated in a similar way to CGT, but there are some differences.

  1. Stage 1: Identify a chargeable disposal

At this stage of the calculation, you must identify any disposal of chargeable assets.

The disposal can be by way of sale or by gift.

1.1. Chargeable assets

Chargeable assets for the purposes of corporation tax are defined in almost exactly the same way as for CGT. The most likely chargeable assets will be land, buildings, and shares in other companies. However, they must not form part of the company’s income stream, otherwise they will be classed as income profits rather than chargeable gains. In addition, different rules apply for the disposal of goodwill and intellectual property.

1.2. Plant and machinery

In practice, plant and machinery are unlikely to increase in value and so a chargeable capital gain is unlikely to arise.

1.3. Goodwill and intellectual property

Goodwill and intellectual property (eg trademarks, patents, design rights and copyright) are intangible fixed assets.

They are capital in nature but receipts from transactions in intangible fixed assets are treated as income receipts for the purposes of the company’s corporation tax calculation.

Expenditure on intangible fixed assets is generally deductible when calculating the company’s income profits.

When an intangible fixed asset is disposed of, and provided the conditions are met, any profit can be rolled over into the acquisition of replacement intangible fixed assets.

This means that the corporation tax in relation to the profit from that disposal is deferred.

  1. Stage 2: Calculate the gain (or loss)

To calculate the gain (or loss) on a chargeable disposal, you should follow the calculation set out below.

Proceeds of disposal (or market value in the case of a gift or sale at an undervalue)
LESS
Costs of disposal
= Net proceeds of disposal
LESS
Other allowable expenditure (initial and subsequent expenditure)
= Gain (before indexation) or loss
LESS
Indexation allowance
= Gain (after indexation)

2.1. What is a sale at an undervalue?

If the company sells an asset below market value, this may just be a bad bargain, in which case, the actual sale price will be used as the figure for proceeds of disposal.

When will the sale price be deemed to be at market value?

However, if there was a gift element to the sale at an undervalue, the market value of the asset will be used as the figure for proceeds of disposal.

If the sale was to a ‘connected person’, it will be deemed to have taken place at market value instead.

A person is a ‘connected person’ if that person controls the company, either alone or with others connected to them.

A company is connected to another company if they are both controlled by the same person, or by a combination of that person and others connected with them.

If disposing of the asset results in a loss and not a gain, the loss can be deducted from the company’s chargeable gains for the accounting period, but not from its income profits. Any unused loss can be carried forward to subsequent accounting periods to be deducted from the first available chargeable gains made by the company.

2.2. The indexation allowance (only relevant for disposals between 1 April 1862 up to 30 December 2017)

When an asset increases in value, the gain cannot all be regarded as profit. Some of the gain will be as a result of inflation, rather than the asset becoming inherently more valuable.

An indexation allowance removes the inflationary gains from the corporation tax calculation, meaning that a smaller gain is charged to tax.

To calculate the indexation allowance, you need to apply to initial and subsequent expenditure (but not costs of disposal) the percentage increase in the Retail Prices Index from the date the expenditure was incurred to the date of disposal of the asset (or, if earlier, 31 December 2017 – see below).

HMRC publishes tables to show this figure as an indexation factor to make the calculation easier.

After calculating the allowance, you must deduct it from the gain (before indexation) to give the gain (after indexation). The indexation allowance is used when calculating the gain on an asset owned for any period from 31 March 1982. If various items of expenditure were incurred at different times, different indexation factors will need to be used for each item of expenditure. Indexation was frozen on 31 December 2017, meaning that no indexation allowance is applied to any initial and
subsequent expenditure from December 2017 onwards.

Since the concept of indexation was not introduced until 31 March 1982, the indexation
allowance for assets owned on 31 March 1982 is based on the market value of the assets on 31 March 1982, not actual cost.

Note that the indexation allowance cannot be used to create or increase a loss.

  1. Stage 3: Apply reliefs

Fewer reliefs available to companies

3.1. Rollover relief on the replacement of qualifying business assets

This relief allows companies to postpone the payment of corporation tax following the disposal of a qualifying asset, when the consideration received for the qualifying asset is used to acquire another qualifying asset.

3.1.1. What is a qualifying asset?

The main qualifying assets are land and buildings, although other more unusual assets, such as ships, also qualify. The company must use the asset in its trade: assets held by way of an investment are not qualifying assets.

3.1.2. What is not a qualifying asset?

a) Company shares are not qualifying assets;
b) The relief does not apply to goodwill and other intellectual property, which are subject to a separate rollover relief;
c) fixed plant and machinery are qualifying assets, but selling such assets will rarely produce a gain because they usually depreciate in value. Further, rollover relief in relation to such assets will be restricted if they are wasting assets. There is no statutory definition of fixed plant and machinery, but if the item is movable, and intended to be movable so that the layout of the workplace can change, it will not be classed as fixed.

For the relief to apply, the asset disposed of and the asset acquired with the proceeds of sale do not need to be the same type of asset. They both just need to fall within the definition of qualifying asset.

3.1.3. Time limits

The company must acquire (or have acquired) the replacement asset within one year before or within three years after it disposes of the original asset, unless HMRC allows the company to have an extended period to acquire the replacement asset.

3.1.4. The effect of the relief

If the company disposes of a qualifying asset, and purchases another qualifying asset to be used in the company’s trade within the time limits set out above, the company’s liability to corporation tax arising from the disposal can be postponed by rolling over the gain on the disposal of the original asset into the acquisition cost of the replacement asset.

The gain is notionally deducted from the acquisition cost of the replacement asset. This gives a lower acquisition cost to use in the corporation tax calculation when the replacement asset is sold.

In essence, when the replacement asset is sold, it will be deemed to have a lower acquisition cost than it actually had. This will mean that the difference between the initial acquisition cost of the replacement asset and its sale price will be greater, and more corporation tax will be paid overall, to account for the gain on the original asset. If only part of the consideration received on the disposal of the original asset is used to purchase the new asset, there will be restrictions on the availability of rollover relief.

531
Q

What is the deadline for filing forms at CH if there are changes to the PSC register?

A

14 days from the date the company made the change in its PSC register

532
Q

Regarding income tax, which types of income are deducted at source?

A

Tax on salary, cash and most non- cash benefits is generally deducted at source by the employer.

When tax is deducted at source, the payer of the income acts as a tax collector. They will deduct the correct amount of income tax from the income payment and pay it to HMRC.

The taxpayer receives the income net of tax so the tax has already been taken off.

533
Q

What are the administration and notification requirements regarding the resignation/removal of company secretaries?

A

The company must notify the Registrar of Companies within 14 days of their resignation or removal on form TM02.

The register of secretaries needs to be amended to reflect the fact that the company secretary has left office.

534
Q

Comparing debt and equity finance, what is the degree of statutory control?

A

Equity finance

This is tightly controlled by the CA 2006.

Debt finance

This is predominantly a matter of contract law and may be more of a flexible way for the company raise money.

535
Q

Are public companies required to have a company secretary?

A

Must have a company secretary.

536
Q

Who has liability for the responsibility to file and keep accurate accounting records?

A

The directors have responsibility to ensure that accounts are produced for each financial year.

The accounts must give a true and fair view of the state of affairs at the company as at the end of the financial year.

537
Q

What is the implication if a company has MA but they want to execute a document using the company seal?

A

Execution by company seal is more complicated. MA 49 states that if a company wishes to use its seal to execute a document, the documents must also be signed by at least one authorised person (director, company secretary or other authorised person) in the presence of a witness who attests the signature).

538
Q

When is there no presumption of insolvency for a preference?

A

Unlike with transactions at an undervalue there is no presumption of insolvency if the preference was given to an unconnected person.

539
Q

Allotment of shares

If a company is a private company with one class of share incorporated pre CA 2006, what needs to be considered with regards to the director’s authority to allot shares?

A

The shareholders must pass an ordinary resolution to activate s 550. Before the CA 2006 came into force, the directors did not automatically have authority to allot shares and the shareholders had to give permission.

540
Q

What are the consequences for failing to keep accurate statutory records or files at CH?

A

Criminal offence - penalty of a fine for the company and every officer in default.

541
Q

How is CGT calculated in a partnership (made up of individuals rather than corporate partners)?

A

When a chargeable asset owned by a partnership of individuals is disposed of, resulting in a chargeable gain, a charge to CGT will arise just as it would if an individual or sole trader were to sell a chargeable asset.

The context in which CGT is likely to be relevant for partnerships is the sale of partnership assets, most likely land, premises, goodwill and investments.

When a firm disposes of a chargeable asset, this is treated for CGT purposes as if each partner is making a separate disposal of part of the asset.

Each partner pays a proportion of the CGT based on their percentage ownership of the partnership’s assets.

Sometimes the partners have expressly agreed how they will share the partnership’s capital assets. For example, they may have agreed to share capital and capital profits in the same proportion as their initial capital contributions.

  1. Calculating the gain

Just as each partner will only pay tax on part of the capital gain arising from the sale of the partnership asset, the disposal proceeds and allowable expenditure must also be apportioned among the partners, according to their share of the asset.

  1. Business reliefs

Each partner can make their own decision regarding which reliefs, if any, they want to apply for.

Which relief or reliefs are available will depend on the circumstances. It may be that one partner can claim a relief to which the other partners are not entitled.

  1. Individual disposals

A partner may dispose of their share in the partnership assets even when the firm as a whole is not making a disposal, usually when a partner leaves a partnership.

The principles explained above govern how that partner’s fractional share of any gain is calculated.

When a new partner joins a partnership, this will involve the other partners disposing of part
of their existing share of the asset because the assets are shared amongst more partners.

542
Q

What happens when a bankruptcy order has been made?

A

The OR acts as a trustee in bankruptcy and takes control of the bankrupts assets.

The OR will ask the debtor for a statement of affairs to detail their financial position and recent financial transactions.

The OR may then make further investigations into the debtor’s financial affairs.

The OR will also take steps to protect the debtor’s property and will dispose of or sell any property which is perishable or decreasing in value.

It is possible to appoint a private trustee if they have enough assets to pay for them.

The bankrupts assets vest automatically in the trustee from the moment the bankruptcy order is made.

The trustee has the power to investigate the bankrupt’s affairs and set aside or challenge transactions which the bankrupt entered into before the bankruptcy order was made.

The trustee must realise and sell the bankrupt’s assets and use the proceeds of the sale to pay the creditors.

When the trustee has done this, they will apply to be released.

The trustee has the power to investigate the bankrupt’s affairs and set aside or challenge transactions which the bankrupt entered into before the bankruptcy order was made, which is effectively part of the realisation of the assets process.

543
Q

How are partnerships taxed (all partners are individuals)?

A

The partners are taxed separately as self-employed individuals paying income tax on their share of the profits of the partnerships.

544
Q

What rights do shareholder’s derive from the articles of association?

A

This is the company’s constitution and is a statutory contract between each shareholder and the company and between each shareholder and every other shareholder.

Shareholders have a remedy for breach of contract if one or more shareholders or the company itself does not abide by the terms of the constitution.

It allows shareholders to take action against other shareholders of the company where that shareholder’s membership rights have been infringed.

545
Q

When should a director declare interest in existing transaction or arrangement?

A

They must declare the nature and extent of the interest to the board of directors. This applies where the director is directly or indirectly interested in a transaction or arrangement that has been entered into by the company.

So this is when the transaction has already happened.

When must the declaration be made?

As soon as reasonably practicable.

546
Q

What must be done after shares have been transferred?

A

The company must register the transfer so enter the new person on the register of members and or reflect the existing shareholders increase in shares, as soon as possible.

547
Q

How can a charge holder protect themselves if they know the asset is already charged?

A

The charging agreement should include a covenant by the company that there are no earlier charges which are subject to a negative pledge clause.

If this is not true, the company will be in breach of the agreement and it may be terminated immediately.

548
Q

Once the NSNDI has been worked out from taxable income for step 4 of income tax, what needs to be done with the NSNDI figure?

A

First to be taxed is NSNDI. Once you have worked out what NSNDI is, you must tax it at the
appropriate rates. You also need to know the tax bands, so that you know at what point a taxpayer will start to pay tax at a higher rate. The income tax bands for NSNDI are:

Basic rate of 20% is used for taxable income of 0 - 37,700

Higher rate of 40% is used used for taxable income of 37,701 - 125,140

Additional rate of 45% is used for taxable income of over 125,140

549
Q

Does there have to be a register of members if there is only one member?

A

If the company only has one member, there must be a statement to that effect on the register of members.

550
Q

Who needs to be given notice of a GM?

A

The directors must give notice to every shareholder and every director and to the auditor if there is one.

It must be given in hard copy, in electronic form or by means of a website or a combination of these means.

551
Q

What is the implication of a group structure on VAT and Stamp Duty?

A

Firstly, stamp duty and stamp duty land tax will not be charged on transfers of assets between companies which are in a qualifying group, provided that certain conditions are met.

Secondly, in terms of VAT, a group of companies may be able to register for VAT as a group under a single registration.

552
Q

How is an administrative receiver appointed?

A

The loan agreement will list the events that trigger the lender being able to appoint an administrative receiver.

553
Q

What information do business accounts provide?

A

Businesses need to be able to calculate their profits. To do this, they need to know how much income they have received in an accounting period, and what expenses they have incurred.

They also need to know what their assets and liabilities are, so that they can ascertain what the business is worth. Understanding these four categories – income, expenses, assets and liabilities – is key to understanding bookkeeping and accounts.

  1. Income

Income is what the business earns from trading or offering its services, for example, money received for the sale of furniture in a furniture shop, or money paid for legal services supplied by a law firm.

  1. Expenses

Expenses are items the business has paid for, and which it will benefit from for a short time. Examples include, the price paid for gas, electricity, petrol, or stock bought for resale – those sums it needs to incur in order for the business to operate.

  1. Assets

Assets are essentially what the business owns, or has a right to own. For example, its business premises, if it owns them; machinery; vehicles; cash and debtors (because it has a right to the money owed to it by its debtors, and, in theory, it will receive this money when the debtor pays). Whereas expenses will benefit a business for a short time, assets tend to provide a
longer- term benefit to a business.

  1. Liabilities

Liabilities are what the business owes. Examples are money outstanding on a bank loan, or
money owed to creditors.

554
Q

Can a company’s articles alter the percentage of shareholders with voting rights in the campy who can require the company to circulate a written resolution?

A

The company’s articles can reduce this to below than 5% but cannot increase it to more than 5%.

555
Q

What is the notice for dissolution of partnership?

A

The notice does not need to be for a particular length of time or need to be in writing unless the partnership agreement is a deed.

Therefore, a partner can end a partnership with immediate effect by merely saying to the other partners that they wish the partnership to end which is risky.

556
Q

Which reliefs cannot be used in conjunction with rollover relief on the incorporation of a business?

A

If rollover relief on incorporation applies to a transfer of a business, business asset disposal relief and the annual exemption cannot be used. In addition, rollover relief on incorporation cannot be used at the same time as rollover relief on replacement of qualifying assets (because shares are not qualifying business assets) or hold- over relief (because there is usually no gift).

However, if part of the consideration for the sale is cash and part is shares, it is sometimes possible to use rollover relief on incorporation for the part of the gain relating to the part of the business exchanged for shares. Business asset disposal relief and the annual exemption could be used on the part of the gain relating to the part of the business sold for cash.

557
Q

What is the corporation tax financial year?

A

The corporation tax financial year runs from 1 April to 31 March (which may differ from the company’s financial year, which can start at any time of year). Financial years are described by reference to the period in which they commence, so the corporation tax year running from 1 April 2023 to 31 March 2024 is ‘Financial Year 2023’.

558
Q

What are the CGT tax rates for business asset disposal relief?

A

Any gains which qualify for business asset disposal relief are taxed at 10%, regardless of the taxpayer’s income.

How is this calculated with other capital gains?

Sometimes taxpayers have gains that qualify for business asset disposal relief and other gains that do not qualify.

In these cases, the business asset disposal relief gains (taxed at 10%) will be added to their income first, meaning that their other gains will be treated as the top slice of their income and therefore be more likely to be taxed at 20%, or at 18/24% on residential property (depending on the income tax band).

So effectively it takes away a chunk of the lower tax band so the rest is charged at a higher rate.

559
Q

What are the administration and notification requirements regarding the appointment of company secretaries?

A

The company must notify the Registrar of Companies on form AP03 (human secretary) or AP04 (corporate secretary) within 14 days of the appointment of a company secretary.

Any company that has a company secretary must keep a register of secretaries with certain specified particulars.

Private companies can elect to keep this register at Companies House as long as they file and keep the information there up to date.

560
Q

What are the advantages of a shareholder’s agreement?

A

Privacy and protection of minority shareholders.

The company’s articles provide very little protection for minority shareholders so a shareholder’s agreement can provide this.

561
Q

How is administration commenced through the out-of-court route - appointment by company or directors?

A

Notice of intention of administration is served on:
a) The court
b) Any QFCH
c) Any lender who is entitled to appoint an administrative receiver.

Directors file a statutory declaration at court that the company is unable to pay its debts and is not in liquidation.

Means that if the compulsory winding up petition in relation to the company has already been presented at court, the directors cannot use the out of court route and must instead apply to the court for an order that the company go into administration in place of a winding up petition being allowed to proceed.

The moratorium comes into effect as soon as the notice of intention to appoint is filed at court.

562
Q

What is the duty to promote the success of the company?

A

A director must act in the way he considers in good faith would be most likely to promote the success of the company for the benefit of its members as a whole.

563
Q

What should be included in a PA regarding dissolution?

A

The partnership is dissolved even if all but the outgoing partner carry on the business together on the same terms.

The PA should set out under what circumstances a partner can retire or when the partnership will come to an end. This gives partners more control than relying on the default position in the PA 1890, which is that any partner can end the partnership at any time by giving notice of their intentions to do so to all of the other partners.

Good idea to include a provision stating that notice to retire from the partnership cannot be given in the first year of trading to prevent the disastrous scenario of a partner leaving whist the partnership is not making profit.

564
Q

Who brings wrongful trading claims?

A

A liquidator or an administrator.

565
Q

What should be included in a PA regarding the name of the partnership?

A

Good idea to include the name in the partnership agreement so that they can be sure what has been agreed.

Must not:
a) Include any company prefix that does not imply a partnership
b) Be offensive
c) Be the same as an existing trademark
d) Contain a sensitive word or expression or suggest a connection with government of local authorities without permission

566
Q

When do individuals have to pay income tax?

A

Aside from the type of income below, individuals may have to pay income tax on other types of income too, for example, income from investments and pensions, interest on bank or building society accounts and dividends received from companies in which they hold shares.

Employees:

Individuals who are employed will have to pay income tax if their earnings exceed a certain threshold (and this is usually assessed by their employer and paid directly to HMRC by the employer)

Sole traders:

Sole traders will also have to pay income tax, based on an assessment of their trading profits.

567
Q

What is the purpose of the PSC register?

A

The PSC register enables third parties to understand who holds power in the company. This must be done even if there are no persons of significant control so it can indicate to there are no PSCs.

568
Q

What should be included in a PA regarding shares in income and capital profits and losses?

A

Under the PA 1890, the partners share equally in the capital and the income profits and the losses of the business.

Any clause setting out that the partners share capital in unequal proportions will vary the default decision set out in the PA 1890. Case law has established that it can sometimes be inferred through course of conduct.

The partnership agreement might provide for interest to be paid on capital contributions, to encourage investments and to reward the partners for contributing capital to the partnership.

The partnership agreement should also state the proportions in which income profits after any salaries and interest payable on capital are to be shared.

Often the partners’ share of income profits will depend on their working hours.

569
Q

How does a mortgage work?

A

With the exception of land, a montage involves the transfer of legal ownership from the mortgagor to the mortgagee (lender).
Whilst the mortgagee has the right of immediate possession of the property, this is only exercised if the borrower defaults on the money.

When the money is repaid, the title is transferred back to the borrower.

A separate mortgage must be created over each asset.

A mortgage taken over the land is actually a charge by deed that is expressed be by way of legal mortgage.

The rights of the mortgage over land include the rights to take possession of the land and to sell it.

570
Q

What is the priority of charges?

A

If all charges are registered properly:

a) Fixed charge or mortgage take priority over a floating charge of the same asset even if the floating charge was created before the fixed charge or mortgage
b) If there is more than one fixed charge or mortgage over the same asset, they have priority in order of their date of creation not their date of registration
c) If there is more than one registered floating charge over the same asset, they have priority in order of their date of creation not their date of registration.

571
Q

Can the 21 day period for registration of charge be extended?

A

The court has a limited power to extend the 21 day period if the failure to deliver the required documents was accidental or due to inadvertence or if it would not prejudice the position of other creditors or shareholders of the company.

If the application to extend the registration time is successful, the charge will only be effective from the date of actual registration and in that in between period would have lost priority so other charges could have been registered.

572
Q

Is there a desire to prefer or an intention to prefer for a preference?

A

There must be a desire to prefer the other party rather than just an intention to prefer them.

573
Q

What different types of term loans are there?

A

Term loans can be secured or unsecured and bilateral or syndicated.

A bilateral loan is between a bank and a borrower. A syndicated loan is between a borrower and a number of different lenders.

Syndicated loans are common when the risk is high because it spreads the risk.

The term loan may allow the business to take out the loan in one go or instalments on agreed dates.

The advantage of the instalment option is that it will reduce the amount of interest that needs to be paid.

574
Q

What action may the court take if prejudicial conduct is found in an unfair prejudice claim?

A

May make such order as it sees fit.

Most commonly an order that the other shareholders/the company must buy the shares of the unfairly prejudiced shareholder.

Or a restriction on the company altering its articles of association without the leave of the court and an order that the unfairly prejudiced shareholder has permission to bring a derivative claim.

575
Q

What should be included in a PA regarding financial input ?

A

The agreement should set out the amount of the partners’ initial capital contributions and whether they will be obliged to contribute more capital in the future.

576
Q

How are shares transferred?

A

The transferor must complete and sign a stock transfer form and give it to the transferee along with the share certificate relating to the shares.

Pay any stamp duty if applicable.

Then transferee must send the share certificate and the stock transfer form to the company.

The company then sends the new shareholder a new share certificate in their name within two months.

The company should also enter their name on the register of members within two months.

The company should notify the Registrar of Companies of the change in ownership of the shares when the company files its annual confirmation statement (CS01).

577
Q

What are the exceptions to the rule that directors need to declare their interest in a transaction?

A

It cannot reasonably be regarded as likely to give rise to a conflict of interest

The directors are already aware of it

To the extent that it concerns the terms of a service contract that have been or are to be considered by a meeting of the directors

578
Q

What happens if a company becomes insolvent whilst undergoing members’ voluntary liquidation?

A

The liquidator will concert the MVL into a CVL.

579
Q

When can the bank demand repayment of a term loan or revolving credit facility?

A

The bank can only demand repayment of term loan or revolving credit facility only in line with the terms of the facility agreement.

The facility agreement will set out the agreed repayment schedule for the loan.

It may provide for the repayment of:
a) The whole loan in one go at the end of the term (bullet repayment)
b) In equal instalments over the term of the loan (amortisation)
c) In unequal instalments with the final payment being the largest (balloon repayment)

Advantage for lender of repayment in instalments:

This will give the lender early notice should the business have difficulty in making repayments because the business is likely to default on a payment if it is in financial difficulty.

580
Q

What is the written resolution procedure for shareholder resolutions?

A

The board may propose that the shareholders pass a resolution by written procedure instead of in person.

Instead of issuing a notice of general meeting, the board will instead hand out, post or email a written resolution or place the resolution on a website.

The document will set out the text the ordinary and/or special resolutions which the board is proposing and the shareholder will have to sign and return the written resolution if they would like to vote in favour of it.

If the written resolution is sent out by post or email, each shareholder will receive their own copy to sign and return.

The written resolution must be sent to every eligible member I.e. every shareholder who is entitled to vote on the resolution as at the circulation date of the resolution.

581
Q

Do PRs pay income tax?

A

Personal representatives pay the deceased’s outstanding income tax and income tax chargeable during the administration of the estate.

582
Q

What is a share certificate?

A

All shareholders have a right to receive share certificates.

This is important because the share certificate is the evidence of the holder’s title of shares.

583
Q

How does the administration end?

A

The administration ends automatically one year from the date the administration took effect but this can be extended.

It can also be ended earlier if:
a) its objective has been achieved,
b) if the administrator believes its objective cannot be achieved or
c) by application to the court by a creditor

The creditor can then present a winding up petition.

584
Q

Before commencing trading what must a public company obtain?

A

Before commencing trading, it must obtain a trading certificate as proof that it can trade and borrow and it has met the allotted share capital requirements. This is set out in a trading certificate.

The application for a trading certificate is made to CH on form SH50.

585
Q

What is a key feature of the restructuring plan?

A

Cross-class cram down provision

This provision allows a dissenting class of creditors to be crammed down so that they cannot block otherwise viable plans and it must be sanctioned by the court.

The court will only sanction it where it is satisfied that no member of the dissenting class would be any worse off under the plan than they would be if the court were not to sanction the plan.

586
Q

What is meant by implied actual authority?

A

The board has not expressly permitted the director to act in a certain way but the director has acted that way in the past and the board has not tried to stop them.

587
Q

What is a floating charge?

A

If the company needs the assets as part of its business operations a floating charge is more suitable.

It is possible to create more than one floating charge over the same assets.

Features of a floating charge:
a) Consists of an equitable charge over the whole or a class of the company/LLP assets such as stock
b) The assets subject to the charge are constantly changing
c) The company/LLP retains the freedom to deal with the assets in the ordinary course of business until the charge crystallises

It is possible to take a floating charge over the whole of the company undertaking.

588
Q

Which reliefs cannot be used in conjunction with business asset disposal relief?

A

Business asset disposal relief cannot be used in conjunction with rollover relief on replacement of qualifying assets or held over on the gift of business assets.

The annual exemption can be used to reduce gains before applying business asset disposal relief.

589
Q

What kind of implied covenants can be implied into a contract between a lender and borrower?

A

The court’s power to imply terms is limited.

It will only imply terms if it is necessary to give business efficacy to the contract or if the term is so obvious that it goes without saying.

The court could not imply a term that is inconsistent with an express term of the contract.

590
Q

What is a non-cash asset in the context of a substantial property transaction?

A

This is any property or interest in property other than cash.

A loan would not be covered.

591
Q

What is the timeline of a share buyback out of capital?

A

Board must prepare the counts no more than 3 months.

Directors must make a statement of solvency and hold a board meeting to call a GM and circulate a written resolution and approve the auditor’s report.

There must be no more than a week between the statement of solvency and the general meeting.

The GM/WR needs to pass an ordinary resolution to authorise the buyback and a special resolution to authorise the payment out of capital.

Within one week of the special resolution, the statement of solvency and the notice in the London Gazette and other newspapers need to be filed.

The board resolves to enter into the contract and between 5 and 7 weeks after the SR, the payment should be made out of the capital.

592
Q

What is meant by ‘holding out’?

A

When a creditor of a partnership has relied on a representation that a particular person was a partner in the firm this is known as holding out. The creditor may be able to hold that person liable for the firm’s dent.

This is the case if the person was never a partner or they retired before the contract was made.

The holding out may be oral, in writing or b conduct.

The representation may be made by that person or provided it is made with the person’s knowledge by another person.

Any creditor who can establish that someone held themselves out or allowed themselves to be held out as current partner, that they relied on the holding out and that they gave credit to the firm as a consequence will be able to sue that person for the debt owed by the firm.

593
Q

What are the consequences of failing to comply with s 182 duty to declare?

A

This is a criminal offence and is punishable by a fine. This contrasts with s 177 where failure is a civil matter.

594
Q

What are preference shares?

A

Their rights are enhanced over and above the ordinary shareholders. These are set out in the articles of association.

If the preference is 5% share, then the holder will receive a fixed dividend of 5p for each £1 preference share they own. So in an example where an investor agrees to invest £100,000, if enough profit is made to pay dividend, the investor will receive 5% of their initial investment so £5,000.

Preferential share holders are often people who wish to invest in a company and are willing to forego voting rights in return for greater financial returns.

595
Q

What is the effect of an administration order?

A

The director’s powers cease even though they remain in office and the administrator manages the company.

They are in control of the company’s assets although they do not own them and the administrator carries out the proposals approved by the creditors.

The moratorium continues throughout the administration including the above.

596
Q

Which documents are kept at a company’s registered office?

A

Board minutes and minutes of general meetings. Many companies will have their statutory bools at the registered office. A company’s statutory books are records which the company is required by statute to maintain such as the register of members, the register of directions and the register of director’s residential addresses.

Companies can now elect to keep these records at Companies House instead.

597
Q

What is the effect of director’s disqualification from office?

A

A director subject to a disqualification order cannot without leave of the court be a director in any way concerned in the promotion, formation or management of a company.

598
Q

Who can request a general meeting?

A

Shareholders representing atleast 5% of paid up capital of the company WITH voting rights can request a general meeting.

599
Q

How is a negative pledge clause disclosed?

A

Completing a section of form MR01 sent to CH and the clause itself will be included in the certified copy of the charging document which is delivered to the Registrar.

600
Q

What is the default position under statute regarding the capital and profits of an LLP?

A

Members of the LLP share in the capital and the profits equally.

No default as to losses as these are borne by the LLP itself. The members only risk losing their financial capital contributions and if they have loaned money to the LLP, not being repaid.

601
Q

What are the formalities for release of charge/sale of property?

A

If for whatever reason, the lender decides to release the borrower’s property from the charge or allow it to sell the asst covered by the charge, a person with an interest in the registration of the asset must sign and send Form MR04 to the Registrar of Companies House.

The Registrar will include a statement, either of the release of the charge or that the property no longer belongs to the company on its file.

If any entries were made against the land at the Land Registry, these should now be removed.

602
Q

Why does income need to be divided into chargeable sources?

A

Income is divided into chargeable sources because the sources of income are treated differently. For example, a landlord may receive income from a rental property, but is permitted to deduct from this expenses such as repairs to the property. Income tax is then charged on the profit, rather than the whole amount of the rent received.

Other types of income are not treated in the same way as rental income so need to be calculated separately in the tax calculation.

For example:

  1. Sole traders - trading loss

Sole traders may suffer a trading loss, for which relief may be available (see relevant section).

  1. Deductions at source - PAYE

The taxpayer receives most types of income without any tax having been deducted beforehand. For example, a landlord will receive rental payments in full: the tenant does not deduct a sum to cover the landlord’s income tax before paying the rent. This income is referred to as being received ‘gross’.

However, the main source of income for most taxpayers will be their salary. Tax will already have been deducted at source from this income. The payee receives the net amount and not the gross amount, and the employer pays the employee’s income tax directly to HMRC at the time the salary is paid through the Pay As You Earn (PAYE) system.

The PAYE system also ensures that the personal allowance is deducted. This is relevant because the fact that an employee has already paid income tax must be reflected in the income tax calculation, otherwise the end figure, the tax payable, is inaccurate.

Therefore we must ‘gross up’ any sum received by the employee to find the original sum from which the tax was deducted. We must then enter this gross figure into the calculation of total income.

Grossing up employment income is not a simple task because income tax will be deducted from salary at various rates and national insurance contributions will also be deducted.

The position may be complicated further by pension payments or benefits such as health insurance.

  1. Savings and income

Interest and dividends are both paid gross. Both savings and dividend income benefit from annual tax- free allowances. Both savings and dividend income form part of total income and must be added to other income at this stage.

603
Q

Who is liable for the debts of a partnership?

A

They are personally liable for all of the debts of the partnership.

They personal assets are at risk if there is not enough money in the business to pay creditors.

604
Q

Are damages for personal injury chargeable for CGT purposes?

A

Damages for personal injury are exempt from CGT, even though recovery of other damages or compensation could constitute the disposal of a chargeable asset.

605
Q

What is a mental health crisis breathing space?

A

This is a type of debt respite scheme.

A mental health crisis breathing space is only available to a client who is receiving mental health crisis treatment.

The mental health crisis breathing space has some stronger protections than the standard breathing space.

It lasts as long as the client’s mental health crisis treatment, plus 30 days (no matter how long the crisis treatment lasts).

606
Q

What are ordinary shares?

A

Gives the shareholders the right to attend and vote at general meetings.

They are also entitled to receive dividends if they are declared (if the directors recommend payment and the shareholders approve it).

There may be different types of ordinary shareholder. The rights attaching to the shares will be set out in the company’s articles of association and if they are not, all shares rank equally.

607
Q

When assessing the second stage of a derivative claim, what must the court take into account?

A

Whether the shareholder is acting in good faith by seeking to continue the claim.

The importance someone acting in good faith would attach on continuing the claim.

Whether any past or future action or omission was authorised, or if not, would be likely to be ratified

Whether the company has decided not to pursue the claim

Whether the act or omission gives rise to a cause of action that a member could pursue in their own right.

Whether there is any evidence showing the views of shareholders who have no personal interest in the matter as they are likely to be more objective

608
Q

What are the advantages of a credit facility?

A

Flexibility.

Possible to reduce the amount of interest payable by reducing the amount of interest payable by reducing borrowing.

609
Q

How do sole traders calculate their trading profits?

A

Sole traders determine their trading profits with reference to an accounting period, which is usually 12 months long.

The sole trader will choose an accounting period to suit them and their business, and it will not necessarily be the same as the income tax year, which runs from 6 April to 5 April the following year. For this reason, there are rules on how trading profit is assessed for the purposes of income tax.

From the tax year commencing 6 April 2024, where a business’s accounting period does not coincide with the tax year, profits and losses will be apportioned between the different tax years.

This will usually be calculated on the basis of the number of days of the accounting period falling in each tax year. Whilst from 6 April 2024 this rule will apply to all businesses, it will have immediate effect for all new businesses starting trading in the 2023/24 tax year.

610
Q

Can an LLP own its own property?

A

LLPs can own property and it is the legal owner.

611
Q

In what scenarios may a preference be a possibility?

A

Where one creditor is paid before one of the other creditors or where an unsecured creditor is given security.

612
Q

Is a partnership a separate legal entity?

A

A partnership is not a separate legal entity. The partnership does not own any of the assets, they are owned by the partners.

613
Q

What are examples of VAT zero rated supplies?

A

a) Book
b) Certain food
c) Water

Can reclaim the VAT they have paid from HMRC.

614
Q

What happens if a charge is not registered at the CH within the time period?

A

This renders the charge void against a liquidator or an administrator of the company and also against the company’s other creditors.

The company is still obliged to repay the debt and it is is repayable immediately but the lender cannot enforce the security,

The liquidator/administrator must ignore the original charge in determining how much creditors get and in what order, if the company goes insolvent.

The original charge holder therefore loses their priority over proceeds of sale of the asset and is treated the same as other unsecured creditors.

615
Q

Can a board resolution be passed in writing?

A

Yes but only if there is a unanimity between directors i.e. they all agree.

616
Q

What are the disadvantages of a term loan?

A

The time and expense in negotiating and agreeing all the legal documents for the loan.

Once repaid, the money cannot then be re-borrowed by the business, it is a one time agreement.

617
Q

Who does the director of a company owe their duties to?

A

The director’s duties are owed to the company itself not to the company’s shareholders or creditors.

When a claim is made against a director for breach of duty, the claimant is the company itself.

It is the board of directors who will decide to take legal action for breach of duty on the company’s behalf (derivative actions are an exception to this rule).

618
Q

Can directors vote in board resolutions to promote their own interests?

A

They must act as directors and promote the success of the company without thinking of their personal interests as a shareholder.

They must also adhere to their other duties.

619
Q

Can the statutory rule that directors need to declare an interest in a transaction be disapplied by the MA?

A

This cannot be disapplied by the MA.

620
Q

What are the advantages of an LLP?

A

Members have limited lability for debts of the LLP.

They are able to grant fixed and floating charges over their assets, unlike general partnerships.

Flexibility in the management structure.

They can appoint an administrator.

621
Q

What are the key shareholder ordinary resolutions?

A

To approve a long- term service contract
To ratify a director’s breach of duty
To appoint a director
To remove a director
To appoint an auditor
To remove an auditor
To authorise a substantial property transaction
To temporarily suspend
To authorise a loan to a director
To authorise a payment for loss of office
To ‘activate’ s550
To authorise directors to allot shares
To remove the authorised share capital clause in the company’s articles
To authorise a buyback contract

622
Q

What is the duty to exercise reasonable care, skill and diligence?

A

This is a) to the standard of a reasonably diligent person with the general knowledge, skill and experience that may reasonably be expected of a person carrying out the functions carried out by the director in relation to the company (objective test) and b) the general knowledge, skill and experience that that person has (subjective test).

Part A is the minimum standard expected.

Part B is a higher standard that is applied where the director’s knowledge, skill and expertise s greater than the Part A standard.

623
Q

What are a shareholder’s rights relating to the exercise of power to vote at a GM?

A

Right to send a proxy to a GM on their behalf

Right to a poll vote

Right to receive notice of general meetings

Right to requisition a general meeting

Right to apply to court to call a GM if not possible otherwise e.g. shareholders are refusing to attend a GM and it is not possible to hold a meeting that is quorate otherwise.

Right to require circulation of a written statement to be dealt with at a general meeting. They must have 5% or more voting rights of the company.

Right for shareholders holding 5% or more of the company’s shares to require the company to circulate a written resolution and accompanying statement.

624
Q

Does a partnership agreement have to be written?

A

The terms do not need to be written down they can be an oral contract but written is more desirable to avoid disputes.

Agreements can also be implied by conduct e.g. if a partner is used to acting in a certain way over a period of time and other partners have not objected.

625
Q

What is an unfair prejudice petition?

A

Allows any shareholder to apply to court for an order for a remedy where they feel that they have been unfairly prejudiced as a shareholder.

626
Q

Who meets the costs of the derivative claim?

A

The applicant shareholder if permission is refused.

The company will meet the costs as well as the other party’s costs if the permission to continue is granted.

627
Q

What is meant by express actual authority?

A

The authority is set out in the director’s service contract or it may have been given following a discussion between the board of directors.

628
Q

When disposing of a chargeable asset, what should a group company consider?

A

Sometimes companies in a group can arrange for the company that will benefit most from a tax perspective to dispose of a chargeable asset owned by another company in the group.

  1. What is a group for the purposes of chargeable gains?

For this relief, the group consists of a company, its direct 75% subsidiaries and the direct 75% subsidiaries of those subsidiaries, and so on.

All of the subsidiaries in the group must be effective 51% subsidiaries of the principal company.

This means that the principal company must be beneficially entitled to more than 50% of the available profits and assets of the subsidiary. The group is only permitted to have one principal company.

A owns 80% of B and B owns 80% of C and C owns 75% of D.

A, B and C are in the same group for the purposes of chargeable gains, because A owns at least 75% of B and B owns at least 75% of C, and A effectively owns 64% of C (80% x
80% = 64%), satisfying the ‘effective 51%’ test. However, D does not form part of the group
because A has an indirect shareholding of only 48% in D (80% x 80% x 75% = 48%), which
means that D is not an effective 51% subsidiary of A.

  1. How does relief for chargeable gains work?

Once it has been established that the two companies are within a group for chargeable gains purposes, one of the companies can transfer a chargeable asset to the other on a tax neutral basis.

This means that the disposal is treated as giving rise to neither a gain nor a loss by the transferor. The transferee can then use the loss to reduce its own chargeable gains, so, as a group, the companies will pay less tax overall.

629
Q

What should be included in a PA regarding expulsion?

A

Under PA 1890, no majority of partners may expel another partner unless the partners have expressly agreed to this.

This effectively means that a partner cannot be expelled without an express agreement allowing the other partners to do so I.e. a partnership agreement.

630
Q

What is the statement of capital?

A

This is information about the shares.

Number of shares of each type the company has and their total nominative value i.e. the company’s share capital.
Names and addresses of all shareholders known as subscribers.

The rights of each type of share AKA the prescribed particulars. This must include what share of dividends they receive, whether they can exchange their shares for money, whether they can vote on certain company matters and how many voted their shares entitle them to.

631
Q

When is a revolving credit facility useful?

A

To businesses that do not have steady income throughout the year.

632
Q

What is a partnership at will?

A

This is the term for a partnership which continues indefinitely until notice is given under the PA 1890.

633
Q

What is the liability between the partners?

A

The partnership agreement should set out a mechanism for valuing an outgoing partner’s share.

Usually when a partner leaves the partnership they will leave in the partnership bank account a sum of money in satisfaction of their share of any outstanding debts.

If the remaining partners and the outgoing partner are sued by a creditor in respect of a debt incurred before the partner left the partnership, the outgoing partner may have to pay the third party because they always remain liable to third parties.

They may have a contractual right to be reimbursed by the remaining partners if such a right is included in the agreement or they could try to claim an indemnity under PA 1890 on the basis that they have incurred liabilities in the ordinary and property conduct of the business.

634
Q

Can all business arrangements enter into debentures?

A

The term can be used to describe a loan agreement in writing between a borrower and lender that is registered at CH. It gives the lender security over the borrower’s assets.

Only companies and LLPs can enter into debentures.

635
Q

Can shareholders vote in shareholder resolutions to promote their own interests?

A

As shareholders they are able to vote to promote their own interests.

636
Q

How is an auditor removed?

A

They can be removed by the shareholder by ordinary resolution.

The shareholders must give special notice to the company of the proposal to remove the auditor.

The auditor can resign at any time by notice in writing sent to the company’s registered office.

The consequences of removal of auditor will depend on the terms of the contract between the company and the auditor.

When an auditor resigns/is removed, they must deliver a statement to the company explaining the circumstances of them leaving.

637
Q

What form needs to be sent to CH to change details of an individual shareholder that is already set out in the PSC register?

A

Form PSC04

638
Q

What types of liquidation are there?

A

a) Compulsory liquidation - a third party commences insolvency proceedings against an insolvent company

b) Creditors’ voluntary liquidation - commenced by the company itself when it is insolvent, usually in response to pressure from creditors

c) Members’ voluntary liquidation - commenced by a solvent company because it wishes to cease trading or because it is dormant and it wishes to bring affairs to an end in an orderly manner.

639
Q

What is the deadline for distribution of a written resolution?

A

The company must circulate a copy of the resolution and any accompanying statement to all eligible shareholders within 21 days of the shareholder’s request.

640
Q

What is a non-executive director?

A

Are appointed by the board and are registered at CH as directors of the company but they do not have service contracts with the company.

They do not have a salary but they will receive director’s fees for attending board meetings.

Directors duties apply to them as much as executive directors.

641
Q

Comparing debt and equity finance, how does tax treatment of income payments differ?

A

Equity finance

The payment of a dividend is not a deductible expense for the company, it is simply a distribution of profit, after it has paid corporation tax.

Debt finance

The payment of debenture interest as it is incurred, is a normal trading expense of the company so it is tax deductible.

642
Q

What is a fixed charge?

A

A fixed charge is taken over property such as machinery and shares.

A separate fixed charge needs to be made over each asset.

The effect of the charge is that the lender has control over the asset e.g. the chargor will not be permitted to dispose of the asset without the charge holder’s consent.

The charge holder will also require the chargor to keep the asset in good condition.

If the chargor goes into receivership or liquidation, the fixed charge holder has the right to sell the asset and be paid out of the proceeds of the sale before any other claims.

It is possible to create more than one fixed charge over the same asset.

The first fixed charge holder will be able to pay itself out of the proceeds and the second will be able to pay itself out of the remainder.

643
Q

When are CVAs used?

A

They are used when the business of the company is fundamentally sound but there are temporary cash flow issues and the company wants to avoid the risk of liquidation.

644
Q

As soon as the bankruptcy order is made, the bankrupt is subject to certain restrictions.

What are the restrictions?

A

There are some restrictions on business activities and personal activities which apply to all bankrupts.

Culpable bankrupts are subject to further restrictions that last beyond the initial year of bankruptcy.

  1. Business restrictions

They can keep items they need for work and their vehicle if it is essential for work.

It is a criminal offence for a bankrupt to obtain credit of more than £500 without disclosing their bankruptcy which makes it difficult for them to carry on their business.

The bankrupt also cannot:

a) Act as a director of a company
b) Be involved in the management, promotion or formation of a company unless the court grants permission
c) Trade under a different name from the name in which the bankruptcy order was made, without disclosing to anyone they trade that they are an undischarged bankrupt
d) Continue in partnership (unless the default position under PA 1890 has been varied which states that a bankrupt partner automatically ceases their function)

  1. Personal restrictions

Undischarged bankrupt cannot obtain more than £500 without informing lender of the fact that they are an undisclosed bankrupt.

A bankrupt cannot obtain a credit card or have a normal current account. The bankrupt’s bank will probably only allow the bankrupt to a current account with an overdraft facility.

They cannot practice as a solicitor without leave of the SRA.

  1. Bankruptcy restrictions orders and undertakings

What are bankruptcy restriction orders?

They are designed to protect the public from those bankrupts who are considered to be culpable and to have caused their own bankruptcy due to negligence or recklessness.

They are made by the court and they last between two and 15 years.

What is a bankruptcy restriction undertaking?

This is an agreement which has the same effect as an BRO but is not through the court.

Implications of a BRO or BRU:

They cannot act as a company director/insolvency practitioner/MP for the length of the order or the undertaking or obtain credit over a certain amount £500) without disclosing the BRO.

645
Q

Avoidance of certain floating charges

When will a floating charge be automatically void?

A

If the charge was granted without the company giving fresh consideration in exchange for granting security at the ‘relevant time’ before the onset of insolvency.

646
Q

Which formalities are required to start a general partnership?

A

None but always a good idea to get legal advice when starting a partnership.

Partners need to be aware of the default terms under the PA 1890 if they do not make an alternative provision.

They should ideally make a partnership agreement

647
Q

How are income profits calculated for corporation tax?

A

The most common type of income which is chargeable to corporation tax is trading profit.

Note that dividends paid to shareholders and a company’s payments to shareholders for shares bought back by the company are not deductible expenditure when calculating a company’s trading profits.

This is Step 1 of corporation tax calculation.

648
Q

When may there be a tax relief for interest paid on money which the tax payer has borrowed?

A

This is relevant for step 2 of the income tax calculation.

To benefit from this relief, interest must be payable on a ‘qualifying loan’, including:
a) a loan to buy a share in a partnership, or to contribute capital or make a loan to a partnership;
b) a loan to invest in a close trading company; and
c) a loan to personal representatives to pay inheritance tax.

649
Q

Does a general partnership have a separate legal personality?

A

No

650
Q

When a director is removed, does this automatically terminate the director’s service contract?

A

Removal of a director will not not terminate the director’s service contract. This can only be terminated in accordance with its terms unless the director is in repudiatory breach of their service contract and can be summarily dismissed on that basis.

Ending a director’s service contract does not necessarily mean that they will be removed from the office of director.

651
Q

What happens if there has been a mistake on the statements or notice that have been delivered to the Registrar of Companies with regards to registration of a charge?

A

If there has been a mistake on the statement or notice delivered to the Registrar, the court can rectify inaccurate details.

652
Q

How is private residence relief applied for CGT?

A

Any gains made by individuals who dispose of a dwelling house, including grounds of up to half a hectare, are exempt from CGT under private residence relief, provided that they have occupied the dwelling house as their only or main residence through their period of ownership.

The last nine months of ownership are ignored.

This enables people to move house without having to pay CGT on the gain.

653
Q

What is the default lapse date for the written resolution procedure for shareholder resolutions?

A

Unless the articles state otherwise the lapse date is 28 days from circulation of the written resolution.

Unlike the with the notice of a general meeting, the method of circulation of the written resolution is irrelevant for the purposes of calculating the lapse date.

The lapse date is the 28th day following circulation of the written resolution regardless of how it was delivered. Generally interpreted as midnight of the 28th day following circulation of the written resolution.

654
Q

What is the role of accountants in creating business accounts?

A

Almost all businesses will have a firm of accountants which they use to help them to prepare
the financial records they need to keep, and advise them on taxation.

Accountants will produce accounts which follow the rules on how financial information is recorded. It is important that this information is standardised to enable those who are viewing accounts to understand the information they contain quickly and easily, and to enable them to compare accounts for the same business across different accounting periods, or to compare the accounts of different businesses.

Most companies have to file audited accounts at Companies House, and these must be examined by a registered auditor, who will provide their opinion as to whether the accounts give a true and fair view of the affairs of the business. Accountants also advise on tax matters such as reliefs and exemptions.

655
Q

What is the general anti- avoidance rule (‘GAAR’)?

A

Was brought into the UK tax system to target abusive tax- avoidance schemes.

The GAAR applies to income tax and also applies to capital gains tax, corporation tax, inheritance tax and National Insurance.

The general tenor of the GAAR is that tax avoidance is acceptable but abusive arrangements are not.

Tax arrangement:

An arrangement is a ‘tax arrangement’ if, having regard to all the circumstances, it would be reasonable to conclude that the obtaining of a tax advantage was the main purpose, or one of the main purposes, of the arrangement.

Abusive:

A tax arrangement is ‘abusive’ if entering into or carrying out the arrangement cannot reasonably be regarded as a reasonable course of action in relation to the relevant tax provisions, having regard to all the circumstances.

Those circumstances include:
- whether the effect of the arrangement is consistent with the policy objectives of the tax legislation;
- whether the means of achieving those results involves one or more contrived or abnormal steps; and
- whether the arrangements are intended to exploit shortcomings or loopholes in the tax legislation.

If the tax arrangements accord with established practice, and HMRC had indicated its acceptance of that practice before the arrangement was entered into, this might indicate that the arrangement is not abusive.

The ‘cannot reasonably be regarded as a reasonable course of action’ element of the test for ‘abusive’ is known as the double reasonableness test, and it allows a wide range of potentially acceptable behaviour on the part of the taxpayer.

656
Q

What is an overdraft facility?

A

This is a contact between the business and its bank which allows the business to go overdrawn in its current account.

It is a type of temporary loan use to cover everyday business expenses when there is no other source of money available.

The maximum amount of the overdraft will be agreed with the bank in advance but the amount used or owe to the business may vary on a day to day basis.

An overdraft is known as an uncommitted facility which means that it will usually be payable on demand without notice.

657
Q

What happens if a director has no authority to make decisions?

A

The director is personally liable to the third party and the company is not party to the contract or liable to the third party.

658
Q

Who can request a s 303 general meeting?

A

Shareholders representing atleast 5% of paid up capital of the company WITH voting rights can request a general meeting.

659
Q

When shares are transmitted, does this mean that the trustee and the PRs automatically become shareholders?

A

No under MA 27, they are not shareholders but they are entitled to any dividends declared on the shares.
They can choose to be registered as shareholders (unless directors can refuse them under the articles) and can then sell the shares or they can sell the shares directly in their capacity as representatives.

660
Q

What is holdover relief on gifts?

A

This relief allows an individual to make a gift of certain types of business asset, or sell them at an undervalue, without paying CGT.

If the donee (the recipient of the assets) disposes of the asset, the donee will be charged to tax on their own gain and the donor’s gain.

  1. Conditions for the relief to apply

It is only available on gifts, or the gift element of a sale at an undervalue.

Only the part of the gain relating to chargeable business assets qualify for the relief.

1.1. ‘Business assets’ include:

a) assets used in the donor’s trade, or their interest in such assets, where the donor is a sole trader or a partner whose assets are being used by the partnership;
b) shares in a trading company which are not listed on a recognised stock exchange (AIM is not deemed to be a stock exchange for the purposes of this condition);
c) shares in a personal trading company, even if the company is listed on a recognised stock exchange. A ‘personal company’ is one in which the donor owns at least 5% of the company’s voting shares; and
d) assets owned by the shareholder and used by their personal trading company.

If the donee is a company, the relief does not apply to a gift of shares.

  1. What agreements need to be reached between the donor and the donee?

Both the donor and the donee must elect to apply for the relief, because the donee is accepting liability for any CGT payable in relation to the gift when the donee disposes of the asset.

The donor and donee must elect for the relief to apply within four years from the end of the tax year of the disposal.

  1. How is the relief applied?

Disposal expenditure (market value) - deemed gain = notional acquisition cost

Donee sale price of asset/market value of gift - notional acquisition cost - qualifying expenditure = donee’s gain (then think of reliefs)

The chargeable gain is calculated by taking market value as the consideration for the disposal.

The deemed gain is then deducted from the market value of the asset to produce an artificially low ‘acquisition cost’ for the donee.

When the donee later disposes of the asset, the notional acquisition cost and any qualifying expenditure is deducted from the sale price or the market value (on a gift) to find the donee’s gain.

The donee’s gain will therefore include the held- over gain plus any gain made during their period of ownership of the asset.

Once the donee’s chargeable gain has been calculated, reliefs should be considered as usual to attempt to reduce or postpone the donee’s tax liability.

If the donee dies before disposing of the asset, all of the gains will escape CGT altogether.

Note that the annual exemption cannot be applied before the gain is rolled over.

661
Q

Once the dividend income has been worked out from taxable income for step 4 of income tax, what needs to be done with the dividend income figure?

A

To work out the dividend income which will be subject to tax, you must deduct the dividend allowance from the dividend income figure.

Dividend income less dividend allowance (taxed at 0%) = ‘remaining taxable dividend
income’

  1. Dividend allowance:

All rates of taxpayers are entitled to the first £500 of dividends free of tax.

  1. Remaining dividend income:

The remaining taxable dividend income is taxed at the dividend ordinary rate, upper rate and
additional rate. For the tax year 2023/24, the rates are as follows:

Dividend ordinary rate of 8.75% = 0 - 37,700
Dividend upper rate of 33.75% = 37,701 - 125,140
Dividend additional rate of 39.35% = over 125,140

To ascertain whether the dividends will be taxed at the dividend ordinary, upper or additional rate, you must add the dividend allowance to total taxable NSNDI and total savings income. To the extent that the dividend income falls within the basic rate limit, it will be taxed at 8.75 per cent. To the extent that dividend income falls between the basic rate limit and the higher rate limit, it will be taxed at 33.75 per cent. Any excess over the higher rate limit will be taxed at 39.35 per cent.

662
Q

What are pre-emption rights?

A

Pre-emption rights are rights of first refusal over shares that are being allotted. The company must not allot equity securities to a person unless it has first offered them to existing holders of ordinary shares in the company on the same or more favourable terms.

The proportion of the new shares offered must equal as much as practicable the proportion of shares they already hold in the company. So if a person has 600 shares equal to 60% of the total shares and 500 new shares are being allotted, the person with the pre-emption rights is entitled to 300 shares as this is 60% of the new shares.

Equity securities:

This includes ordinary shares, right to subscribe for or convert securities into ordinary shares in the company.

Ordinary shares generally give the owner the right to vote at general meetings.

663
Q

In bankruptcy proceeding, who do the trustees owe a duty to?

A

The trustees primary duty is to the creditors and they have powers to investigate the bankrupt’s affairs and challenge past transactions with a view to increasing the assets available to repay creditors.

664
Q

What is income tax payable on?

A

Tax is payable on taxable income.

665
Q

Who must be on the PSC register?

A

A person who owns more than 25% of the shares and controls more than 25% of the voting rights of the company must appear on the PSC register.

666
Q

What is a restructuring plan?

A

This is court supervised and is an arrangement or compromise between the company and all of its secured, unsecured creditors and shareholders.

667
Q

How is corporation tax paid on the seller’s profit of the buyback of shares?

A

When a company sells shares back to the company in which the shares are held, it must work out how any profit on the sale will be treated in its corporation tax calculation.

If the buyback satisfies the CGT rules, the profit will be taxed as part of the selling company’s chargeable gains. If not, it will be taxed as income.

668
Q

Do partners pay income tax?

A

Partners are responsible for the tax due on their individual share of the partnership profits.

The method of calculating trading profit is the same as for sole traders, but the profit will need to be apportioned between the partners in accordance with their shares in the income profits of the partnership.

Note that if a partner is not an individual but a company, that corporate partner will be liable to pay corporation tax rather than income tax on its share of the profits.

669
Q

How much notice should a director give to other directors when calling a board meeting?

A

When a director calls a board meeting, they must give notice to the other directors.

Notice must be reasonable and what is reasonable depends on the facts.

No need for the notice to be in writing but does need to include the time, date and place of the meeting.

If not intended that the directors should meet in the same place, the notice must state the method of communication.

670
Q

What is the default position under statute regarding decision making?

A

Ordinary matters of the LLP can be decided by a majority of the members.

Changing the nature of the business and changing the name of the terms of the contract between the members can only be done by unanimous consent.

671
Q

Under the PA 1890, when can the partners can apply to court to dissolve the partnership?

A

a) Partner becomes permanently incapable of performing their part of the partnership contract
b) A partner’s conduct is calculated to be prejudicial to the business
c) A partner wilfully or persistently breaches the partnership agreement
d) The partnership can only be carried on at a loss or
e) The court thinks that for other reasons it is just and equitable to order that the partnership be dissolved

672
Q

Where can statutory records be kept?

A

a) Company register

b) Single Alternative Inspection Location (SAIL)

c) Central register of Companies House

673
Q

What administration is required in relation to the approval of a long term service contract?

A

When the board proposes an ordinary resolution to vote on a long-term service contract, the company must keep a copy of the memorandum setting out the terms of the proposed service contract at the registered office for 15 days prior to the GM and at the GM itself.

If the OR is proposed by a written resolution, a copy of the memorandum must be circulated to the shareholders along with the written resolution.

All of the service contracts/a memorandum setting out their terms must be available for inspection by the shareholders at the company’s registered office during their term and until a year after termination of the service contract.

Shareholders have a right to inspect them without charge and within seven days of their request.

674
Q

What is the effect of a CVA once it has been approved?

A

It is binding on all unsecured creditors in relation to past debts but not future debts.

675
Q

Which businesses are not eligible for a moratorium?

A

a) Financial services companies such as banks.
b) UK companies that have outstanding winding up petitions against them
c) Non-UK companies

676
Q

What administrative requirements are there when a new member joins an LLP?

A

Must deliver a notice to the Registrar of Companies, notifying them of the new member within 14 days of appointment.

This can be done by filing LL AP01 for an individual member or LL AP02 for a corporate member at Companies House.

The member needs include their service address, residential address, full name, former names and date of birth in the form.

677
Q

What will the court assess in the first stage of derivative claim?

A

The shareholder has to apply to the court to for permission to continue the claim.

The court will consider the claim and the evidence in support of the claim without a hearing.

The court will only allow the application to continue if the application and evince disclose a prima facie case for continuing the claim.

The reason for this is to filter out vexatious litigant shareholders who simply do not agree with a decision of the board.

If the court decides there is a prima facie case, the court may give directions as to the evidence to be provided by the company or adjourn the proceedings to enable evidence to be obtained.

The court will then list a full hearing to determine the shareholder’s application for permission to continue the claim .

678
Q

What forms need to be sent to CH to inform of a change of particulars of directors?

A

CH form CH01 and CH02 are used to notify a change in particulars for natural persons and corporate directors.

679
Q

Who is classed as a person connected with a director in the context of a substantial property transaction?

A

a) Member of family
i) Director’s spouse or civil partner
ii) Director’s child/stepchild
iii) Director’s parents
iv) Any person who lives in an enduring relationship with the director as their partner
v) Any children of a person who lives in an enduring relationship with the director as their partner.

b) Company in which the director or a person/persons connected with a director (or the director and the persons connected with the director taken together)
I) Own(s) at least 20% of the body corporate’s shares AND
ii) Is/are entitled to exercise or control the exercise of more than 20% of the voting power at any general meeting of the company.

So if a company in which the director owns 20% of voting shares buys or sells a non-cash asset of substantial value, this is caught.

680
Q

How is CGT calculated if only part of the asset is disposed of?

A

Where only part of an asset is disposed of, any initial and subsequent expenditure are
apportioned when calculating the gain.

681
Q

What is carry-across/one year carry-back relief for trading losses generally?

A

a) Trading losses can be set against the total income from the same tax year or
b) Set against the total income from the tax year preceding the tax year of the loss
c) Set against the total income from the same tax year until that income is reduced to zero with the balance of the loss being set against total income from the tax year preceding the year of the loss
d) Set against total income from the tax year preceding the tax year of the loss until that income is reduced to zero with the balance of the loss being set against total income from the tax year of the loss

When this reliefs claimed, it must be set against the total income which means that the taxpayer loses their personal allowance.

The claim for the relief must be made on or before the first anniversary of 31 January following the end of the tax year in which the loss is assessed.

Subject to a cap of £50,000 or 25% of the taxpayer’s income in the tax year in which the relief is claimed. But only applies to income from sources other than the trade which produced the loss.

682
Q

What is allotment of shares?

A

When a company decides to create shares and give them to an existing shareholder or a new shareholder in return for payment.

The company will then issue a share certificate relating to the shares and enter the person on the register of members or amend their existing entry to show their increased shareholding.

The form of payment will usually be cash but sometimes consideration will be an object such as property.

683
Q

What is a shadow director?

A

A person with whose instructions or directions the company is accustomed to act, but who have not been formally appointed to as a director of a company.

This could be a major shareholder or a lender or a management constant.

684
Q

What are the requirements for the members’ voluntary liquidation?

A

The directors must first swear a statutory declaration that the company is solvent.

685
Q

What is the first step that needs to be worked out for CGT?

A

Step 1: Disposal (sale or gift) of a chargeable asset.

Identify the disposal of a chargeable asset, for example the sale of a factory.

This can be a sale or a gift.

If someone makes a gift of a chargeable asset, there is no sale price to use as a basis for calculating the gain so HMRC will use the market value of the asset at the time of the gift.

  1. What if the taxpayer only sells or gives away part of an asset?

It is still chargeable to CGT.

  1. Is there CGT when the taxpayer dies?

There is no deemed disposal so no charge to CGT. The PRs are deemed to acquire the deceased’s assets at the market value at the date of death aka probate value.

The gains accrued during the deceased’s lifetime are never charged to tax.

686
Q

Which test will the court apply when considering whether a director has breached the duty to promote the success of the company?

A

The court will apply a subjective test.

So the director may be in breach of one or more of the six factors but they will not be in breach if they considered in good faith that their actions were most likely to promote the company’s success.

If the court is satisfied that the director acted in good faith, even if the court believes that this belief was misguided of that disastrous consequences have resulted from the director’s actions, the director will not be in breach of duty.

687
Q

When is a claim for transaction defrauding creditors made (personal insolvency)?

A

It is only really used when the time limit for applying to set aside the transaction at an undervalue has expired.

688
Q

What are the requirements for buying back shares (private company procedure known as off-market transaction)?

A

The company’s articles must not forbid buy backs

The shares must be fully paid

The company must pay for the shares at the time of the purchase out of distributable profits* or the proceeds of a fresh issue of shares made for the purpose of financing the purchase. The board should also consider whether any upcoming debts and liabilities need to be paid and whether the company will have enough distributable profits for this.

The shareholders must pass an ordinary resolution authorising the buy back contract.

A copy of the buy back contract must be available for inspection atleast 15 days before the general meeting and at the general meeting itself or be send with the proposed written resolution when or before it is circulated

A copy of the buyback contract or if the contract is not in writing a written memorandum setting out its terms must be made available for inspection at the company’s registered office or SAIL as soon as the contract has been concluded for a period of the year starting with the date of buyback

689
Q

What controls are put on directors?

A

Long term service loan

Substantial property transactions

Loans to directors (or guarantee of security in relation to a loan)

Payment for loss of office

690
Q

What liability does a sole trader have?

A

The sole trader is personally liable for all of the debts of the business. The sole trader’s business assets and personal assets are all treated the same for legal purposes. AKA they have unlimited liability.

691
Q

What is a transaction defrauding creditors?

A

This is a transaction at an undervalue with the company has entered into to put the assets beyond the reach of someone making a claim against it or to prejudice the interests of that person in relation to any claim that might make.

It is very difficult to show intention to put assets beyond someone’s reach or to prejudice their interests.

The claim is usually only made when the liquidator or administrator cannot bring a claim relating to a transaction at an undervalue because the time limit has expired.

692
Q

When are term loans used?

A

Typically used by a business to purchase a capital asset such as land, building or machinery.

693
Q

What is the registration and administrative process of registering a charge?

A

Step 1

Within 21 days of creation of the charge, the company or a person interested in the charge (including charge holder) must file at CH:
a) A statement of particulars (usually form MR01),
b) A certified copy of the instrument creating the charge
c) The fee

Step 2

Once the documents have been delivered, the Registrar of Companies registers the charge and includes a certified copy of the charge on the register.

Step 3

The Registrar must give the person who delivered the documents a certificate of registration.

Step 4

Form MR01 and certified copy of the charging document will be put on the company’s file at CH, which is available for public inspection.

If the required documents are correctly delivered on time, the charge will be fully valid against another creditor of the company or administrator or a liquidator of the company.

Step 5

A copy of the charging document and Form MR01 should be kept available for inspection at its registered office or SAIL.

Failure to do this is a criminal offence but it does not affect the validity of the charge.

If a fixed charge is taken over the land, then this must also be registered at the LR, otherwise the buyer could acquire the land without it being subject to the fixed charge even if they knew it existed.

694
Q

At what point does the winding up start in compulsory liquidation?

A

When the petition has been filed at court and served on the company.

695
Q

Can all shareholders vote in the buyback?

A

Shareholders are generally not prevented from voting when they have personal interests.

However, the exception is where a shareholder owns the shares that are subject the buyback.

696
Q

How are losses treated in relation to CGT?

A

A taxpayer’s CGT losses in a tax year may exceed their overall CGT gains.

If so, setting the losses against the gains will wipe them out completely and the taxpayer will have no CGT to pay.

In such a situation, the setting- off of the current year’s losses to wipe out the gains entirely means that the taxpayer loses the use of their annual exemption, which cannot be carried forward to use against gains of future tax years.

What if there are still unabsorbed losses?

If, after setting losses against gains, there are still unabsorbed losses, these may be carried forward to future years and then used to the extent necessary to reduce gains to the limit of the annual exemptions available in those future years. Unabsorbed losses can be carried forward indefinitely.

The approach should be:
(a) work out the gain or loss on each disposal made during the tax year;
(b) deduct any losses of the current year from gains;
(c) deduct any losses brought forward from previous years to reduce any remaining gains to the limit of the annual exemption;
(d) deduct the annual exemption from any remaining gains.

697
Q

When are shares considered to be issued?

A

When the shareholder has been entered on the register of members.

698
Q

Which six factors must directors have regard for to avoid the duty to promote the success of the company?

A

The likely consequences of any decision in the long term

The interests of the company’s employees

The need to foster the company’s business relationships with suppliers, customers and others

The impact of the company’s operations on the community and the environment

The desirability of the company maintaining a reputation of high standards of business conduct

The need to act fairly as between members of the company

699
Q

When must the declaration under the director’s duty to declare interest in a proposed transaction or arrangement be made?

A

The declaration must be made before the company enters into the transaction or arrangement in question.

700
Q

What does the administrative receiver do?

A

They run the company and sell the charged assets and use the proceeds to pay their own costs and pay the charge holder what they are owed under the loan secured by the charge. The administrative receiver then resigns and hands the management of the company back to the directors although it is more likely that the company will go into liquidation.

701
Q

What is a share buyback?

A

This is where the company buys back some of its own shares for one or more shareholders.

Instead of creating new shares, the company reabsorbs some of its shares so the number of shares decreases as the shares that are cancelled.

702
Q

When can companies expense the full amount of their plant and machinery for the purpose of calculating trading profits?

A

This means that the company can deduct the full amount of their expenditure on plant and machinery in an accounting period from chargeable receipts.

On disposal of the asset, a balancing charge is applied equal to 100% of the disposal value where full expensing has been claimed. (Set to end March 2026).

For all expensing to apply, the asset must be brand new.

703
Q

What are share transfer provisions?

A

To avoid any issues between shareholders, a company may want to include share transfer provisions in the articles.

It is common in smaller companies to state that share transfers to family members or other existing shareholders are permitted but that any other transfer must be approved by the board of directors.

Or shareholders may be allowed to transfer their shares to other people but only if they have offered their shares to existing shareholders first at a fair value.

The article cannot restrict a shareholder from selling the shares and cannot stop a particular purchaser from buying them.

704
Q

What is a negative pledge clause?

A

A floating charge ranks behind a later fixed charge or mortgage over the same asset, provided that later fixed charge or mortgage is properly registered.

In order to prevent a floating charge ranking behind other charges, it is usual to include a negative pledge clause in the floating charge documentation.

This clause prevents the company from creating any later charges with priority to the floating charge without the floating charge holder’s permission.

If a subsequent lender takes a charge over the same asset and has actual knowledge of the negative pledge clause, then the subsequent lender’s fixed charge will be subordinate to the original floating charge.

Constructive knowledge is not enough e.g. existence of negative pledge revealed by Form MR01.

Actual knowledge can be gained if the subsequent charge holder conducts a search of the company’s records at CH (usually part of the process of checking if a potential borrower is suitable) and it comes across the charging document containing this clause.

705
Q

How is the onset of insolvency defined for a preference?

A

Liquidation (CVL):

The date the company formally entered into liquidation.

Administration:

The date that the company files a notice of intention to appoint an administrator or the date it actually goes into administration if that is earlier.

706
Q

Roughly how is income tax calculated?

A

Step 1: Calculate total income
Step 2: Deduct any allowable reliefs = The resulting sum is net income
Step 3: Deduct any personal allowances = The resulting sum is taxable income
Step 4: From the taxable income sreparate NSNDI, savings income and dividend income, and calculate the tax on each type of income at the applicable rate(s) (starting rate, basic rate, higher rate and additional rate)
Step 5: Add together the amounts of tax from Step 4 to give the overall income tax liability

707
Q

Avoidance of certain floating charges

How will the liquidator/administrator commence proceedings?

A

They will write to the charge holder and say that they believe the charge is invalid.

If the charge holder tries to enforce the charge, the liquidator or administrator will seek an injunction on the basis that the charge is invalid.

708
Q

Who is responsible for the circulation of accounts, directors’ report and auditor’s report?

A

It is the director’s responsibility to circulate to every shareholder and debenture holder and anyone else who is entitled to receive notice of general meetings.

709
Q

What is a de facto director?

A

A person who acts as a director although they have never been appointed as a director.

710
Q

How is a restructuring plan initiated?

A

The directors will usually prepare a restructuring plan and apply to court for approval to call meetings of the company’s creditors and shareholders.

Creditors and shareholders are also allowed to do this but in practice, the directors will do it because they have detailed information needed for the meetings.

711
Q

How can the name of an LLP be changed?

A

An LLP can change its name at any time with the consent of all members or the procedure for changing name may be set out in the PA.

712
Q

What are the consequences of a share buyback for the company?

A

The company essentially has to pay the shareholders to buy the shares so the company is worse off financially. The shares are then cancelled.

713
Q

If the company keeps the register of members at registered office or SAIL, what must the company ensure?

A

It must be available for inspection to shareholders free of charge and to anyone else for a fee.

Failure to allow this is a criminal offence.

714
Q

Which sections of the PA 1890 cannot be overridden by a partnership agreement?

A

Ss1 and ss2 - which govern when a partnership can come into existence

Ss 5-18 - covers the relationship between partners and third parties and liability for debts

715
Q

Comparing debt and equity finance, what is the involvement in the company?

A

Equity finance

Dependant on the type of shares, the shareholders will have the right to attend general meetings and vote which enables them to influence the direction of the company.

Debt finance

A lender does not have any ownership rights and has no say in the way the company is run, provided the company sticks to the terms of the facility agreement.

716
Q

Who needs to be VAT registered?

A

Anyone making taxable supplies of more than £90,000 in any 12 month period must register and charge VAT and those making less than this can choose to register but are not obliged to.

Only those registered can reclaim input tax.

Need to evaluated weather being able to reclaim input tax is worth losing the advantage of being able to undercut VAT registered rivals when competing for business.

A partnership can be registered for VAT in the name of the partnership itself (unusual as this is not the position for other taxes).

HMRC will issue each person registering with a VAT number which applies to all businesses operated by that person.

Subject to a cap of £50,000 or 25% of the taxpayer’s income in the tax year in which the relief is claimed. But only applies to income from sources other than the trade which produced the loss.

717
Q

How can the trustee disclaim onerous property in order to preserve and increase the bankrupt’s property?

A

These are unprofitable contracts, land the has the burden of an onerous contract or a lease that does not have a capital value to be realised for the creditors’ benefit.

The trustee’s disclaimer means that all of the bankrupt’s rights and liabilities in respect of the onerous property come to an end and the trustee is discharged from personal responsibility of the property

If anyone suffers a loss as a result of the disclaimer, they can prove that they are an unsecured creditor of the bankrupt so they can make a formal claim against them as creditor.

718
Q

How is an auditor appointed?

A

The directors of a private company usually appoint the company’s first auditor.

After that, the shareholders have the power to appoint the auditor by ordinary resolution.

The terms of appointment are a matter of negotiation for the auditor and the company.

An auditor of a private company are deemed to be reappointed every year.

719
Q

What key terms will there be in a charging document?

A

a) Security

It will state that the borrower charges the property to the lender and what type of security it is.

The security might be in the form of a fixed charge or a floating charge or might be a combination of the two.

The specific assets subject to each charge will be listed.

b) Representations and warranties

The borrower will have to make a series of contractual statements relating to the assets which it is charging.

This is done with the intention of getting the borrower to reveal all relevant information about the assets.

They will have to warrant that the property is free of any other charge. If there is, the borrower must disclose this and if it does not the lender has a right to terminate the loan agreement.

c) Covenants

The covenants will seek to ensure that the value of the assets is maintained by the borrower.

d) Enforcement and powers

The agreement would set out the circumstances in which the security becomes enforceable.

It will set out lender’s powers including the power to sell the assts which are the subject of the charge to recover the debt due.

720
Q

What type of resolution is required to authorise a buyback out of share capital?

A

A payment out of capital must be authorised by a special resolution in addition to the ordinary resolution that the shareholders must pass to authorise the buyback contract.

Again, if in a written resolution holders of the shares being bought are not eligible to vote.

If at a GM, the votes of the shareholders who are eligible to vote will not be counted if those votes make the difference between the resolution passing or not.

721
Q

Which factors help determine if there is a partnership?

A

a) The individuals take part in the decision making
b) Their names are on the title deeds of any property
c) The profits are shared

The partners can be companies as well as individuals.

722
Q

What is a special notice in the context of the removal of a director?

A

A special notice is a notice given to the company at least 28 days before the GM at which the resolution is proposed.

Once the company has received the special notice, it must inform the director in question of the notice and let the other shareholders know of the GM and the proposed resolution.

If it is not practicable to let the shareholders know in the same way as the director, the company must give its shareholders notice at least 14 days before the GM by advertisement in a newspaper having an appropriate circulation or any other manner allowed by the company’s articles.

The director can speak at the GM and require the company to send copies of any written representations the director wishes to make to the shareholders.

723
Q

What needs to be kept available for inspection following authorisation of share buyback out of share capital?

A

Directors’ statement and auditors’ report kept available for inspection

This must be at the company’s registered office from the time the company publishes its first notice until five weeks after the passing of the SR.

724
Q

What are the disadvantages of a security of a loan for a borrower?

A

The downside is that the business must accept restrictions on the use of the asset or assets it has given as security.

725
Q

What are the limits to a shareholder agreement?

A

It cannot restrict shareholders from voting a particular way in board meetings if they are also a director because this could lead to the shareholder being in breach of their directors’ duties.

726
Q

What are the consequences if the register of members is incomplete or incorrect?

A

It is a criminal offence if the register of members is incomplete or incorrect, including where there is no reference to the fact that it is a one-member company.

727
Q

What should be included in a PA regarding commencement and duration?

A

A partnership begins when the definition of a partnership is satisfied not when the partners decide it has commenced. Their solicitor should advise them of this as the terms of the PA 1890 will apply until the PA comes into place.

However, it is still good to include a commencement date to make it clear from which date the rights in the PA start from.

Some partnership agreements are fixed term so it will be clear from the terms of the contract when the partners expect the partnership to end.

If the partners carry on the business after the expiry of the fixed term and no new agreement is made, they will be presumed to be partners on the same terms as before.

728
Q

Who starts compulsory liquidation?

A

A third party, usually a creditor, presents a winding up petition at court.

The third party is known as the petitioner.

729
Q

What should be included in a PA regarding ownership of assets?

A

The partners should ensure that the partnership agreement sets out how the assets the partnership uses are owned.

Buildings and land will have title deeds showing who is the legal owner of the property although there may be a dispute as to who the beneficial owners are. Assets such as stock and machinery will not have title deeds.

730
Q

What is done at Step 5 of a CGT calculation?

A

The correct tax rate is applied.

Capital gains are treated as if they were the top slice of the taxpayer’s income for the tax year.

  1. Categories of chargeable asset

a) assets which are not residential property and do not qualify for business asset disposal relief. These are taxed at 10% or 20%, depending on the taxpayer’s income;
b) Assets which qualify for business asset disposal relief, which are taxed at a flat rate of 10%; and
c) residential property, which attracts a surcharge of 8% for basic rate tax payers and 4% for higher rate taxpayers on top of the normal rates of 10% or 20%.

  1. Calculating CGT when there is more than one disposal

Need to separate out the chargeable assets into different categories so that the correct tax rate can be applied.

Any losses and the annual exemption can be deducted from gains in the best way possible for the taxpayer.

a) Losses and the annual exemption should be deducted from gains that would otherwise be subject to higher tax rates. It makes sense, therefore, to deduct them from residential property gains first, then gains on assets which do not qualify for business asset disposal relief and, lastly, gains on assets which do qualify for business asset disposal relief.

731
Q

What is the procedure for an IVA?

A

a) Debtor appoints a nominee to act for them
b) Debtor prepares a statement of affairs for the nominee to consider
c) Debtor should then apply to the court for an interim order to obtain a moratorium. This is usually in force for 14 days and no other proceedings or enforcement action can be taken or continued against the debtor.
d) The nominee prepares a report for the court stating whether the debtor has put forward any realistic proposals and whether they are prepared to support the calling of a creditors’ decision making process.
e) Approval of proposals are needed from 75% or more of the creditors in value or which at least 50% in value are not associates of the debtor. f) The Insolvency Practitioner who is the chair of the meeting decides if the a creditor is an associate of a debtor.
g) The nominee is now called the supervisor and will implement the proposals.

732
Q

How is a board resolution passed?

A

Passed by a simple majority - over half of those present must vote in favour in order for the board resolution to be passed.

733
Q

Can anyone be a director of a company?

A

Every company needs to have atleast one director who is a natural person, so not a company and is over 16.

734
Q

What is a debt relief order?

A

The debtor will be protected from enforcement action by most of their creditors in the same way as bankruptcy and will be free of debt at the end of the DRO (12 months unless extended due to non-cooperation of debtor).

The debtor must cooperate with the Official Receiver and provide information they request,
and is expected to pay their creditors if their financial situation improves.

This is applied for online. These can only be used by debtors whose assets are low in value.

735
Q

What is the period of acceptance of pre-emption rights?

A

The offer must state the period of acceptance and the offer must not be withdrawn in that period.

The period of acceptance cannot be less than 14 days.

736
Q

What are the director’s duties?

A

Duty to act within powers - s 171

Duty to promote the success of the company - s 172

Duty to exercise independent judgement - s 173

Duty to exercise reasonable care, skill and diligence - s 174

Duty to avoid conflicts of interest - s 175

Duty not to accept benefits from third parties - s 176

Duty to declare interest in a proposed transaction or arrangement - s 177

Declaration of interest in existing transaction or arrangement - s 182

737
Q

What form needs to be sent to CH to change details of an legal entity that is already set out in the PSC register?

A

Form PSC05

738
Q

Who uses business accounts?

A

Business owners will need accounts to assess the financial state of the business and plan for the future. There are many other people who will need to see a business’s accounts.

For example,
a) bankers will ask to see business accounts when they are deciding whether to lend money to the business, and
b) solicitors will want access to a business’s accounts if their client is buying the business.
c) HMRC will need a copy of the business’s accounts to assess tax, and
d) companies usually have to send a copy of their accounts to Companies House every year.

739
Q

What happens when the liquidator has finished distributing debts to company’s creditors?

A

They will prepare final accounts and the liquidator will apple to be released.

The Registrar of Companies dissolves the company three months later.

740
Q

Who may register a charge?

A

The company or a person interested in the charge may decide to register it.

Lender’s solicitor will usually do this as the lender has the most at stake.

741
Q

How is a change of partnership accounted for in terms of taxation of the partnership?

A

New partners and retiring partners will be assessed to income tax differently, because for them, the business is either a new one or is coming to an end.

From 2024/25, profits and losses will be apportioned between the different tax years where a business’s accounting period does not coincide with the tax year.

When partners join or leave a partnership part way through a tax year, their tax liability will be calculated using this apportionment principle.

742
Q

What is the exception to the rule that a director has a duty not to accept benefits from third parties

A

There is no breach if the acceptance of the benefit cannot reasonably be regarded as likely to give rise to a conflict of interest.

Normal corporate hospitality is not caught by this duty as long as, viewed objectively, it cannot be reasonably regarded as likely to give rise to a conflict of interest.

743
Q

Will a non-compete clause be implied under PA 1890?

A

Yes, in the absence of an express agreement.

744
Q

Is holiday entitlement, sickness and maternity and paternity provisions set out in the PA 1890?

A

No, need to set this out in an express agreement.

745
Q

Are shareholders prevented from counting or voting in a quorum if they have a personal interest in the matter?

A

No

However, there are two key shareholder resolutions where the votes of a shareholder with a personal interest in the matter are not counted.

746
Q

For roll over relief what is the time limit for acquiring the replacement asset?

A

The taxpayer must acquire the replacement asset within one year before or three years after the disposal of the original asset, unless HMRC allows the taxpayer an extended time period to claim.

747
Q

What does a company need to consider when it comes to choose a name on incorporation?

A

Suffix

Need the correct ending (ltd, plc).

Same as existing company

Should not be the same as an existing company so this must be looked up on the CH website. A name can be deemed to be the same if they effectively mean the same thing e.g. using the symbol instead of the word.

Exceptions for name being same as an existing company

However, it can be the same as another company if it is part of the same group of companies. Or if it is an LLP and it has written conformation that the company or LLP has no issue with it using the same name.

Restricted or prohibited names

Some names are prohibited or restricted. E.g. offensive names. Some names require SoS approval if they suggest a governmental connection. Some sensitive words or restrictions need to be approved by the SoS before they can be used e.g. words that refer to a geographical area or that denote regulated professions.

Other

Certain letters, characters, signs, symbols and punctuation cannot be used in the company name.

Length

The name of the company cannot exceed 160 characters.

Trading names

A company can use a different trading name from their registered name. This can be decided once the company is incorporated so it does not need to be on the IN01 form.

Trademark infringement and passing off

If the owner of a company makes a complaint about the similarity with a company name, the company may have to change its name.

A trademark search should therefore be performed when choosing the company name.

The owner of the trademark could also bring a claim in trademark infringement.

Even if the company name is not a registered trademark, they could bring an action against the client in the tort of passing off.

Fraud

CH can reject an application to register a name where it believes that the name is intended to facilitate fraud.

Computer code

CH can reject an application to register a name where it is comprised of or contains a computer code.

Connected to foreign government

CH can reject an application to register a name that is likely to give the false impression that the company is connected to a foreign government and an international organisation who’s members include two or more countries or territories or their governments.

748
Q

Under what circumstances can a CH direct a company to change their name?

A

Companies house can direct companies to change their name in a wider range of circumstances e.g. where the name has been used or is intended to be used by the company to facilitate fraudulent activity.

If a company fails to change its name within 28 days of being directed to do so Companies House can choose a name for the company e.g. changing the company name to its registered company number.

Companies House also has the power to suppress a name from the register while a company responds to a direction to change its name.

It is an offence if a company does not respond to a direction to change the company name within 28 days or if the company continues to use a company name which Companies House has directed the company to change.

Despite Companies Houses’ new powers in relation to company names, it is the Company Names Tribunal not Companies House which is responsible for considering objections to the use of a company name which is the same as an existing name in which another person has goodwill or a name which is sufficiently similar to that name that is likely to mislead.

749
Q

What kind of an address does a registered address have to be?

A

Companies must ensure that their registered office is an appropriate address which means an address where any documents delivered there would be expected to come to the attention of a person acting on behalf of the company.

PO Boxes are not permitted.

750
Q

What are the consequences if a company’s registered address does not meet the CH requirements?

A

CH is permitted to change the registered office of the company and in practice this will be changed to the CH address.

Both the company and the directors will be guilty of an offence which is punishable by a fine if the company does not have an appropriate registered office address.

751
Q

What email address needs to be included in an IN01 form?

A

Companies must include an appropriate email address for correspondence when they complete IN01.

An email address is an appropriate email address if in the ordinary course of events emails sent to it by the registrar would be expected to come to the attention of a person acting on behalf of the company.

752
Q

What is required when the IN01 has been completed?

A

The applicant will have to complete the statement of lawful purposes in Part 9 of the form to confirm that the company is being formed for lawful purposes.

753
Q

Do LLPs also require an appropriate email address as well as companies?

A

Yes

754
Q
A